Re: [QE-users] Issues with vc-relax for a slab

2024-01-26 Thread Thomas Brumme
Dear Anupriya,


Is there a specific reason why you chose to shift the k-point grid away from 
Gamma?

Also, I didn't look at the geometry in detail but be aware that you also might 
need to
add the dipole correction if your surface has a dipole. Add tefield and 
dipfield to the
 section and edir=3, eamp=0.0, to the  section.

Or consider using the option assume_isolated='2D'


Maybe this will also help with the convergence.


Kind regards


Thomas

--
Dr. rer. nat. Thomas Brumme
Theoretical chemistry
TU Dresden - KOE / 312
Bergstr. 66c
01069 Dresden

Tel:  +49 (0)351 463 39449

email: thomas.bru...@tu-dresden.de



From: users  on behalf of Anupriya 
Nyayban 
Sent: Friday, January 26, 2024 11:47
To: users@lists.quantum-espresso.org
Subject: Re: [QE-users] Issues with vc-relax for a slab

Dear Chiara Cignarella,

Yes, this has a vacuum of 20 Angstrom along the z direction.
 I will check by reducing the mixing_beta first and then check with 
mixing_mode/ mixing_ndim.

Thank you!

with regards
Anupriya Nyayban
Postdoc, IIT Madras

On Fri, Jan 26, 2024 at 3:58 PM Anupriya Nyayban 
mailto:anupriya...@phy.nits.ac.in>> wrote:
Dear Dr. Chirantan Pramanik,

Thank you for the reply!
No, I haven't done vc-relax for a surface earlier but with relax also I am 
facing a similar problem. However, I will try 'relax calculation' by increasing 
the electron_maxstep.

with regards
Anupriya Nyayban
Postdoc, IIT Madras


On Fri, Jan 26, 2024 at 12:44 PM Anupriya Nyayban 
mailto:anupriya...@phy.nits.ac.in>> wrote:
Dear users and experts,

I have considered a (311) plane of 2 layers of ZnFe2O4 structure. The attached 
input file is for the vc-relax calculation.


  calculation='vc-relax'
  restart_mode='from_scratch',
  prefix='zn'
  pseudo_dir = '.'
  outdir='tmp'
  verbosity='high'
  etot_conv_thr = 1.0D-5
  forc_conv_thr = 1.0D-4
/

  ibrav = 0
  A =   21.62370
  nat = 112
  ntyp = 3
  ecutwfc = 30.0,
  ecutrho = 240.0,
  occupations = 'smearing',
  smearing = 'mv',
  degauss = 0.02,
  vdw_corr  = 'DFT-D3'
/

   conv_thr =  1.d-9
   mixing_beta = 0.3
  electron_maxstep = 300
  startingpot = "atomic"
  startingwfc = "atomic+random"
/

  ion_dynamics = "bfgs"
/

  cell_dynamics = "bfgs"
  press_conv_thr = 5.0e-01
/
CELL_PARAMETERS {alat}
  1.000   0.000   0.000
  0.29977738132   0.331662474918549   0.000
  0.000   0.000   2.019068156482969
ATOMIC_SPECIES
  Zn   65.38000  Zn_ONCV_PBE-1.0.upf
  Fe   55.84500  Fe_ONCV_PBE-1.0.upf
   O   15.99900  O_ONCV_PBE-1.0.upf
ATOMIC_POSITIONS {crystal}
Zn   0.8636360   0.9545450   0.4757970
Zn   0.8636360   0.4545450   0.4757970
Zn   0.3636360   0.4545450   0.4757970
Zn   0.3636360   0.9545450   0.4757970
Zn   0.5454540   0.1818190   0.4876030
Zn   0.0454550   0.1818180   0.4876030
Zn   0.0454550   0.6818180   0.4876030
Zn   0.5454550   0.6818180   0.4876020
Zn   0.5909090   0.8636360   0.5230200
Zn   0.5909090   0.3636360   0.5230200
Zn   0.0909090   0.3636360   0.5230200
Zn   0.0909090   0.8636360   0.5230200
Zn   0.2727270   0.0909090   0.5348250
Zn   0.7727270   0.0909090   0.5348260
Zn   0.7727270   0.5909090   0.5348260
Zn   0.2727270   0.5909090   0.5348250
Fe   0.9545460   0.0681820   0.4817000
Fe   0.7045450   0.3181820   0.4817000
Fe   0.2045460   0.5681820   0.4817000
Fe   0.5909090   0.8636360   0.4580880
Fe   0.2045460   0.8181810   0.4817000
Fe   0.7045450   0.5681820   0.4817000
Fe   0.5909090   0.3636360   0.4580880
Fe   0.9545450   0.5681820   0.4817000
Fe   0.7045460   0.8181820   0.4817000
Fe   0.2045450   0.0681820   0.4817000
Fe   0.2045450   0.3181820   0.4817000
Fe   0.7045460   0.0681810   0.4817000
Fe   0.4545450   0.0681820   0.4817000
Fe   0.0909090   0.8636370   0.4580880
Fe   0.0909090   0.3636360   0.4580880
Fe   0.4545450   0.5681820   0.4817000
Fe   0.6818180   0.9772720   0.5289220
Fe   0.4318180   0.2272730   0.5289220
Fe   0.9318180   0.4772730   0.5289230
Fe   0.3181820   0.7727270   0.5053110
Fe   0.9318180   0.7272720   0.528923

Re: [QE-users] wrong plane average electrostatic potential. !! Sorry, I forgot to upload the files in the first message.

2024-01-13 Thread Thomas Brumme
Dear Wilbur,

Can you please explain what should be wrong? You have the same number of 
minima, the relative depths seem to match. The absolut position is not 
important but only relative energies.

Cheerio

Thomas

From: users  on behalf of Wilber 
Muriel 
Sent: Saturday, January 13, 2024 5:33:54 AM
To: Quantum ESPRESSO users Forum
Subject: [QE-users] wrong plane average electrostatic potential. !! Sorry, I 
forgot to upload the files in the first message.

Dear QE users and depeveloper:

I am trying to reproduce the planar average electrostatic potential of In2Se3, 
as in figure 3B of the article 
https://www.frontiersin.org/articles/10.3389/fchem.2023.1278370/full, but I get 
a wrong result. Attached are the input files and the graph I get.

I don't know if the error is due to my input, or some software bug.

Can someone tell me if I am doing something wrong?
___
The Quantum ESPRESSO community stands by the Ukrainian
people and expresses its concerns about the devastating
effects that the Russian military offensive has on their
country and on the free and peaceful scientific, cultural,
and economic cooperation amongst peoples
___
Quantum ESPRESSO is supported by MaX (www.max-centre.eu)
users mailing list users@lists.quantum-espresso.org
https://lists.quantum-espresso.org/mailman/listinfo/users


Re: [QE-users] How does one calculate dipole moments or access (readable) wave functions from e.g. PW?

2023-08-09 Thread Thomas Brumme
Thank you Stefano! I was actually searching for a better reference but 
did not think about your Rev. Mod. Phys. which I usually suggest to our 
students for reading and understanding a bit more about DFPT. Concerning 
optical properties also this might be interesting: 
https://journals.aps.org/rmp/abstract/10.1103/RevModPhys.74.601


On 8/9/23 11:30, Stefano Baroni wrote:


Hi All. Many thanks for this interesting discussion.

Thomas: the literature you recommend 
(https://pubs.rsc.org/en/content/articlelanding/2023/CP/D2CP05991F) is 
rather ill informed. It overlooks the fact that the problem of a 
proper definition of the position operator in the calculation of 
dielectric and optical properties is as old (actually older) than 
density-functional perturbation theory. None of the original 
literature is properly cited. A better informed reference, with a 
historically accurate account of the relevant literature is 
https://journals.aps.org/rmp/abstract/10.1103/RevModPhys.73.515


Best regards – SB

*From: *users  on behalf of 
Thomas Brumme 

*Date: *Wednesday, 9 August 2023 at 11:13
*To: *Quantum ESPRESSO users Forum , 
Brian de Keijzer 
*Subject: *Re: [QE-users] How does one calculate dipole moments or 
access (readable) wave functions from e.g. PW?


Dear Brian,

Sorry to be a bit blunt but your question cannot be answered in a
one-liner and indicates that
you might miss a few important points.

I guess you have read the input description of bands.x and since you're
are still asking how to
get the dipole matrix elements you are probably unaware of the fact that
those are not well
defined in solids. Well, at least the position operator is not well
defined in an infinite solid
because every point could be the origin. Thus, one has to use the
momentum operator instead.
(see, e.g., Eq. (7) in this paper
https://journals.aps.org/prl/pdf/10.1103/PhysRevLett.80.1800)

I suggest to read some publications first, books, in order to understand
what you need to do.
This paper seems quite useful - I didn't read it carefully though but
just found it via a quick
Google search:

https://pubs.rsc.org/en/content/articlelanding/2023/CP/D2CP05991F

If you're interested in optical properties then there are many more
things to take into account.
A good starting point might be the pages of the Yambo code, in
particular this:

https://www.yambo-code.eu/wiki/index.php/Main_Page

and more specifically the thesis of Andrea Marini has a good collection
of the equations and
references

https://www.yambo-code.eu/wiki/images/2/20/Marini_PhD.pdf

In a nutshell: you need to calculate the dielectric function and then
there are different
approximations involved.

Kind regards

Thomas

On 8/9/23 09:27, Brian de Keijzer wrote:
> Hi all,
>
>
> Would anyone know how one retrieves the dipole matrix elements from 
bands.x (or in another way?)?

>
> Any help is appreciated.
>
>
> Bests,
>
> Brian de Keijzer
>
>> On 25 Jul 2023, at 16:09, Brian de Keijzer 
 wrote:

>>
>> Hi Paolo,
>>
>> Thank you for your answer’
>>
>> Bands.x had caught my eye before but I did not notice how to make 
it output transition dipole moments. How would one compute those?

>>
>> Meanwhile I managed to indeed get hdf5 output. That’s great! If 
there is a native way to obtain the values I want it’s preferable tho.

>>
>> Best regards,
>>
>> Brian de Keijzer
>>
>>> On 25 Jul 2023, at 13:37, Paolo Giannozzi 
 wrote:

>>>
>>> I think code "bands.x" can compute dipole matrix elements.
>>>
>>> About direct access to wavefunctions: the default fortran binary 
format is basically unreadable except via the routines of QE. One may 
optionally write hdf5 files that are much easier to read with external 
utilities, though.

>>>
>>> Paolo
>>>
>>>> On 7/25/23 10:40, Brian de Keijzer wrote:
>>>> [You don't often get email from briandekey...@hotmail.com. Learn 
why this is important at https://aka.ms/LearnAboutSenderIdentification ]

>>>> Hi all,
>>>> New QE users here. I’ve used QE so far to compute band structures 
for crystals. I am however very much interested in calculating the 
dipole matrix elements for said crystals. Does QE have a module that 
allows one to do this? If so, how would that work?
>>>> Moreover, I have read that it is possible to use the wave 
functions directly. That would allow me to compute the dipole 
elements. Unfortunately most posts that I come across appear to be a 
bit dated tho. What would be the best way to access the wave functions 
using more recent versions of e.g. Python? I’ve come across postqe but 
that appears to no longer be in development nor does the installation 
work on my Python 3.9 distribution.
>>>> Anyhow; I’m looking to learn how to compute k-dependent 
tra

Re: [QE-users] Errors of phonon calcualtions with Grimme's DFT-D3

2023-08-09 Thread Thomas Brumme
            0.00 1.5955976180        4.5130315840    0   0   0
Pd           -2.7636561440 6.3823904740        4.5130315840    0   0   0
Pd            5.5273122870 1.5955976180        4.5130315840    0   0   0
Pd            2.7636561440 6.3823904740        4.5130315840    0   0   0
Pd           -0.00 4.7867928550        2.2565157920    0   0   0
Pd            1.3818280720 2.3933964280        2.2565157920    0   0   0
Pd            2.7636561400 0.00        2.2565157920    0   0   0
Pd           -1.3818280720 2.3933964280        2.2565157920    0   0   0
Pd            4.1454842150 2.3933964280        2.2565157920    0   0   0
Pd            0.00 0.00        2.2565157920    0   0   0
Pd           -2.7636561440 4.7867928550        2.2565157920    0   0   0
Pd            5.5273122870 0.00        2.2565157920    0   0   0
Pd            2.7636561440 4.7867928550        2.2565157920    0   0   0
Pd           -0.00 3.1911952370        0.00    0   0   0
Pd            5.5273122870 3.1911952370        0.00    0   0   0
Pd            1.3818280720 0.7977988090        0.00    0   0   0
Pd           -1.3818280720 5.5845916650        0.00    0   0   0
Pd            6.9091403590 0.7977988090        0.00    0   0   0
Pd            4.1454842150 5.5845916650        0.00    0   0   0
Pd            4.1454842150 0.7977988090        0.00    0   0   0
Pd            1.3818280720 5.5845916650        0.00    0   0   0
Pd            2.7636561440 3.1911952370        0.00    0   0   0
K_POINTS automatic
  6 6 1   0 0 0




Jibiao Li

Department of Materials Science and Engineering

Yangtze Normal University

Juxian Avenue 16, Fuling, Chongqing, China 408100

Scopus Research ID: 54944118000 
<https://www.scopus.com/authid/detail.uri?authorId=54944118000>


Web of Science Research ID: 
http://www.webofscience.com/wos/author/record/GLS-7259-2022




___
The Quantum ESPRESSO community stands by the Ukrainian
people and expresses its concerns about the devastating
effects that the Russian military offensive has on their
country and on the free and peaceful scientific, cultural,
and economic cooperation amongst peoples
___
Quantum ESPRESSO is supported by MaX (www.max-centre.eu)
users mailing listus...@lists.quantum-espresso.org
https://lists.quantum-espresso.org/mailman/listinfo/users


--
Dr. rer. nat. Thomas Brumme
Theoretical chemistry
TU Dresden - KOE / 103
Bergstr. 66c
01069 Dresden

Tel:  +49 (0)351 463 39449

email:thomas.bru...@tu-dresden.de
___
The Quantum ESPRESSO community stands by the Ukrainian
people and expresses its concerns about the devastating
effects that the Russian military offensive has on their
country and on the free and peaceful scientific, cultural,
and economic cooperation amongst peoples
___
Quantum ESPRESSO is supported by MaX (www.max-centre.eu)
users mailing list users@lists.quantum-espresso.org
https://lists.quantum-espresso.org/mailman/listinfo/users

Re: [QE-users] How does one calculate dipole moments or access (readable) wave functions from e.g. PW?

2023-08-09 Thread Thomas Brumme

Dear Brian,

Sorry to be a bit blunt but your question cannot be answered in a 
one-liner and indicates that

you might miss a few important points.

I guess you have read the input description of bands.x and since you're 
are still asking how to
get the dipole matrix elements you are probably unaware of the fact that 
those are not well
defined in solids. Well, at least the position operator is not well 
defined in an infinite solid
because every point could be the origin. Thus, one has to use the 
momentum operator instead.
(see, e.g., Eq. (7) in this paper 
https://journals.aps.org/prl/pdf/10.1103/PhysRevLett.80.1800)


I suggest to read some publications first, books, in order to understand 
what you need to do.
This paper seems quite useful - I didn't read it carefully though but 
just found it via a quick

Google search:

https://pubs.rsc.org/en/content/articlelanding/2023/CP/D2CP05991F

If you're interested in optical properties then there are many more 
things to take into account.
A good starting point might be the pages of the Yambo code, in 
particular this:


https://www.yambo-code.eu/wiki/index.php/Main_Page

and more specifically the thesis of Andrea Marini has a good collection 
of the equations and

references

https://www.yambo-code.eu/wiki/images/2/20/Marini_PhD.pdf

In a nutshell: you need to calculate the dielectric function and then 
there are different

approximations involved.

Kind regards

Thomas

On 8/9/23 09:27, Brian de Keijzer wrote:

Hi all,


Would anyone know how one retrieves the dipole matrix elements from bands.x (or 
in another way?)?

Any help is appreciated.


Bests,

Brian de Keijzer


On 25 Jul 2023, at 16:09, Brian de Keijzer  wrote:

Hi Paolo,

Thank you for your answer’

Bands.x had caught my eye before but I did not notice how to make it output 
transition dipole moments. How would one compute those?

Meanwhile I managed to indeed get hdf5 output. That’s great! If there is a 
native way to obtain the values I want it’s preferable tho.

Best regards,

Brian de Keijzer


On 25 Jul 2023, at 13:37, Paolo Giannozzi  wrote:

I think code "bands.x" can compute dipole matrix elements.

About direct access to wavefunctions: the default fortran binary format is 
basically unreadable except via the routines of QE. One may optionally write 
hdf5 files that are much easier to read with external utilities, though.

Paolo


On 7/25/23 10:40, Brian de Keijzer wrote:
[You don't often get email from briandekey...@hotmail.com. Learn why this is 
important at https://aka.ms/LearnAboutSenderIdentification ]
Hi all,
New QE users here. I’ve used QE so far to compute band structures for crystals. 
I am however very much interested in calculating the dipole matrix elements for 
said crystals. Does QE have a module that allows one to do this? If so, how 
would that work?
Moreover, I have read that it is possible to use the wave functions directly. 
That would allow me to compute the dipole elements. Unfortunately most posts 
that I come across appear to be a bit dated tho. What would be the best way to 
access the wave functions using more recent versions of e.g. Python? I’ve come 
across postqe but that appears to no longer be in development nor does the 
installation work on my Python 3.9 distribution.
Anyhow; I’m looking to learn how to compute k-dependent transition dipole 
moments in QE. It would be awesome if anyone would want to provide an outline 
of how such a thing is to be done using the latest version of QE.
Bests,
Brian de Keijzer
___
The Quantum ESPRESSO community stands by the Ukrainian
people and expresses its concerns about the devastating
effects that the Russian military offensive has on their
country and on the free and peaceful scientific, cultural,
and economic cooperation amongst peoples
___
Quantum ESPRESSO is supported by MaX (www.max-centre.eu)
users mailing list users@lists.quantum-espresso.org
https://lists.quantum-espresso.org/mailman/listinfo/users

--
Paolo Giannozzi, Dip. Scienze Matematiche Informatiche e Fisiche,
Univ. Udine, via delle Scienze 208, 33100 Udine, Italy
Phone +39-0432-558216, fax +39-0432-558222

___
The Quantum ESPRESSO community stands by the Ukrainian
people and expresses its concerns about the devastating
effects that the Russian military offensive has on their
country and on the free and peaceful scientific, cultural,
and economic cooperation amongst peoples
___
Quantum ESPRESSO is supported by MaX (www.max-centre.eu)
users mailing list users@lists.quantum-espresso.org
https://lists.quantum-espresso.org/mailman/listinfo/users


--
Dr. rer. nat. Thomas Brumme
Theoretical chemistry
TU Dresden - KOE / 103
Bergstr. 66c
01069 Dresden

Tel:  +49 (0)351 463 39449

email: thomas.bru...@tu-dresden.de

___
The Quantu

Re: [QE-users] DFT+U - error when using the pseudopotential projectors

2022-12-10 Thread Thomas Brumme
Thanks for the explanation... I will check with the pseudo without semicore 
states and see if this solves the problem.

Thomas

Von: Iurii TIMROV 
Gesendet: Samstag, 10. Dezember 2022 11:33:44
An: Brumme, Thomas; Paolo Giannozzi; users@lists.quantum-espresso.org
Betreff: Re: [QE-users] DFT+U - error when using the pseudopotential projectors


Dear Thomas,


> especially the parenthesis - I can somehow understand the problem; I guess it 
> has to do with the possibility to properly project onto the "localized" 
> subspace, but do you know if there is a more rigorous explanation, maybe a 
> publication? Or Iurii? Maybe in one of your papers?


No, I am not aware of any publication where this is discussed. I do not think 
this is discussed anywhere, it is just a technicality (how things are 
implemented).


Regarding the parenthesis: In the older versions of QE (before the new 
DFT+Hubbard syntax introduced in v7.1), the code was considering in some 
routines the last "L channel" (where L is the orbital quantum number) as the 
Hubbard channel. But since v7.1 there is no this logic for DFT+U and DFT+U+V 
("atomic" and "ortho-atomic" projectors), because now the user has to specify 
in the input N and L (where N is the principal quantum number), so there is no 
ambiguity. I presume (but I am not sure) that this might be the reason why 
there is a problem for the "pseudo" type of Hubbard projectors (i.e. the same 
projectors as in VASP) when there are also semicore states (so you have the 
same L twice).


Greetings,

Iurii


--
Dr. Iurii TIMROV
Senior Research Scientist
Theory and Simulation of Materials (THEOS)
Swiss Federal Institute of Technology Lausanne (EPFL)
CH-1015 Lausanne, Switzerland
+41 21 69 34 881
http://people.epfl.ch/265334

From: Thomas Brumme 
Sent: Friday, December 9, 2022 11:18:47 PM
To: Paolo Giannozzi; users@lists.quantum-espresso.org
Cc: Iurii TIMROV
Subject: AW: [QE-users] DFT+U - error when using the pseudopotential projectors


Thanks Paolo for this important information - I haven't had the time yet to 
look at the code itself.


Concerning this small info:

one might run into troubles when using a PP with semicore states with same l as 
valence states
(also otherwhere for DFT+Hubbard)


especially the parenthesis - I can somehow understand the problem; I guess it 
has to do with the
possibility to properly project onto the "localized" subspace, but do you know 
if there is a more

rigorous explanation, maybe a publication? Or Iurii? Maybe in one of your 
papers?


Kind regards


Thomas

--
Dr. rer. nat. Thomas Brumme
Theoretical chemistry
TU Dresden - KOE / 103
Bergstr. 66c
01069 Dresden

Tel:  +49 (0)351 463 40844

email: thomas.bru...@tu-dresden.de



Von: Paolo Giannozzi 
Gesendet: Freitag, 9. Dezember 2022 17:06
An: users@lists.quantum-espresso.org
Cc: Brumme, Thomas; Iurii TIMROV
Betreff: Re: [QE-users] DFT+U - error when using the pseudopotential projectors

On 07/12/2022 16:26, Iurii TIMROV via users wrote:

> it might be that something has been broken over the years

it might be, given the complete absence of tests and examples for that
feature. It isn't, though: with a different pseudopotential
(Co.pbesol-n-kjpaw_psl.1.0.0.UPF) not containing semi-core states,
neither the current code nor the first version (5.0.3) implementing
DFT+U with pseudopotential projectors crash. The results seem to be the
same (convergence is nasty as usual so it is difficult to get exactly
the same numbers).

A comment in PW/src/init_q_aeps.f90 seems to confirm that the problem is
in the logic used to choose the projectors:

   ! NOTE: one might run into troubles when using a PP with semicore
   ! states with same l as valence states (also otherwhere for DFT+Hubbard

Paolo
--
Paolo Giannozzi, Dip. Scienze Matematiche Informatiche e Fisiche,
Univ. Udine, via delle Scienze 206, 33100 Udine Italy, +39-0432-558216
___
The Quantum ESPRESSO community stands by the Ukrainian
people and expresses its concerns about the devastating
effects that the Russian military offensive has on their
country and on the free and peaceful scientific, cultural,
and economic cooperation amongst peoples
___
Quantum ESPRESSO is supported by MaX (www.max-centre.eu)
users mailing list users@lists.quantum-espresso.org
https://lists.quantum-espresso.org/mailman/listinfo/users

Re: [QE-users] DFT+U - error when using the pseudopotential projectors

2022-12-09 Thread Thomas Brumme
Thanks Paolo for this important information - I haven't had the time yet to 
look at the code itself.


Concerning this small info:

one might run into troubles when using a PP with semicore states with same l as 
valence states
(also otherwhere for DFT+Hubbard)


especially the parenthesis - I can somehow understand the problem; I guess it 
has to do with the
possibility to properly project onto the "localized" subspace, but do you know 
if there is a more

rigorous explanation, maybe a publication? Or Iurii? Maybe in one of your 
papers?


Kind regards


Thomas

--
Dr. rer. nat. Thomas Brumme
Theoretical chemistry
TU Dresden - KOE / 103
Bergstr. 66c
01069 Dresden

Tel:  +49 (0)351 463 40844

email: thomas.bru...@tu-dresden.de



Von: Paolo Giannozzi 
Gesendet: Freitag, 9. Dezember 2022 17:06
An: users@lists.quantum-espresso.org
Cc: Brumme, Thomas; Iurii TIMROV
Betreff: Re: [QE-users] DFT+U - error when using the pseudopotential projectors

On 07/12/2022 16:26, Iurii TIMROV via users wrote:

> it might be that something has been broken over the years

it might be, given the complete absence of tests and examples for that
feature. It isn't, though: with a different pseudopotential
(Co.pbesol-n-kjpaw_psl.1.0.0.UPF) not containing semi-core states,
neither the current code nor the first version (5.0.3) implementing
DFT+U with pseudopotential projectors crash. The results seem to be the
same (convergence is nasty as usual so it is difficult to get exactly
the same numbers).

A comment in PW/src/init_q_aeps.f90 seems to confirm that the problem is
in the logic used to choose the projectors:

   ! NOTE: one might run into troubles when using a PP with semicore
   ! states with same l as valence states (also otherwhere for DFT+Hubbard

Paolo
--
Paolo Giannozzi, Dip. Scienze Matematiche Informatiche e Fisiche,
Univ. Udine, via delle Scienze 206, 33100 Udine Italy, +39-0432-558216
___
The Quantum ESPRESSO community stands by the Ukrainian
people and expresses its concerns about the devastating
effects that the Russian military offensive has on their
country and on the free and peaceful scientific, cultural,
and economic cooperation amongst peoples
___
Quantum ESPRESSO is supported by MaX (www.max-centre.eu)
users mailing list users@lists.quantum-espresso.org
https://lists.quantum-espresso.org/mailman/listinfo/users

[QE-users] DFT+U - error when using the pseudopotential projectors

2022-12-07 Thread Thomas Brumme

Dear all,

I wanted to try different projectors in my calculations in order to 
estimate their influence and also to
see if this could explain certain differences we see in comparison with 
VASP calculations. I will paste
the input file below. Now, in order to use the pseudos of the pslibary 
and to be sure that the
projectors are there, I recreated the corresponding pseudos, explicitly 
setting lsave_wfc = .true. as
mentioned in the INPUT_PW files. Yet, the calculation does not start but 
stops with this error:


 > QQ_AE matrix:
 0.985684 0.060950 0.00 0.00 0.00 0.00
 0.060950 0.086116 0.00 0.00 0.00 0.00
 0.00 0.00 0.970171-0.011672 0.00 0.00
 0.00 0.00-0.011672 0.513952 0.00 0.00
 0.00 0.00 0.00 0.00 0.829735 0.134914
 0.00 0.00 0.00 0.00 0.134914 0.343559
 nbH=  -1 , lH   2

 %%
 Error in routine init_q_aeps (1):
 could not set nbH
 %%

 stopping ...

I could not find anything related to this error in the mailing list. 
Does anyone ever used the pseudo
projectors? Because usually it is suggested to use ortho-atomic... Ah, I 
used the developer version

of QE from yesterday, so it should be pretty recent.

Any help concerning the proper usage of the pseudo projectors in the 
DFT+U approach is highly

appreciated :)

Kind regards

Thomas Brumme


The input:


    calculation='vc-relax'
    restart_mode='from_scratch',
    prefix='CoPS3'
    pseudo_dir = '/home/tbrumme/pseudos/pslibrary.1.0.0/pbesol/'
    outdir='/scratch/ws/0/tbrumme-WSe2_MoSe2/'
    verbosity='high'
    max_seconds=84600,
 /
 
    ibrav = 0,
    nat = 20,
    ntyp = 4,
    nspin = 2,
    ecutwfc = 80.0,
    ecutrho = 640.0,
    occupations = 'smearing',
    smearing = 'mv',
    degauss = 0.02,
    starting_magnetization(1) =  2.0,
    starting_magnetization(2) = -2.0,
    vdw_corr = 'grimme-d3'
    dftd3_version = 4
    Hubbard_occ(1,1) = 7.00
    Hubbard_occ(2,1) = 7.00
/

    conv_thr =  1.d-8
    mixing_beta = 0.2
    electron_maxstep = 500
/

/

/
ATOMIC_SPECIES
Co1  58.93  Co.pbesol-spn-kjpaw_psl.1.0.0.UPF
Co2  58.93  Co.pbesol-spn-kjpaw_psl.1.0.0.UPF
P    30.97   P.pbesol-n-kjpaw_psl.1.0.0.UPF
S    32.06   S.pbesol-n-kjpaw_psl.1.0.0.UPF

K_POINTS {automatic}
6 4 6 0 0 0

CELL_PARAMETERS (angstrom)
  5.8969998360 0.00 0.00
  0.00    10.2159996033 0.00
 -1.9672647655 0.00 6.3670060311
ATOMIC_POSITIONS (angstrom)
Co1  0.0 3.394776821 0.0
Co2  0.0 6.821222782 0.0
Co2  2.948499918 8.502776146 0.0
Co1  2.948499918 1.713223100 0.0
P    0.002085491 0.0 1.088757992
P    3.927649736 0.0 5.278247833
P    2.950585365 5.107999802 1.088757992
P    0.979149818 5.107999802 5.278247833
S    3.915214777 0.0 1.566283464
S    0.014520367 0.0 4.800722599
S    0.966715097 5.107999802 1.566283464
S    2.963020086 5.107999802 4.800722599
S    0.992270052 1.715266347 1.559916496
S    2.937465191 8.500733376 4.807089329
S    2.937465191 1.715266347 4.807089329
S    0.992270052 8.500733376 1.559916496
S    3.940770149 6.823265553 1.559916496
S   -0.011034934 3.392733574 4.807089329
S   -0.011034934 6.823265553 4.807089329
S    3.940770149 3.392733574 1.559916496

HUBBARD pseudo
U Co1-3d 5.0
U Co2-3d 5.0


--
Dr. rer. nat. Thomas Brumme
Theoretical chemistry
TU Dresden - KOE / 103
Bergstr. 66c
01069 Dresden

Tel:  +49 (0)351 463 40844

email: thomas.bru...@tu-dresden.de

___
The Quantum ESPRESSO community stands by the Ukrainian
people and expresses its concerns about the devastating
effects that the Russian military offensive has on their
country and on the free and peaceful scientific, cultural,
and economic cooperation amongst peoples
___
Quantum ESPRESSO is supported by MaX (www.max-centre.eu)
users mailing list users@lists.quantum-espresso.org
https://lists.quantum-espresso.org/mailman/listinfo/users

Re: [QE-users] Problem in vc-relax calculation with Hubbard U in magnetic system

2022-11-27 Thread Thomas Brumme
Hi Iurii,


I'm working with Vivek on the structure - one question I would have is why is 
QE different from VASP? (and which solution is correct?)

In VASP the relaxation is not restricted and since both codes use more or less 
the same theory, the difference might come from some
details in the implementation of +U or from some things related to the pseudo. 
Do you have any feeling/idea if changing to a different
pseudo or something different concerning +U might influence the relaxation?


Kind regards


Thomas

--
Dr. rer. nat. Thomas Brumme
Theoretical chemistry
TU Dresden - KOE / 103
Bergstr. 66c
01069 Dresden

Tel:  +49 (0)351 463 40844

email: thomas.bru...@tu-dresden.de



Von: users  im Auftrag von Iurii 
TIMROV via users 
Gesendet: Sonntag, 27. November 2022 12:17
An: Vivekanand Shukla; Quantum ESPRESSO users Forum
Betreff: Re: [QE-users] Problem in vc-relax calculation with Hubbard U in 
magnetic system


> The output structure does not hold the monoclinic symmetry; Hubbard U should 
> help it achieve that. However, I do not get it despite using any Hubbard U.


Maybe you can try to impose the monoclinic symmetry (ibrav = 12 or -12), apply 
+U, and use cell_dofree = "ibrav" during the vc-relax calculation.


HTH


Iurii


--
Dr. Iurii TIMROV
Senior Research Scientist
Theory and Simulation of Materials (THEOS)
Swiss Federal Institute of Technology Lausanne (EPFL)
CH-1015 Lausanne, Switzerland
+41 21 69 34 881
http://people.epfl.ch/265334

From: Vivekanand Shukla 
Sent: Friday, November 25, 2022 5:30:59 PM
To: Iurii TIMROV; Quantum ESPRESSO users Forum
Subject: Re: [QE-users] Problem in vc-relax calculation with Hubbard U in 
magnetic system


Dear Iurii,


Thank you for the reference links. I followed them and ran some tests. It works 
with U prediction. However, with CoPS3, I wonder whether even Hubbard's 
inclusion is working.


So there is some change. Is it not what you expect?


Problem: Without Hubbard U inclusion lattice parameter comes {a ≠ sqrt(3)*b}, 
but with the inclusion of U, it should be corrected to more like {a = 
sqrt(3)*b}. According to the literature.


The output structure does not hold the monoclinic symmetry; Hubbard U should 
help it achieve that. However, I do not get it despite using any Hubbard U.


I also ran calculations with higher convergence criteria but ended up with 
similar lattice parameters. I am now running a calculation with ibrav= -12, but 
I am wondering if it is the right approach in this system.


Regards

Vivek

TU Dresden



From: users  on behalf of Iurii 
TIMROV via users 
Sent: Friday, November 25, 2022 5:11:30 PM
To: users@lists.quantum-espresso.org
Subject: Re: [QE-users] Problem in vc-relax calculation with Hubbard U in 
magnetic system


Dear Vivek,


> I want to predict the self-consistent Hubbard parameter using HP code.


Please have a look here:

https://www.materialscloud.org/learn/sections/tctKdH/advanced-quantum-espresso-tutorial-2022


> ...but the structure remains the same.


>From your input and output files I see this:


OLD

CELL_PARAMETERS (angstrom)
  5.8969998360 0.00 0.00
  0.0010.2159996033 0.00
 -1.9672647655 0.00 6.3670060311


NEW

CELL_PARAMETERS (angstrom)
   5.771620672   0.0   0.036554788
   0.0  10.142807293   0.0
  -1.885622677   0.0   6.187226171


So there is some change. It is not what you expect?


> conv_thr =  1.d-6


Try 1.d-10 or even smaller


> forc_conv_thr  = 1.d-3


Try forc_conv_thr  = 1.d-4 or 1.d-5

and

etot_conv_thr = 1.d-5 or 1.d-6


Have a look also here:

https://www.materialscloud.org/work/tools/qeinputgenerator


HTH


Iurii


--
Dr. Iurii TIMROV
Senior Research Scientist
Theory and Simulation of Materials (THEOS)
Swiss Federal Institute of Technology Lausanne (EPFL)
CH-1015 Lausanne, Switzerland
+41 21 69 34 881
http://people.epfl.ch/265334

From: users  on behalf of Vivekanand 
Shukla 
Sent: Friday, November 25, 2022 4:34:20 PM
To: users@lists.quantum-espresso.org
Subject: [QE-users] Problem in vc-relax calculation with Hubbard U in magnetic 
system


Dear all,

I am trying to run a vc-relax calculation for CoPS3 in bulk with the Hubbard U 
parameter. CoPS3 results in off-lattice parameters without Hubbard U. I checked 
it with VASP, and it seems realistic. However, in QE, the Hubbard U parameter 
has no effect on the lattice parameter. It remains off with or without U. I can 
see the Hubbard effect in forces in the output file, but it does not reflect on 
the lattice parameter in QE output.

I want to predict the self-consistent Hubbard parameter using HP code. It does 
not make sense if I do not get the inclusion of the U parameter right in the 
first place. I used QE versions 6.4.1, 6.7, and 7.0 and 

Re: [QE-users] Spin projections not working OR (more probable) really dumb misunderstanding

2022-07-07 Thread Thomas Brumme
OK, I retract the question... It is really dumb. One just has to think a 
bit about what "expectation value" really means.


Anyways, cheerio

Thomas

On 7/7/22 15:03, Thomas Brumme wrote:

Dear all,

I'm calculating the full-relativistic band structure of some WSe2 
nanotubes which works fine and also the band structure agrees with 
calculations done with FHI-aims (which I have to use for larger systems).
In order to understand some interesting feature in the band splitting 
I'm also calculating the expectation values of the spin operators 
using bands.x and setting lsigma(1..3) to .true.


Yet, for some of the bands all expectation values are zero. I don't 
understand this. OK, I know that only the total angular momentum is 
conserved and spin is not a good quantum number anymore but does this 
also mean that the expectation value for the "spin vector" can be 
zero? I had a similar problem in the past for a monolayer WSe2 and 
there the spin was also not really 1/2 anymore which might actually be 
a DFT-specific problem as far as I understood it after discussing with 
other colleagues. But zero?


Kind regards

Thomas Brumme


--
Dr. rer. nat. Thomas Brumme
Theoretical chemistry
TU Dresden - KOE / 103
Bergstr. 66c
01069 Dresden

Tel:  +49 (0)351 463 40844

email: thomas.bru...@tu-dresden.de

___
The Quantum ESPRESSO community stands by the Ukrainian
people and expresses its concerns about the devastating
effects that the Russian military offensive has on their
country and on the free and peaceful scientific, cultural,
and economic cooperation amongst peoples
___
Quantum ESPRESSO is supported by MaX (www.max-centre.eu)
users mailing list users@lists.quantum-espresso.org
https://lists.quantum-espresso.org/mailman/listinfo/users


[QE-users] Spin projections not working OR (more probable) really dumb misunderstanding

2022-07-07 Thread Thomas Brumme

Dear all,

I'm calculating the full-relativistic band structure of some WSe2 
nanotubes which works fine and also the band structure agrees with 
calculations done with FHI-aims (which I have to use for larger systems).
In order to understand some interesting feature in the band splitting 
I'm also calculating the expectation values of the spin operators using 
bands.x and setting lsigma(1..3) to .true.


Yet, for some of the bands all expectation values are zero. I don't 
understand this. OK, I know that only the total angular momentum is 
conserved and spin is not a good quantum number anymore but does this 
also mean that the expectation value for the "spin vector" can be zero? 
I had a similar problem in the past for a monolayer WSe2 and there the 
spin was also not really 1/2 anymore which might actually be a 
DFT-specific problem as far as I understood it after discussing with 
other colleagues. But zero?


Kind regards

Thomas Brumme

--
Dr. rer. nat. Thomas Brumme
Theoretical chemistry
TU Dresden - KOE / 103
Bergstr. 66c
01069 Dresden

Tel:  +49 (0)351 463 40844

email: thomas.bru...@tu-dresden.de

___
The Quantum ESPRESSO community stands by the Ukrainian
people and expresses its concerns about the devastating
effects that the Russian military offensive has on their
country and on the free and peaceful scientific, cultural,
and economic cooperation amongst peoples
___
Quantum ESPRESSO is supported by MaX (www.max-centre.eu)
users mailing list users@lists.quantum-espresso.org
https://lists.quantum-espresso.org/mailman/listinfo/users


Re: [QE-users] How to compute spin projections?

2022-03-30 Thread Thomas Brumme
Dear Luiz,


the correct tool you search is bands.x - if you look at the description of it, 
you will find the key word

lsigma(i), i=1,3
which does the job. Usually, bands.x is used for paths along high symmetry 
lines in the BZ but you
can also provide a full MP grid if you want to plot the spin texture in the 
whole BZ. I actually did this
for a manuscript which is in the still-to-be-finalized stage since ages...

Kind regards

Thomas


--
Dr. rer. nat. Thomas Brumme
Theoretical chemistry
TU Dresden - KOE / 103
Bergstr. 66c
01069 Dresden

Tel:  +49 (0)351 463 40844

email: thomas.bru...@tu-dresden.de



Von: users  im Auftrag von Luiz 
Gustavo Davanse da Silveira via users 
Gesendet: Mittwoch, 30. März 2022 18:07
An: Lorenzo Paulatto
Cc: Quantum ESPRESSO users Forum
Betreff: Re: [QE-users] How to compute spin projections?

Dear Lorenzo Paulatto,

Thanks for your reply. Indeed it does. However I would like to have spin
projections in reciprocal space to plot spin-textures, for example.

Best regards,
Luiz G. D. Silveira
Associate Professor - UFPR - Brazil

Em Qua, Março 30, 2022 3:56 am, Lorenzo Paulatto escreveu:
> I think if you do a non-collinear spin calculation, QE prints the spin
> around each atom on screen., i.e. :
> =
> =
> atom number 1 relative position : 0. 0. 0. charge : 9.813443
> magnetization : -0.86 0.00 0.00 magnetization/charge:
> -0.09 0.00 0.00
> polar coord.: r, theta, phi [deg] : 0.86 90.00 180.00
>
> =
> =
>
>
> --
> Lorenzo Paulatto - Paris
> On Mar 30 2022, at 4:09 am, Luiz Gustavo Davanse da Silveira via users
>  wrote:
>
>> Dear QE users and developers,
>>
>>
>> For non-linear calculations VASP print out on the PROCAR file three
>> othogonal spin projections at each k point of each band. Is there an way
>>  to obtain the same information from calculations done in QE?
>>
>> Best regards,
>> Luiz G. D Silveira
>> Associate Professor - UFPR - Brazil
>>
>>
>> ___
>> The Quantum ESPRESSO community stands by the Ukrainian
>> people and expresses its concerns about the devastating effects that the
>> Russian military offensive has on their
>> country and on the free and peaceful scientific, cultural, and economic
>> cooperation amongst peoples
>> ___
>> Quantum ESPRESSO is supported by MaX 
>> (www.max-centre.eu<http://www.max-centre.eu>)
>> users mailing list users@lists.quantum-espresso.org
>> https://lists.quantum-espresso.org/mailman/listinfo/users
>>
>>
>
>


___
The Quantum ESPRESSO community stands by the Ukrainian
people and expresses its concerns about the devastating
effects that the Russian military offensive has on their
country and on the free and peaceful scientific, cultural,
and economic cooperation amongst peoples
___
Quantum ESPRESSO is supported by MaX 
(www.max-centre.eu<http://www.max-centre.eu>)
users mailing list users@lists.quantum-espresso.org
https://lists.quantum-espresso.org/mailman/listinfo/users
___
The Quantum ESPRESSO community stands by the Ukrainian
people and expresses its concerns about the devastating
effects that the Russian military offensive has on their
country and on the free and peaceful scientific, cultural,
and economic cooperation amongst peoples
___
Quantum ESPRESSO is supported by MaX (www.max-centre.eu)
users mailing list users@lists.quantum-espresso.org
https://lists.quantum-espresso.org/mailman/listinfo/users

Re: [QE-users] Fe (110) slab isn't converging

2022-02-10 Thread Thomas Brumme

Dear Omer,

what is not converging? The scf or the relaxation?
Looking at your input I can see one main problem: the cutoff. Please 
have a look at this very useful page:


https://www.materialscloud.org/discover/sssp/plot/efficiency/Fe

The pseudopotential you're using needs much more than 30 Ry cutoff. I 
guess that you will need to increase the cutoff to at least 50 Ry for 
the wave functions and 400 Ry for the density.
Furthermore, your smearing is quite low. If you have problems with 
convergence, increasing this to ~100meV can help.


Kind regards

Thomas

On 2/10/22 11:41, Omer Mutasim via users wrote:

Dear all
I tried to relax Fe (110) slab , but it doesn't converge. It takes 
very long time even though it is small slab with only 45 atoms.

 I tried f-d, m-p smearing but to no avail.
Can you help me please ? to know what is wrong with my input file 
attached below:



    calculation   = "relax"
prefix = 'x'
    outdir = '/scratch'
    pseudo_dir = '/xx/'
!restart_mode = 'restart'
    forc_conv_thr =  1.0e-03
etot_conv_thr=1e-04
    nstep         = 

/


ibrav  =0
    ecutrho   = 150
    ecutwfc   = 30
    nat       = 45
    ntyp      = 1
occupations ='smearing' , smearing='f-d' , degauss=0.005
vdw_corr    = 'DFT-D3'
nspin = 2
starting_magnetization(1) = 0.1
/

    conv_thr         =1e-06
    electron_maxstep = 
mixing_mode='local-TF'
    mixing_beta      =  0.3
/


/

K_POINTS {automatic}
221000

ATOMIC_SPECIES
Fe  55.845   Fe.pbe-spn-kjpaw_psl.0.2.1.UPF
CELL_PARAMETERS {angstrom}
        7.4474000931         0.00  0.00
        2.4824640508         7.0214770799  0.00
        0.00         0.00 25.1350002289
ATOMIC_POSITIONS (angstrom)
Fe            5.5818789877        2.5026591675  10.5421792561    0   0   0
Fe            4.1125866520        4.8151882619  10.6772393880
Fe            2.1958530690        7.2414707420  10.3206255931
Fe            1.1033564060        1.9504258780 0.00    0   0   0
Fe            1.1240846033        1.9239546627 4.4015746387
Fe            0.5929191720        2.2727761723 9.2300878464    0   0   0
Fe            1.9418633362        0.7673280599 2.1503683749
Fe            1.8505911190        0.7816285240 6.8697562640
Fe            3.5857982640        1.9504258780 0.00    0   0   0
Fe            3.6010156782        2.0356057803 4.4040536951
Fe            3.3892107507        2.4333408596 9.2315052153    0   0   0
Fe            4.4107991582        0.7671459373 2.2844419078
Fe            4.3403612910        0.9089118831 6.8687340519
Fe            6.0682401770        1.9504258780 0.00    0   0   0
Fe            6.0921186925        1.9492701251 4.5081309218
Fe            6.8957462620        0.7801563310 2.0268864960    0   0   0
Fe            6.8261914569        0.9046029160 6.7661838994
Fe            1.8724868770        4.4014805858  -0.2143812725
Fe            1.8485507821        4.2994702275 4.3824953904    0   0   0
Fe            1.8240284641        4.7344494204 9.2345798280
Fe            2.7582676160        3.1206253240 2.0268864960    0   0   0
Fe            2.6683169740        3.1739888190 6.8797363658
Fe            4.3474449433        4.3631009861  -0.1911888976
Fe            4.3246850209        4.4213381253 4.3668090656    0   0   0
Fe            5.2753804683        3.1017230426 2.1588431075
Fe            5.1435681580        3.2467521647 6.6302721725    0   0   0
Fe            6.8866553226        4.2727158920  -0.2151129276
Fe            6.8148177194        4.3601995734 4.3973105493
Fe            6.4979897094        4.6088015071 9.2447583930    0   0   0
Fe            7.7586179448        3.1003648378 2.1698200674
Fe            7.6317422302        3.2430726605 6.8687193850    0   0   0
Fe            2.7205563083        6.7460510219  -0.2000658247
Fe            2.7608454757        6.6172987829 4.5215696688
Fe            3.5857721560        5.4611644180 2.0268864960    0   0   0
Fe            3.6115378719        5.4595736549 6.6122865627
Fe            5.2407577750        6.6313638640 0.00    0   0   0
Fe            5.2548206739        6.7098271636 4.3861096452
Fe            4.8691076063        6.9050837980 9.2329583714
Fe            6.0682884220        5.4611644180 2.0268864960    0   0   0
Fe            6.1028694761        5.5498338413 6.8798135598
Fe            7.7231996880        6.6313638640 0.00    0   0   0
Fe            7.7426438544        6.6251084192 4.3864901090
Fe            7.3977446239        6.9684054207 9.2402605725
Fe            8.5507305580        5.4611644180 2.0268864960    0   0   0
Fe            8.6014581434        5.5444835700 6.8763383415

___
Quantum ESPRESSO is supported by MaX (www.max-centre.eu)
users mailing listus...@lists.quantum-espresso.org
https://lists.quantum-espresso.org/mailman/listinfo/users


--
Dr. rer. nat. Thomas Brumme
Theoretical chemistry
TU Dresden - KOE / 103
Bergstr. 66c
01069 Dr

Re: [QE-users] Assunto: Re: Assunto: Re: Difference in Fermi energy by increasing k-points

2022-01-04 Thread Thomas Brumme
Dear José,


what is the Fermi energy? Well, at 0K it tells you that all states below are 
occupied and all states above are unoccupied.
Yet, if you have a band gap, as in your case, the Fermi energy could be 
anywhere in the band gap and this wouldn't
change a thing - standard DFT is for 0K and the broadening used in the code 
(which is only needed for metals) is a
numerical trick to help with the convergence. If people plot the band structure 
for a semiconductor w.r.t. the Fermi

energy, the result can depend on all possible things in the calculation and 
this is what you see - for one specific

k-point mesh you probably sample the valence band better and thus the Fermi 
energy is closer to the valence-band
maximum. Again, for a semiconductor, the Fermi energy can be anywhere in the 
band gap and this does not change
the result. In principle you don't need broadening for a semiconductor and then 
the code will print the HOMO and
LUMO energies. You should plot the band structure to understand where is the 
position of the valence-band

maximum and then you will understand why 10x10x10 is different...


Regards


Thomas


--
Dr. rer. nat. Thomas Brumme
Theoretical chemistry
TU Dresden - KOE / 103
Bergstr. 66c
01069 Dresden

Tel:  +49 (0)351 463 40844

email: thomas.bru...@tu-dresden.de



Von: users  im Auftrag von José 
Xavier via users 
Gesendet: Dienstag, 4. Januar 2022 20:47
An: Quantum ESPRESSO users Forum; Paolo Giannozzi; Kazume NISHIDATE
Betreff: Re: [QE-users] Assunto: Re: Assunto: Re: Difference in Fermi energy by 
increasing k-points

Dear Giannozzi,

Yes, the bandgap and peaks are the same with a shift in the axis x.
However, all the tutorials I've watched and read showed me that, by using the 
Efermi to set the zero in the graph, the last peak of the valence band should 
end in the zero (as in my reference). But, in my case, it only occurs when I 
use 10x10x10 in the NSCF calculation and all other combinations of k-points 
gave me the second figure.

It is strange, for me, that only one value of k-points showed me the behavior 
that I was expecting, and that is the reason for my question. What is happening 
in my calculations to make the k-point 10x10x10 the only one that shows me a 
value of EFermi that corresponds to the "real" zero of the graph?

Best wishes
José Xavier



Em terça-feira, 4 de janeiro de 2022 06:45:32 BRT, Paolo Giannozzi 
 escreveu:





The two figures are exactly the same, only the origin of the x axis is shifted.

Paolo

On Tue, Jan 4, 2022 at 10:29 AM José Xavier via users 
 wrote:
> Dear Nishidate,
>
> Thank you for your answer.
>
> I don't know if I understood... Could you explain me better what should I do, 
> please?
>
> I'm sending a figure of my DOS plotted using the HOMO and the EFermi as the 
> zero, respectively, to make my doubt clearer.
>
> I learned that I should use a "large" number of k-points to converge the 
> system. However, I only found the first figure when I used the 10x10x10 
> k-points in the NSCF calculation, and 2x2x2, 4x4x4 or 6x6x6 in the SCF. When 
> I increased or decreased the k-points in the NSCF, it showed me the second 
> figure (that is different from the reference one).
>
> I tried to use the smearing occupations. In this test, the EFermi in SCF and 
> NSCF calculations were the same (2.74 eV), except when I used 10x10x10 
> k-points in the NSCF. In this case, the EFermi was 2.74 in SCF and 1.04 eV in 
> the NSCF (it is the same value that I found previously as the HOMO in SCF 
> calculation).
>
> My main question is: Why is this 10x10x10 a "magic number"?
>
> Best wishes.
> I wish you a happy new year.
>
> José Xavier
>
>
>>
>>
>> Em qua., 29 29e dez. 29e 2021 às 18:22, Kazume NISHIDATE
>>  escreveu:
>
>
>>
>>
>>
>>
>
>
>
> Dear Xavier
>
>> When I used the k-points 4x4x4 (SCF) and 8x8x8 (NSCF), the HOMO and
>> EFermi were different, and using EFermi to subtract the values in the
>
> If the k-point mesh you selected covers the typical energy peaks of
> the valence band maximum (VBM or the HOMO in your description) depends
> on the shape of Brillouin zone (BZ) of your system.
>
> You should try to plot the band structure and assess the VBM and find
> out its k-point location. If the k-point mesh you selected does not
> cover the k-point, then the VBM peak will not contribute to your DOS
>
> plot.
>
>
>
>
>
> 敬具 西館
>
> best regards
>
>
>
> nisid...@iwate-u.ac.jp
>
> kazume.nishid...@gmail.com
>
> ___
> Quantum ESPRESSO is supported by MaX 
> (www.max-centre.eu<http://www.max-centre.eu>)
> users mailing list users@lists.quantum-espresso.org
> https://lists.quantum-espre

Re: [QE-users] orbital character order with spin-orbit coupling

2021-11-10 Thread Thomas Brumme

Dear Hari Paudyal,

in case of SOC, the orbital quantum number is not a good quantum number 
anymore and you
have to use the total angular momentum. Thus, strictly speaking there is 
no pz-state anymore.
Yet, if you work through the details of spin-orbit coupling and the 
details given in this

publication which - I think - describes the implementation in QE:

https://journals.aps.org/prb/abstract/10.1103/PhysRevB.71.115106

you will understand that the pz mixes with other states and that the SOC 
states with a large
contribution of pz-character are those with m_j = +- 1/2 for both j = 
5/2 and j = 3/2


Cheerio

Thomas Brumme


P.S.: Signing your email with your affiliation is highly recommended.


On 11/9/21 10:01 PM, Hari Paudyal via users wrote:

Hi experts,

Can anyone help me to identify pz, px, py characters in the spin-orbit 
coupling (SOC) band projection?


It is well explained without SOC, the order will be pz, px, py as 
follows (in my calculation for Se atom)

..
     state #  12: atom   2 (Se ), wfc  2 (l=1 m= 1)
     state #  13: atom   2 (Se ), wfc  2 (l=1 m= 2)
     state #  14: atom   2 (Se ), wfc  2 (l=1 m= 3)
.

However, with SOC, it shows as follows based on j = l+s, and j = l-s, 
where s = 0.5


     state #  23: atom   2 (Se ), wfc  2 (l=1 j=1.5 m_j=-1.5)
     state #  24: atom   2 (Se ), wfc  2 (l=1 j=1.5 m_j=-0.5)
     state #  25: atom   2 (Se ), wfc  2 (l=1 j=1.5 m_j= 0.5)
     state #  26: atom   2 (Se ), wfc  2 (l=1 j=1.5 m_j= 1.5)
     state #  27: atom   2 (Se ), wfc  3 (l=1 j=0.5 m_j=-0.5)
     state #  28: atom   2 (Se ), wfc  3 (l=1 j=0.5 m_j= 0.5)

for l = 1 (p orbital), and s =  0.5 j = 1.5, and mj = -1.5, -0.5, 0.5, 1.5
for l = 1 (p orbital), and s = -0.5 j = 0.5, and mj = -0.5, 0.5
This makes sense, but which one are pz, px, and py?

Sincerely,
Hari Paudyal



___
Quantum ESPRESSO is supported by MaX (www.max-centre.eu)
users mailing list users@lists.quantum-espresso.org
https://lists.quantum-espresso.org/mailman/listinfo/users


--
Dr. rer. nat. Thomas Brumme
Theoretical chemistry
TU Dresden - BAR / II49
Helmholtzstr. 18
01069 Dresden

Tel:  +49 (0)351 463 40844

email: thomas.bru...@tu-dresden.de

___
Quantum ESPRESSO is supported by MaX (www.max-centre.eu)
users mailing list users@lists.quantum-espresso.org
https://lists.quantum-espresso.org/mailman/listinfo/users

Re: [QE-users] Interpolation for 2D phonons results in one imaginary branch near Gamma

2021-07-14 Thread Thomas Brumme
Thanks Vahid! This is actually a nice hint. I didn't relax the lattice with QE 
since I got the relaxed structure from someone else (collaboration) and minor 
differences between codes had - in my experience (3D solids) - never such a big 
influence on the phonons modes. But I will have a look at the stresses...


Anyways, meanwhile the phonon dispersion was also calculated using phonopy, 
i.e., the supercell method, and there is no imaginary mode. Dunno if the code 
interpolates differently or enforces the acoustic sum rule in a different way, 
but it is weird that the collaborator uses phonopy with QE and gets positive 
modes and I don't...


Thanks again for the hint!


Thomas


--
Dr. rer. nat. Thomas Brumme
Theoretical chemistry
TU Dresden - BAR / II49
Helmholtzstr. 18
01069 Dresden

Tel:  +49 (0)351 463 40844

email: thomas.bru...@tu-dresden.de



Von: users  im Auftrag von Vahid 
Askarpour 
Gesendet: Mittwoch, 14. Juli 2021 17:29
An: Quantum ESPRESSO users Forum
Betreff: Re: [QE-users] Interpolation for 2D phonons results in one imaginary 
branch near Gamma

Hi Thomas,

This may not be directly related to your question and you may have already 
tried it but I found that for 2D materials, if in the vc-relax step, I ensure 
that the in-plane stresses are almost or exactly zero, I can make the flexural 
mode energies entirely positive along the various BZ segments.

Cheers,
Vahid

Vahid Askarpour
Department of physics and atmospheric science
Dalhousie University
Halifax, NS
CANADA

> On Jul 14, 2021, at 12:00 PM, Thomas Brumme  wrote:
>
> CAUTION: The Sender of this email is not from within Dalhousie.
>
> Did anyone try recently the 2D cutoff technique for phonons? Is there
> maybe an error in my input?
>
> Regards
>
> Thomas
>
> On 7/7/21 11:37 AM, Stefano Baroni wrote:
>> Oh, certo: scusa, dimenticavo. S.
>>
>> --
>> Stefano Baroni, Trieste -- swift message written and sent on the go
>>
>>> Il giorno 7 lug 2021, alle ore 10:42, Thomas Brumme 
>>>  ha scritto:
>>>
>>> Dear all,
>>>
>>> I think I don't see the wood for the trees and hope that someone can help.
>>>
>>> I want to calculate the phonons for a 2D material using the 2D cutoff 
>>> technique. The scf calculations for the well-relaxed system work without a 
>>> problem and I also get no error calculating the phonons on a 5x5x1 q-point 
>>> grid. The relevant portions of the scf and phonon input are attached below. 
>>> All frequencies are positive at every single q point, except the 3 acoustic 
>>> branches at the Gamma point which are "only" -10 cm^-1 and -15 cm^-1. 
>>> However, the interpolation, in order to get the dispersion, results in one 
>>> branch which is negative for quite some portion near Gamma - for the first 
>>> half along G-K I have up to -130 cm^-1 and along G-M up to -60 cm^-1. I 
>>> guess that this is the out-of-plane acoustic branch and now I wonder if 
>>> this is real (i.e., the system is really unstable) or if I make some 
>>> obvious mistake. I will also check by explicitly calculating few q points 
>>> in the relevant region but before I wanted to ask here.
>>>
>>> Any help would be appreciated!
>>>
>>> Regards
>>>
>>> Thomas Brumme
>>>
>>> 
>>> scf:
>>>
>>> 
>>> calculation   = 'scf'
>>> prefix= 'pasc12',
>>> pseudo_dir= '/home/tbrumme/pseudos/pslibrary.1.0.0/rel-pbe/',
>>> outdir= '/scratch/ws/0/tbrumme-pasc/',
>>> verbosity = 'high',
>>> wf_collect= .true.,
>>> /
>>>
>>> 
>>>ibrav = 0,
>>>  nat = 14,
>>> ntyp = 3,
>>>  ecutwfc = 70.0,
>>>  ecutrho = 560,
>>> noncolin = .true.,
>>> lspinorb = .true.,
>>> vdw_corr = 'DFT-D',
>>> nbnd = 96,
>>>  assume_isolated = '2D',
>>> /
>>>
>>> 
>>>  conv_thr = 1.D-14,
>>> /
>>>
>>> ATOMIC_SPECIES
>>> C 12.0 'C.rel-pbe-n-kjpaw_psl.1.0.0.UPF'
>>> P 30.973761 'P.rel-pbe-n-kjpaw_psl.1.0.0.UPF'
>>>As 74.921595 'As.rel-pbe-dn-kjpaw_psl.1.0.0.UPF'
>>>
>>> ATOMIC_POSITIONS crystal
>>> ...
>>>
>>> CELL_PARAMETERS angstrom
>>>  6.69668  0.0   0.0
>>> -3.348340 5.799495  0.0
>>>  0.0  0.0  15.0
>>>
>>> K_POINTS automatic
>>>   25 25 1 0 0

Re: [QE-users] Interpolation for 2D phonons results in one imaginary branch near Gamma

2021-07-14 Thread Thomas Brumme
Did anyone try recently the 2D cutoff technique for phonons? Is there 
maybe an error in my input?


Regards

Thomas

On 7/7/21 11:37 AM, Stefano Baroni wrote:

Oh, certo: scusa, dimenticavo. S.

--
Stefano Baroni, Trieste -- swift message written and sent on the go


Il giorno 7 lug 2021, alle ore 10:42, Thomas Brumme  
ha scritto:

Dear all,

I think I don't see the wood for the trees and hope that someone can help.

I want to calculate the phonons for a 2D material using the 2D cutoff technique. The scf 
calculations for the well-relaxed system work without a problem and I also get no error 
calculating the phonons on a 5x5x1 q-point grid. The relevant portions of the scf and 
phonon input are attached below. All frequencies are positive at every single q point, 
except the 3 acoustic branches at the Gamma point which are "only" -10 cm^-1 
and -15 cm^-1. However, the interpolation, in order to get the dispersion, results in one 
branch which is negative for quite some portion near Gamma - for the first half along G-K 
I have up to -130 cm^-1 and along G-M up to -60 cm^-1. I guess that this is the 
out-of-plane acoustic branch and now I wonder if this is real (i.e., the system is really 
unstable) or if I make some obvious mistake. I will also check by explicitly calculating 
few q points in the relevant region but before I wanted to ask here.

Any help would be appreciated!

Regards

Thomas Brumme


scf:


 calculation   = 'scf'
 prefix= 'pasc12',
 pseudo_dir= '/home/tbrumme/pseudos/pslibrary.1.0.0/rel-pbe/',
 outdir= '/scratch/ws/0/tbrumme-pasc/',
 verbosity = 'high',
 wf_collect= .true.,
/


ibrav = 0,
  nat = 14,
 ntyp = 3,
  ecutwfc = 70.0,
  ecutrho = 560,
 noncolin = .true.,
 lspinorb = .true.,
 vdw_corr = 'DFT-D',
 nbnd = 96,
  assume_isolated = '2D',
/


  conv_thr = 1.D-14,
/

ATOMIC_SPECIES
 C 12.0 'C.rel-pbe-n-kjpaw_psl.1.0.0.UPF'
 P 30.973761 'P.rel-pbe-n-kjpaw_psl.1.0.0.UPF'
As 74.921595 'As.rel-pbe-dn-kjpaw_psl.1.0.0.UPF'

ATOMIC_POSITIONS crystal
...

CELL_PARAMETERS angstrom
  6.69668  0.0   0.0
-3.348340 5.799495  0.0
  0.0  0.0  15.0

K_POINTS automatic
   25 25 1 0 0 0


phonons:

Phonon dispersions for pasc12
  
   tr2_ph=1.0d-18,
   prefix='pasc12',
   ldisp=.true.,
   nq1=5, nq2=5, nq3=1,
   outdir='/scratch/ws/0/tbrumme-pasc/',
   fildyn='pasc12.dyn.xml',
   max_seconds = 83700,
   verbosity='high',
   recover=.true.
   alpha_mix(1)=0.4,
  /


q2r and matdyn:


   fildyn='pasc12.dyn.xml', flfrc='pasc12.551.fc', loto_2d=.false., 
zasr='simple',
/


 flfrc='pasc12.551.fc.xml', flfrq='pasc12.freq', q_in_band_form=.true., 
loto_2d=.false., asr='simple',
 fldyn='pasc12.dyn.xml'
  /
  4
   0. 0. 0.0   60
   0. 0.57735027 0.0   30
   0. 0.57735027 0.0   45
   0. 0. 0.0   1

--
Dr. rer. nat. Thomas Brumme
Theoretical chemistry
TU Dresden - BAR / II49
Helmholtzstr. 18
01069 Dresden

Tel:  +49 (0)351 463 40844

email: thomas.bru...@tu-dresden.de

___
Quantum ESPRESSO is supported by MaX (www.max-centre.eu)
users mailing list users@lists.quantum-espresso.org
https://lists.quantum-espresso.org/mailman/listinfo/users

___
Quantum ESPRESSO is supported by MaX (www.max-centre.eu)
users mailing list users@lists.quantum-espresso.org
https://lists.quantum-espresso.org/mailman/listinfo/users


--
Dr. rer. nat. Thomas Brumme
Theoretical chemistry
TU Dresden - BAR / II49
Helmholtzstr. 18
01069 Dresden

Tel:  +49 (0)351 463 40844

email: thomas.bru...@tu-dresden.de

___
Quantum ESPRESSO is supported by MaX (www.max-centre.eu)
users mailing list users@lists.quantum-espresso.org
https://lists.quantum-espresso.org/mailman/listinfo/users

Re: [QE-users] order of the quantum number mj in the pDOS output

2021-07-08 Thread Thomas Brumme

Dear Elie,


looking at the beginning of one of my outputs of projwfc.x I can see 
that the states are always ordered


ascending, i.e., beginning the lowest m_j. Thus, I guess that the order 
is -3/2, -1/2, 1/2, 3/2


Really in the beginning it is written "Atomic states used for projection"...


Regards


Thomas


On 7/8/21 12:57 AM, Elio Physics wrote:

Dear all,

I am trying to extract the partial density of states of a system in 
the presence of the spin orbit coupling (SOC). For example in the 
j=3/2 file, we  have four columns as expected:


 pdos(E)_1   pdos(E)_2   pdos(E)_3   pdos(E)_4
    0.000E+00  0.000E+00  0.000E+00  0.000E+00
    0.000E+00  0.000E+00  0.000E+00  0.000E+00
   0.000E+00  0.000E+00  0.000E+00  0.000E+00

Which column corresponds to which mj value? What is the underlying 
order? Is it -3/2, -1/2, 1/2, 3/2 or -3/2 3/2, -1/2, 1/2?


I tried to look at the PROJWFC documentation online but could not get 
a direct answer for the SOC case. Any help would be appreciated.


Thanks in advance

Elie
Federal University of Rondonia
Brazil.

___
Quantum ESPRESSO is supported by MaX (www.max-centre.eu)
users mailing list users@lists.quantum-espresso.org
https://lists.quantum-espresso.org/mailman/listinfo/users


--
Dr. rer. nat. Thomas Brumme
Theoretical chemistry
TU Dresden - BAR / II49
Helmholtzstr. 18
01069 Dresden

Tel:  +49 (0)351 463 40844

email: thomas.bru...@tu-dresden.de

___
Quantum ESPRESSO is supported by MaX (www.max-centre.eu)
users mailing list users@lists.quantum-espresso.org
https://lists.quantum-espresso.org/mailman/listinfo/users

[QE-users] Interpolation for 2D phonons results in one imaginary branch near Gamma

2021-07-07 Thread Thomas Brumme

Dear all,

I think I don't see the wood for the trees and hope that someone can help.

I want to calculate the phonons for a 2D material using the 2D cutoff 
technique. The scf calculations for the well-relaxed system work without 
a problem and I also get no error calculating the phonons on a 5x5x1 
q-point grid. The relevant portions of the scf and phonon input are 
attached below. All frequencies are positive at every single q point, 
except the 3 acoustic branches at the Gamma point which are "only" -10 
cm^-1 and -15 cm^-1. However, the interpolation, in order to get the 
dispersion, results in one branch which is negative for quite some 
portion near Gamma - for the first half along G-K I have up to -130 
cm^-1 and along G-M up to -60 cm^-1. I guess that this is the 
out-of-plane acoustic branch and now I wonder if this is real (i.e., the 
system is really unstable) or if I make some obvious mistake. I will 
also check by explicitly calculating few q points in the relevant region 
but before I wanted to ask here.


Any help would be appreciated!

Regards

Thomas Brumme


scf:


    calculation   = 'scf'
    prefix    = 'pasc12',
    pseudo_dir    = '/home/tbrumme/pseudos/pslibrary.1.0.0/rel-pbe/',
    outdir    = '/scratch/ws/0/tbrumme-pasc/',
    verbosity = 'high',
    wf_collect    = .true.,
/


   ibrav = 0,
 nat = 14,
    ntyp = 3,
 ecutwfc = 70.0,
 ecutrho = 560,
    noncolin = .true.,
    lspinorb = .true.,
    vdw_corr = 'DFT-D',
    nbnd = 96,
 assume_isolated = '2D',
/


 conv_thr = 1.D-14,
/

ATOMIC_SPECIES
    C 12.0 'C.rel-pbe-n-kjpaw_psl.1.0.0.UPF'
    P 30.973761 'P.rel-pbe-n-kjpaw_psl.1.0.0.UPF'
   As 74.921595 'As.rel-pbe-dn-kjpaw_psl.1.0.0.UPF'

ATOMIC_POSITIONS crystal
...

CELL_PARAMETERS angstrom
 6.69668  0.0   0.0
-3.348340 5.799495  0.0
 0.0  0.0  15.0

K_POINTS automatic
  25 25 1 0 0 0


phonons:

Phonon dispersions for pasc12
 
  tr2_ph=1.0d-18,
  prefix='pasc12',
  ldisp=.true.,
  nq1=5, nq2=5, nq3=1,
  outdir='/scratch/ws/0/tbrumme-pasc/',
  fildyn='pasc12.dyn.xml',
  max_seconds = 83700,
  verbosity='high',
  recover=.true.
  alpha_mix(1)=0.4,
 /


q2r and matdyn:


  fildyn='pasc12.dyn.xml', flfrc='pasc12.551.fc', loto_2d=.false., 
zasr='simple',

/


    flfrc='pasc12.551.fc.xml', flfrq='pasc12.freq', 
q_in_band_form=.true., loto_2d=.false., asr='simple',

    fldyn='pasc12.dyn.xml'
 /
 4
  0. 0. 0.0   60
  0. 0.57735027 0.0   30
  0. 0.57735027 0.0   45
  0. 0. 0.0   1

--
Dr. rer. nat. Thomas Brumme
Theoretical chemistry
TU Dresden - BAR / II49
Helmholtzstr. 18
01069 Dresden

Tel:  +49 (0)351 463 40844

email: thomas.bru...@tu-dresden.de

___
Quantum ESPRESSO is supported by MaX (www.max-centre.eu)
users mailing list users@lists.quantum-espresso.org
https://lists.quantum-espresso.org/mailman/listinfo/users

Re: [QE-users] How to extract the px, py, pz, dz2, ....contributions with SOC from projdos out

2021-07-04 Thread Thomas Brumme

Dear Elie,


The short answer is: You can't!


Spin-orbit interaction couples the orbital angular momentum with the 
spin momentum and thus neither l or s are good quantum number anymore. 
You now have to use the total angular momentum J. Sure, a lot of people 
still use the nomenclature from the non-SOC calculations, such as 
speaking in the case of 2D TMDCs of dz² states at the K point, but 
strictly speaking this is not correct. If you really want to understand 
the relation between J and L in detail, I can recommend the paper by 
Andrea Dal Corso and Adriano Mosca Conte, Phys. Rev. B 71, 115106 (2005) 
(https://link.aps.org/doi/10.1103/PhysRevB.71.115106) which nicely shows 
which spherical harmonics are included in which spin-angle functions for 
which total angular momentum j. If I remember this correctly (some time 
ago that I did this and I can't find the table anymore) the states with 
higher mj (+-3/2 and +- 5/2) have more in-plane characterfor both j=3/5 
and j=5/2 (i.e., the contain terms with spherical harmonics which are 
usually combined such that the result is in the xy-plane) while the 
states with mj=+-1/2 have more out-of-plane character (z direction). But 
I could be wrong here since I don't have the details anymore.



The only option for you - if you don't want to check the paper or if 
this is too much and if nobody else comments here - is to do a non-SOC 
calculation and then hope that SOC is not mixing states too much and you 
can still call the result, e.g., dz² like. OR you plot the corresponding 
wave function in real space and decide by "looking" at the form of |psi|².



Regards


Thomas



P.S.: another detail concerning, e.g., the states at K in a WS2 
monolayer - is this your system? :)


The states are not simple split into spin up and spin down even if a lot 
of people use this nomenclature. For the valence band the two SOC-split 
bands are to nearly 100% spin up or down but not for the conduction band 
where you won't have states which are 100% up or down, even if there is 
no in-plane contribution... Some details can also be found here: 
https://link.aps.org/doi/10.1103/PhysRevB.101.235408



On 7/4/21 6:52 AM, Elio Physics wrote:

Dear all,

I am studying the contribution of the orbitals to the bands of a 
structure, in the presence of spin orbit coupling. At the beginning of 
the projwfc.x output file, I got:


state #   1: atom   1 (S  ), wfc  1 (l=0 j=0.5 m_j=-0.5)
     state #   2: atom   1 (S  ), wfc  1 (l=0 j=0.5 m_j= 0.5)
     state #   3: atom   1 (S  ), wfc  2 (l=1 j=0.5 m_j=-0.5)
     state #   4: atom   1 (S  ), wfc  2 (l=1 j=0.5 m_j= 0.5)
     state #   5: atom   1 (S  ), wfc  3 (l=1 j=1.5 m_j=-1.5)
     state #   6: atom   1 (S  ), wfc  3 (l=1 j=1.5 m_j=-0.5)
     state #   7: atom   1 (S  ), wfc  3 (l=1 j=1.5 m_j= 0.5)
     state #   8: atom   1 (S  ), wfc  3 (l=1 j=1.5 m_j= 1.5)
.
.
.
state #  39: atom   5 (W ), wfc  3 (l=2 j=1.5 m_j=-1.5)
     state #  40: atom   5 (W), wfc  3 (l=2 j=1.5 m_j=-0.5)
     state #  41: atom   5 (W), wfc  3 (l=2 j=1.5 m_j= 0.5)
     state #  42: atom   5 (W), wfc  3 (l=2 j=1.5 m_j= 1.5)
     state #  43: atom   5 (W ), wfc  4 (l=2 j=2.5 m_j=-2.5)
     state #  44: atom   5 (W ), wfc  4 (l=2 j=2.5 m_j=-1.5)
     state #  45: atom   5 (W), wfc  4 (l=2 j=2.5 m_j=-0.5)
     state #  46: atom   5 (W), wfc  4 (l=2 j=2.5 m_j= 0.5)
     state #  47: atom   5 (W), wfc  4 (l=2 j=2.5 m_j= 1.5)

The l=1 wavefunctions are the p contributions. But How can we 
specifically identify which one is px, py and pz?
Similary, how to identify which ones of the 10 d orbitals are the dz^2 
for example


regards

Elie
Federal Universiy of Rondonia
Brazil

___
Quantum ESPRESSO is supported by MaX (www.max-centre.eu)
users mailing list users@lists.quantum-espresso.org
https://lists.quantum-espresso.org/mailman/listinfo/users


--
Dr. rer. nat. Thomas Brumme
Theoretical chemistry
TU Dresden - BAR / II49
Helmholtzstr. 18
01069 Dresden

Tel:  +49 (0)351 463 40844

email: thomas.bru...@tu-dresden.de

___
Quantum ESPRESSO is supported by MaX (www.max-centre.eu)
users mailing list users@lists.quantum-espresso.org
https://lists.quantum-espresso.org/mailman/listinfo/users

Re: [QE-users] phonon of 2D materials by tetrahedra_opt method

2021-06-13 Thread Thomas Brumme
Dear Yipeng,

My guess: Maybe this is due to the fact that a tetrahedron is a 3D object which 
cannot be properly created by points in a 2D plane...

Kind regards

Thomas Brumme

TU Dresden, Germany

Von: users  im Auftrag von 526587466 
<526587...@qq.com>
Gesendet: Sonntag, 13. Juni 2021 11:14:19
An: QEforum
Betreff: [QE-users] phonon of 2D materials by tetrahedra_opt method

Dear All,

When doing a test of graphene by the tetrahedra_opt  method,  I fount it prints 
the complete negative frequencies if using the nq (6x6x1), while right for nq 
(6x6x2) but cost more times.

So, what's wrong?   How to  calculate the phonon of 2D materials by 
tetrahedra_opt  method.  Should the number of nq3  be larger than 1 for the 
tetrahedra_opt  method?

Thanks.

Best regards,

Yipeng

Henan Normal University, Xinxiang  China


___
Quantum ESPRESSO is supported by MaX (www.max-centre.eu)
users mailing list users@lists.quantum-espresso.org
https://lists.quantum-espresso.org/mailman/listinfo/users

Re: [QE-users] My slab system does not converge during optimization.Can you help me?

2021-05-07 Thread Thomas Brumme

Dear Jayfe,

good to see that you managed to solve the problem :) Next time you can 
help someone else ;)
Remember that once you use the dipole correction you have to set eamp to 
zero since there is a finite default value.


Cheerio

Thomas

On 5/7/21 8:37 AM, Jayfe Anthony Abrea wrote:

Dear Dr. Thomas,

For the long time tweaking and troubleshooting, I was able to provide 
a way for my calculations involving Mn to converge. I want to thank 
you for your inputs and suggestions.


What I did is to follow your suggestion of changing the Mn 
pseudopotential from 1.0.0 to the older 0.3.1 version. I'm not sure 
why it converges on the older version in contrast to the newer one. It 
was the least expected change that I wanted to do to my setup but 
somehow it works. I redo the convergence test using 0.3.1 version and 
I found the convergence at around 65 Ry cutoff, same as the one 
suggested by the materialscloud.org <http://materialscloud.org> site. 
There's only a difference of 0.0015 Ry on calculated energies between 
using 65 Ry cutoff and 120 Ry cutoff. Using beyond 100 Ry cutoff has 
driven my computer crazy thus I would opt to use the least cutoff 
viable for me. I'd be using the dual of 12 to be safe. I can also 
revert back to a simpler setup (e.g. removing eamp, esm; using 
'david') for my further calculations.


I am grateful for your time looking at my predicament Dr. Thomas. Such 
a pleasure having a correspondence with you. Hoping to keep in touch 
with you in the future.


Thank you once again.

Cheers,

Jayfe Anthony Abrea
Graduate Student
University of San Carlos

On Wed, Apr 21, 2021 at 2:48 PM Thomas Brumme 
mailto:tbru...@msx.tu-dresden.de>> wrote:


Dear Jayfe,

having a closer look at your input I'm a bit confused about some
things but lets try to stick first to your specific
problem - you mentioned in your first email, that the calculation
is running with elements other than Mn and
that you thus think that Mn is the problem.

Now, did you try replacing the pseudopotential?

Looking at the graph of the SSSP:
https://www.materialscloud.org/discover/sssp/plot/precision/Mn
<https://www.materialscloud.org/discover/sssp/plot/precision/Mn>
I see that the pseudopotential you're using might have some
problem - even with a dual of 16 it is not really
looking good until a cutoff of about 120 Ry. Sure, phonons and
related properties are different from standard
scf convergence, but it is weird. Maybe try using the older 0.3.1
version of the pslibrary.

Did you try to increase the cutoff?

Considering the very same graph of the SSSP, I think that a cutoff
of 45 Ry is just not enough for Mn.
Try increasing in reasonable steps (i.e., 50, 60, 70) with a dual
of 8 or 16 for the 0.3.1 or the 1.0.0 version of
the pslibrary, respectively. If it converges (only one scf, no
relaxation, but printing the forces) compare results
to see what might be a converged value. How big is the change in
the total energy? How do the forces
converge? What about magnetization? And so on and so forth... The
standard way to get a reasonable set of
ecutwfc and ecutrho for your system...

So, concerning your input:

I know that you created the input with some tool, but it does not
make sense. You should check this...
First of all, delete everything which you don't use, just to be
sure that there is no problem. Why do you set
esm parameters, if you don't use the method? Why is eamp still set
to a finite value? Before trying some
different diagonalization methods try the standard 'david'.
startingpot and startingwfc usually don't need to
be set.

In summary, I would try to change the pseudo and increase the
cutoff. And I would clean the input. Then you
need to make convergence checks.

Regards

Thomas


On 4/20/21 4:55 PM, Jayfe Anthony Abrea wrote:


Hello Dr. Thomas,

First of all, I would like to thank you for giving your inputs on
my case last week. I do hope you can still extend a helping hand
on this predicament of mine.

I would like to give you an update on my predicament on
optimization process. Attached in this email is my input file in
which the eamp was set to zero. During calculations, the results
are still oscillating for each iteration. I didn’t stopped the
calculation but I can see that it’s not going better. It even
gave a positive value for calculated scf energy per iteration as
seen in this pic:


https://cviscpshs-my.sharepoint.com/:i:/g/personal/jaabrea_cvisc_pshs_edu_ph/Eat1rLSbCO9Mth7ra0er6PwBfrWsXtUj9s7f_mOPkckzgA?e=lQWb4L

<https://cviscpshs-my.sharepoint.com/:i:/g/personal/jaabrea_cvisc_pshs_edu_ph/Eat1rLSbCO9Mth7ra0er6PwBfrWsXtUj9s7f_mOPkckzgA?e=lQWb4L>

The accuracies for each iteration are very low for each iteration
as seen in this pic:

Re: [QE-users] My slab system does not converge during optimization.Can you help me?

2021-04-21 Thread Thomas Brumme

Dear Jayfe,

having a closer look at your input I'm a bit confused about some things 
but lets try to stick first to your specific
problem - you mentioned in your first email, that the calculation is 
running with elements other than Mn and

that you thus think that Mn is the problem.

Now, did you try replacing the pseudopotential?

Looking at the graph of the SSSP: 
https://www.materialscloud.org/discover/sssp/plot/precision/Mn
I see that the pseudopotential you're using might have some problem - 
even with a dual of 16 it is not really
looking good until a cutoff of about 120 Ry. Sure, phonons and related 
properties are different from standard
scf convergence, but it is weird. Maybe try using the older 0.3.1 
version of the pslibrary.


Did you try to increase the cutoff?

Considering the very same graph of the SSSP, I think that a cutoff of 45 
Ry is just not enough for Mn.
Try increasing in reasonable steps (i.e., 50, 60, 70) with a dual of 8 
or 16 for the 0.3.1 or the 1.0.0 version of
the pslibrary, respectively. If it converges (only one scf, no 
relaxation, but printing the forces) compare results
to see what might be a converged value. How big is the change in the 
total energy? How do the forces
converge? What about magnetization? And so on and so forth... The 
standard way to get a reasonable set of

ecutwfc and ecutrho for your system...

So, concerning your input:

I know that you created the input with some tool, but it does not make 
sense. You should check this...
First of all, delete everything which you don't use, just to be sure 
that there is no problem. Why do you set
esm parameters, if you don't use the method? Why is eamp still set to a 
finite value? Before trying some
different diagonalization methods try the standard 'david'. startingpot 
and startingwfc usually don't need to

be set.

In summary, I would try to change the pseudo and increase the cutoff. 
And I would clean the input. Then you

need to make convergence checks.

Regards

Thomas


On 4/20/21 4:55 PM, Jayfe Anthony Abrea wrote:


Hello Dr. Thomas,

First of all, I would like to thank you for giving your inputs on my 
case last week. I do hope you can still extend a helping hand on this 
predicament of mine.


I would like to give you an update on my predicament on optimization 
process. Attached in this email is my input file in which the eamp was 
set to zero. During calculations, the results are still oscillating 
for each iteration. I didn’t stopped the calculation but I can see 
that it’s not going better. It even gave a positive value for 
calculated scf energy per iteration as seen in this pic:


https://cviscpshs-my.sharepoint.com/:i:/g/personal/jaabrea_cvisc_pshs_edu_ph/Eat1rLSbCO9Mth7ra0er6PwBfrWsXtUj9s7f_mOPkckzgA?e=lQWb4L 
<https://cviscpshs-my.sharepoint.com/:i:/g/personal/jaabrea_cvisc_pshs_edu_ph/Eat1rLSbCO9Mth7ra0er6PwBfrWsXtUj9s7f_mOPkckzgA?e=lQWb4L>


The accuracies for each iteration are very low for each iteration as 
seen in this pic:


https://cviscpshs-my.sharepoint.com/:i:/g/personal/jaabrea_cvisc_pshs_edu_ph/Ebnkk0mfJjlKiFkMffu63HkBYml45BOtNl3q8JjB_JkTJQ?e=ZRpyqc 
<https://cviscpshs-my.sharepoint.com/:i:/g/personal/jaabrea_cvisc_pshs_edu_ph/Ebnkk0mfJjlKiFkMffu63HkBYml45BOtNl3q8JjB_JkTJQ?e=ZRpyqc>


Then let me share to you the ongoing result for the magnetization for 
each iteration in this pic:


https://cviscpshs-my.sharepoint.com/:i:/g/personal/jaabrea_cvisc_pshs_edu_ph/Eat1rLSbCO9Mth7ra0er6PwBrm1Ki8UqakTaleeJ8Nleng?e=4oIMv9 
<https://cviscpshs-my.sharepoint.com/:i:/g/personal/jaabrea_cvisc_pshs_edu_ph/Eat1rLSbCO9Mth7ra0er6PwBrm1Ki8UqakTaleeJ8Nleng?e=4oIMv9>


I hope you can shed light to this matter Dr. Thomas. I’m not sure now 
as to why the Mn cannot undergo proper optimization unlike the Fe and Co.


Best regards,

Jayfe

Sent from Mail <https://go.microsoft.com/fwlink/?LinkId=550986> for 
Windows 10



--
Dr. rer. nat. Thomas Brumme
Theoretical chemistry
TU Dresden - BAR / II49
Helmholtzstr. 18
01069 Dresden

Tel:  +49 (0)351 463 40844

email: thomas.bru...@tu-dresden.de

___
Quantum ESPRESSO is supported by MaX (www.max-centre.eu)
users mailing list users@lists.quantum-espresso.org
https://lists.quantum-espresso.org/mailman/listinfo/users

Re: [QE-users] My slab system does not converge during optimization.Can you help me?

2021-04-16 Thread Thomas Brumme

Dear Jayfe,

eamp gives the amplitude of the external electric field. Since the 
dipole is internal it is correct to set eamp = 0
The dipole correction will self consistently determine the required 
dipole - how do you want to know this before?


Regards

Thomas

On 4/16/21 1:43 PM, Jayfe Anthony Abrea wrote:


Hello Dr. Thomas,

[I am still adjusting with this QE mailing list. Thus, I made 
duplicate reply. Please bear with me Dr. Thomas]


I’m glad that you took notice on my predicament. Thank you Dr. Thomas. 
My appreciations.


The input file I placed in this email contains my initial positions of 
the atoms. The initial distance between H and surface is actually that 
“bad” or that far. I used the optimized positions of the slab with Fe 
instead of Mn and applied it to my Pt/Mn slab in the hopes that the 
atomic positions are closer to optimization. The z atomic position of 
H went down from 0.618734 to 0.607984. It still gave a similar 
oscillating result. =(


As for the applied external electric field, I’ve read that eamp, along 
with edir, emaxpos, and eopreg are adjusted for dipole correction of 
my slab system. I am not sure if making eamp to zero can still provide 
dipole correction. Will it still be OK to make it zero?


Best regards,

Jayfe

Sent from Mail <https://go.microsoft.com/fwlink/?LinkId=550986> for 
Windows 10


*From: *Thomas Brumme <mailto:tbru...@msx.tu-dresden.de>
*Sent: *Friday, April 16, 2021 4:03 PM
*To: *Quantum ESPRESSO users Forum 
<mailto:users@lists.quantum-espresso.org>; Jayfe Anthony Abrea 
<mailto:06302...@usc.edu.ph>
*Subject: *Re: [QE-users] My slab system does not converge during 
optimization.Can you help me?


Hi Jayfe,

Maybe the initial distance between hydrogen and the surface is just 
"bad"? Maybe reduce it a bit and see if this helps.

Another thing I noticed:
Do you really want an external electric field? Because the eamp is 
nonzero...


Regards

Thomas

On 4/15/21 4:05 PM, Jayfe Anthony Abrea wrote:

Hello fellow QE users and developers,

I am new to Quantum Espresso and I am doing calculations on
adsorption energy. I am using BURAI 1.3 GUI to aid in my QE setup
thus I am not sure of the QE version that I am using. I am trying
to do optimization on the hydrogen atom on the bimetallic slab
(see attached input file). I have already employed several
tweakings since the calculations never converged, such as:

- changed the mixing_mode to local-TF since it is suited for
inhomogeneous system like my slab system

- lowering the mixing_beta to 0.1

- changing the ecutrho, ecutwfc, and starting_magnetization to the
ones recommended by this website:
http://www.materialscloud.org/work/tools/qeinputgenerator
<http://www.materialscloud.org/work/tools/qeinputgenerator>

These tweakings are made based on what I have read so far in this
forum. After calculations, the slab-adsorbate system didn't
achieve convergence in optimization (as seen in attached pics on
SCF and accuracy). Manganese in my system seems to be problematic
since when I tried replacing Mn with other metals (e.g. Fe and Co
in particular), it reaches convergence and becomes optimized. I
don't know now how to resolve this issue. I am hoping that you
have great insights to address my plight.

I would really appreciate your help in this regard. Thank you!

Cheers,

Jayfe Anthony

Student

University of San Carlos



___

Quantum ESPRESSO is supported by MaX (www.max-centre.eu  
<http://www.max-centre.eu>)

users mailing listus...@lists.quantum-espresso.org  
<mailto:users@lists.quantum-espresso.org>

https://lists.quantum-espresso.org/mailman/listinfo/users  
<https://lists.quantum-espresso.org/mailman/listinfo/users>

--
Dr. rer. nat. Thomas Brumme
Theoretical chemistry
TU Dresden - BAR / II49
Helmholtzstr. 18
01069 Dresden
Tel:  +49 (0)351 463 40844
email:thomas.bru...@tu-dresden.de  <mailto:thomas.bru...@tu-dresden.de>


--
Dr. rer. nat. Thomas Brumme
Theoretical chemistry
TU Dresden - BAR / II49
Helmholtzstr. 18
01069 Dresden

Tel:  +49 (0)351 463 40844

email: thomas.bru...@tu-dresden.de

___
Quantum ESPRESSO is supported by MaX (www.max-centre.eu)
users mailing list users@lists.quantum-espresso.org
https://lists.quantum-espresso.org/mailman/listinfo/users

Re: [QE-users] My slab system does not converge during optimization. Can you help me?

2021-04-16 Thread Thomas Brumme

Hi Jayfe,

Maybe the initial distance between hydrogen and the surface is just 
"bad"? Maybe reduce it a bit and see if this helps.

Another thing I noticed:
Do you really want an external electric field? Because the eamp is 
nonzero...


Regards

Thomas

On 4/15/21 4:05 PM, Jayfe Anthony Abrea wrote:

Hello fellow QE users and developers,

I am new to Quantum Espresso and I am doing calculations on adsorption 
energy. I am using BURAI 1.3 GUI to aid in my QE setup thus I am not 
sure of the QE version that I am using. I am trying to do optimization 
on the hydrogen atom on the bimetallic slab (see attached input file). 
I have already employed several tweakings since the calculations never 
converged, such as:


- changed the mixing_mode to local-TF since it is suited for 
inhomogeneous system like my slab system

- lowering the mixing_beta to 0.1
- changing the ecutrho, ecutwfc, and starting_magnetization to the 
ones recommended by this website: 
http://www.materialscloud.org/work/tools/qeinputgenerator 
<http://www.materialscloud.org/work/tools/qeinputgenerator>


These tweakings are made based on what I have read so far in this 
forum. After calculations, the slab-adsorbate system didn't achieve 
convergence in optimization (as seen in attached pics on SCF and 
accuracy). Manganese in my system seems to be problematic since when I 
tried replacing Mn with other metals (e.g. Fe and Co in particular), 
it reaches convergence and becomes optimized. I don't know now how to 
resolve this issue. I am hoping that you have great insights to 
address my plight.


I would really appreciate your help in this regard. Thank you!

Cheers,

Jayfe Anthony
Student
University of San Carlos

___
Quantum ESPRESSO is supported by MaX (www.max-centre.eu)
users mailing list users@lists.quantum-espresso.org
https://lists.quantum-espresso.org/mailman/listinfo/users


--
Dr. rer. nat. Thomas Brumme
Theoretical chemistry
TU Dresden - BAR / II49
Helmholtzstr. 18
01069 Dresden

Tel:  +49 (0)351 463 40844

email: thomas.bru...@tu-dresden.de

___
Quantum ESPRESSO is supported by MaX (www.max-centre.eu)
users mailing list users@lists.quantum-espresso.org
https://lists.quantum-espresso.org/mailman/listinfo/users

Re: [QE-users] script file for plotting Band structure and density of states

2020-11-27 Thread Thomas Brumme
Dear Soumyakanta Panda,


please have a look here:


https://www.quantum-espresso.org/Doc/INPUT_PP.html


or the different examples in the PP subdirectory of the QE distribution.

The manual can also help. Or you try the GUI:


http://www-k3.ijs.si/kokalj/pwgui/

but I never used it and thus can't help.

Regards

Thomas???

--
Dr. rer. nat. Thomas Brumme
Theoretical chemistry
TU Dresden - BAR / II49
Helmholtzstr. 18
01069 Dresden

Tel:  +49 (0)351 463 40844

email: thomas.bru...@tu-dresden.de


Von: users  im Auftrag von 
SOUMYAKANTA PANDA via users 
Gesendet: Freitag, 27. November 2020 06:39
An: Quantum ESPRESSO users Forum
Betreff: [QE-users] script file for plotting Band structure and density of 
states

Hi users
As i am new to quantum espresso so i want a script file for plotting projected 
DOS and projected band structure. Kindly provide the  link or file where i get 
support.
Best Regards,
Soumyakanta Panda
Research Scholar
Nano Magnetism and Magnetic Materials Laboratory
IIT Bhubaneswar


Disclaimer: This email and any files transmitted with it are confidential and 
intended solely for the use of the individual or entity to whom they are 
addressed. If you have received this email in error please notify the system 
manager. This message contains confidential information and is intended only 
for the individual named. If you are not the named addressee you should not 
disseminate, distribute or copy this e-mail. Please notify the sender 
immediately by e-mail if you have received this e-mail by mistake and delete 
this e-mail from your system. If you are not the intended recipient you are 
notified that disclosing, copying, distributing or taking any action in 
reliance on the contents of this information is strictly prohibited.
___
Quantum ESPRESSO is supported by MaX (www.max-centre.eu)
users mailing list users@lists.quantum-espresso.org
https://lists.quantum-espresso.org/mailman/listinfo/users

Re: [QE-users] QE Parallelization for Phonon Dispersion calculations

2020-11-24 Thread Thomas Brumme
Dear Kiran,


if I look at the run_example_1 I see that there is only the splitting via the q 
points.

An irrep, or irreducible representation is the (possibly) degenerated phonon 
mode

at one q point... Splitting in q points is much easier than on irreps which I 
would not

recommend it for a beginner...


Cheers


Thomas


--
Dr. rer. nat. Thomas Brumme
Theoretical chemistry
TU Dresden - BAR / II49
Helmholtzstr. 18
01069 Dresden

Tel:  +49 (0)351 463 40844

email: thomas.bru...@tu-dresden.de


Von: Kiran Yadav 
Gesendet: Dienstag, 24. November 2020 13:22
An: Brumme, Thomas
Betreff: Re: [QE-users] QE Parallelization for Phonon Dispersion calculations

Dear Dr Thomas,
Thanks a lot for the reference. I looked at the run example and run example 1, 
In the run example, they have "Both q-points and irreps are split." If possible 
please answer on - What are irreps here and how does it affects our phonon 
calculation parallelization?
From my output file, I got 20 q-points and q in seq '1 20' but couldn't find 
irr.
KIndly help.
Thanks a lot again

Regards
Kiran Yadav





On Tue, Nov 24, 2020 at 5:10 PM Thomas Brumme 
mailto:thomas.bru...@tu-dresden.de>> wrote:

Dear Kiran,


please have a look at the GRID_example in the PHonon examples and there the 
run_example_1​ in particular.


Regards


Thomas

--
Dr. rer. nat. Thomas Brumme
Theoretical chemistry
TU Dresden - BAR / II49
Helmholtzstr. 18
01069 Dresden

Tel:  +49 (0)351 463 40844

email: thomas.bru...@tu-dresden.de<mailto:thomas.bru...@tu-dresden.de>


Von: users 
mailto:users-boun...@lists.quantum-espresso.org>>
 im Auftrag von Kiran Yadav 
mailto:kiranyadav0...@gmail.com>>
Gesendet: Dienstag, 24. November 2020 10:45
An: Quantum ESPRESSO users Forum
Betreff: Re: [QE-users] QE Parallelization for Phonon Dispersion calculations

Dear Lorenzo,
Thanks a lot for the great help.
If possible, please suggest some reference on how to run each q-point 
simultaneously or do as you have suggested.

Thanks & Regards,
Kiran Yadav




On Tue, Nov 24, 2020 at 3:09 PM Lorenzo Paulatto 
mailto:paul...@gmail.com>> wrote:

Images are only used inside each q-point, if I remember correctly, so it would 
not matter in our question.

But, you can decrease walltime much more by running each q-point simultaneously 
and independently using startq, startq options and a job for each. It is 
sufficient to use a different prefix or outdir. The data from pw can be copied 
or repeated, it does not usually matter in terms of CPU time.

cheers

On 2020-11-24 10:27, Kiran Yadav wrote:
Dear Lorenzo,
Is there any parallelization method so that I can make images such that 20 dyn 
files will get distributed unequally on the equal number of processors or dyn 
files will get distributed equally on unequal number of processors ?

Because in my case as per my observation dyn1-10 are not taking too much time, 
most of the time has been spent on dyn11-dyn20 generation. So, If I could run 
dyn 1-10  on one image and distribute remaining dyn11-20 on 3 images CPU 
walltime can be decreased. Is it possible to do something like that?

I tried parallelization on 256 processors (#PBS -l select=16:ncpus=16) for 
10hrs using the following parallelization command
time -p mpirun -np $PBS_NTASKS ph.x -ni 4 -nk 4 -nt 4 -nd 16 -input  
ph.in<http://ph.in/> > ph.out
in this case 20dyn files got distributed in 4 images i.e 5 dyn on one image.  
256/4=

Thanks & Regards,
Kiran Yadav
Research Scholar
Electronic Materials Laboratory (TX-200G)
Dept. of Materials Science & Engineering
Indian Institute of Technology, Delhi




On Tue, Nov 24, 2020 at 1:19 PM Lorenzo Paulatto 
mailto:paul...@gmail.com>> wrote:

> I have been trying to calculate Phonon dispersion with 6*6*6 nq grid.
> It generates dyn0+20 other dynamical matrices, but the time taken by
> each dynamical matrix file is different for completion. I ran these
> phonon dispersion calculations using parallelization, but couldn't
> optimize correctly.

This is normal, because different q-points have different symmetries,
the code has to use different numbers of k-points.


regards


___
Quantum ESPRESSO is supported by MaX 
(www.max-centre.eu<http://www.max-centre.eu>)
users mailing list 
users@lists.quantum-espresso.org<mailto:users@lists.quantum-espresso.org>
https://lists.quantum-espresso.org/mailman/listinfo/users



___
Quantum ESPRESSO is supported by MaX 
(www.max-centre.eu<http://www.max-centre.eu>)
users mailing list 
users@lists.quantum-espresso.org<mailto:users@lists.quantum-espresso.org>
https://lists.quantum-espresso.org/mailman/listinfo/users

___
Quantum ESPRESSO is supported by MaX 
(www.max-centre.eu<http://www.max-centre.eu>)
users mailing li

Re: [QE-users] QE Parallelization for Phonon Dispersion calculations

2020-11-24 Thread Thomas Brumme
Dear Kiran,


please have a look at the GRID_example in the PHonon examples and there the 
run_example_1? in particular.


Regards


Thomas

--
Dr. rer. nat. Thomas Brumme
Theoretical chemistry
TU Dresden - BAR / II49
Helmholtzstr. 18
01069 Dresden

Tel:  +49 (0)351 463 40844

email: thomas.bru...@tu-dresden.de


Von: users  im Auftrag von Kiran 
Yadav 
Gesendet: Dienstag, 24. November 2020 10:45
An: Quantum ESPRESSO users Forum
Betreff: Re: [QE-users] QE Parallelization for Phonon Dispersion calculations

Dear Lorenzo,
Thanks a lot for the great help.
If possible, please suggest some reference on how to run each q-point 
simultaneously or do as you have suggested.

Thanks & Regards,
Kiran Yadav




On Tue, Nov 24, 2020 at 3:09 PM Lorenzo Paulatto 
mailto:paul...@gmail.com>> wrote:

Images are only used inside each q-point, if I remember correctly, so it would 
not matter in our question.

But, you can decrease walltime much more by running each q-point simultaneously 
and independently using startq, startq options and a job for each. It is 
sufficient to use a different prefix or outdir. The data from pw can be copied 
or repeated, it does not usually matter in terms of CPU time.

cheers

On 2020-11-24 10:27, Kiran Yadav wrote:
Dear Lorenzo,
Is there any parallelization method so that I can make images such that 20 dyn 
files will get distributed unequally on the equal number of processors or dyn 
files will get distributed equally on unequal number of processors ?

Because in my case as per my observation dyn1-10 are not taking too much time, 
most of the time has been spent on dyn11-dyn20 generation. So, If I could run 
dyn 1-10  on one image and distribute remaining dyn11-20 on 3 images CPU 
walltime can be decreased. Is it possible to do something like that?

I tried parallelization on 256 processors (#PBS -l select=16:ncpus=16) for 
10hrs using the following parallelization command
time -p mpirun -np $PBS_NTASKS ph.x -ni 4 -nk 4 -nt 4 -nd 16 -input  
ph.in<http://ph.in/> > ph.out
in this case 20dyn files got distributed in 4 images i.e 5 dyn on one image.  
256/4=

Thanks & Regards,
Kiran Yadav
Research Scholar
Electronic Materials Laboratory (TX-200G)
Dept. of Materials Science & Engineering
Indian Institute of Technology, Delhi




On Tue, Nov 24, 2020 at 1:19 PM Lorenzo Paulatto 
mailto:paul...@gmail.com>> wrote:

> I have been trying to calculate Phonon dispersion with 6*6*6 nq grid.
> It generates dyn0+20 other dynamical matrices, but the time taken by
> each dynamical matrix file is different for completion. I ran these
> phonon dispersion calculations using parallelization, but couldn't
> optimize correctly.

This is normal, because different q-points have different symmetries,
the code has to use different numbers of k-points.


regards


___
Quantum ESPRESSO is supported by MaX 
(www.max-centre.eu<http://www.max-centre.eu>)
users mailing list 
users@lists.quantum-espresso.org<mailto:users@lists.quantum-espresso.org>
https://lists.quantum-espresso.org/mailman/listinfo/users



___
Quantum ESPRESSO is supported by MaX 
(www.max-centre.eu<http://www.max-centre.eu>)
users mailing list 
users@lists.quantum-espresso.org<mailto:users@lists.quantum-espresso.org>
https://lists.quantum-espresso.org/mailman/listinfo/users

___
Quantum ESPRESSO is supported by MaX 
(www.max-centre.eu<http://www.max-centre.eu>)
users mailing list 
users@lists.quantum-espresso.org<mailto:users@lists.quantum-espresso.org>
https://lists.quantum-espresso.org/mailman/listinfo/users
___
Quantum ESPRESSO is supported by MaX (www.max-centre.eu)
users mailing list users@lists.quantum-espresso.org
https://lists.quantum-espresso.org/mailman/listinfo/users

Re: [QE-users] wfck2r.x

2020-11-05 Thread Thomas Brumme
Thanks Duy Le for the suggestion. Sometimes one does not see the wood for the 
trees...

Which leaves only one question:


Am I right, that for the SOC case the second component is missing if I use the 
output in octave format?


Thomas


--
Dr. rer. nat. Thomas Brumme
Theoretical chemistry
TU Dresden - BAR / II49
Helmholtzstr. 18
01069 Dresden

Tel:  +49 (0)351 463 40844

email: thomas.bru...@tu-dresden.de


Von: Duy Le 
Gesendet: Montag, 2. November 2020 16:56
An: Brumme, Thomas
Cc: users@lists.quantum-espresso.org
Betreff: Re: [QE-users] wfck2r.x

I doubt that you will be able to get anything meaningful with , as 
difference in the random phases of the two wavefunctions is not possible to be 
eliminated. How about ||^2?

Duy Le
(UCF)

On Mon, Nov 2, 2020 at 10:47 AM Thomas Brumme 
mailto:thomas.bru...@tu-dresden.de>> wrote:

OK, at least the different sizes were probably a problem on my side - I was 
using the dev version of wfck2r with the rest in an older version. Using only 
the dev version I have the binary file being approximately 2.7 times bigger 
which can be explained - I guess - with the missing second component. But I'm 
still not sure whether I can simply integrate  and expect 
something useful if both wave functions are from different calculations...


--
Dr. rer. nat. Thomas Brumme
Theoretical chemistry
TU Dresden - BAR / II49
Helmholtzstr. 18
01069 Dresden

Tel:  +49 (0)351 463 40844

email: thomas.bru...@tu-dresden.de<mailto:thomas.bru...@tu-dresden.de>


Von: users 
mailto:users-boun...@lists.quantum-espresso.org>>
 im Auftrag von Thomas Brumme 
mailto:thomas.bru...@tu-dresden.de>>
Gesendet: Montag, 2. November 2020 15:21
An: users@lists.quantum-espresso.org<mailto:users@lists.quantum-espresso.org>
Betreff: [QE-users] wfck2r.x


Dear all,


I want to calculate wave function overlaps in order to extract some parameters 
for a collaborator.

Since I want the overlap in a heterostructure of 2 monolayers for the wave 
functions of the single layers (i.e., I need 3 calculations), the easiest 
solution would be to look at the changes in the projection on the atomic states 
using projwfc. Yet, this local basis is not complete and I would like to know 
how "much" is missing.


I found the wfck2r.f90 code. Yet, I think there are several problems with using 
this and maybe someone can comment. I think I once read that one cannot easily 
compare the wave functions between different calculation because of an 
arbitrary phase shift. Is this correct? Because if I can't do this, then I 
don't even need to consider using the code. If I can do it, then am I right, 
that the wfck2r code does not include the SOC case for output in octave format? 
Because in line 248, only pol=1 is used... Another problem I noticed is a huge 
difference in sizes of the octave output and the binary output. The octave 
output was in my test case about 68 MB while the binary file was 6 GB.


Kind regards


Thomas

--
Dr. rer. nat. Thomas Brumme
Theoretical chemistry
TU Dresden - BAR / II49
Helmholtzstr. 18
01069 Dresden

Tel:  +49 (0)351 463 40844

email: thomas.bru...@tu-dresden.de<mailto:thomas.bru...@tu-dresden.de>

___
Quantum ESPRESSO is supported by MaX 
(www.max-centre.eu<http://www.max-centre.eu>)
users mailing list 
users@lists.quantum-espresso.org<mailto:users@lists.quantum-espresso.org>
https://lists.quantum-espresso.org/mailman/listinfo/users
___
Quantum ESPRESSO is supported by MaX (www.max-centre.eu)
users mailing list users@lists.quantum-espresso.org
https://lists.quantum-espresso.org/mailman/listinfo/users

Re: [QE-users] wfck2r.x

2020-11-02 Thread Thomas Brumme
OK, at least the different sizes were probably a problem on my side - I was 
using the dev version of wfck2r with the rest in an older version. Using only 
the dev version I have the binary file being approximately 2.7 times bigger 
which can be explained - I guess - with the missing second component. But I'm 
still not sure whether I can simply integrate  and expect 
something useful if both wave functions are from different calculations...


--
Dr. rer. nat. Thomas Brumme
Theoretical chemistry
TU Dresden - BAR / II49
Helmholtzstr. 18
01069 Dresden

Tel:  +49 (0)351 463 40844

email: thomas.bru...@tu-dresden.de


Von: users  im Auftrag von Thomas 
Brumme 
Gesendet: Montag, 2. November 2020 15:21
An: users@lists.quantum-espresso.org
Betreff: [QE-users] wfck2r.x


Dear all,


I want to calculate wave function overlaps in order to extract some parameters 
for a collaborator.

Since I want the overlap in a heterostructure of 2 monolayers for the wave 
functions of the single layers (i.e., I need 3 calculations), the easiest 
solution would be to look at the changes in the projection on the atomic states 
using projwfc. Yet, this local basis is not complete and I would like to know 
how "much" is missing.


I found the wfck2r.f90 code. Yet, I think there are several problems with using 
this and maybe someone can comment. I think I once read that one cannot easily 
compare the wave functions between different calculation because of an 
arbitrary phase shift. Is this correct? Because if I can't do this, then I 
don't even need to consider using the code. If I can do it, then am I right, 
that the wfck2r code does not include the SOC case for output in octave format? 
Because in line 248, only pol=1 is used... Another problem I noticed is a huge 
difference in sizes of the octave output and the binary output. The octave 
output was in my test case about 68 MB while the binary file was 6 GB.


Kind regards


Thomas

--
Dr. rer. nat. Thomas Brumme
Theoretical chemistry
TU Dresden - BAR / II49
Helmholtzstr. 18
01069 Dresden

Tel:  +49 (0)351 463 40844

email: thomas.bru...@tu-dresden.de
___
Quantum ESPRESSO is supported by MaX (www.max-centre.eu)
users mailing list users@lists.quantum-espresso.org
https://lists.quantum-espresso.org/mailman/listinfo/users

[QE-users] wfck2r.x

2020-11-02 Thread Thomas Brumme
Dear all,


I want to calculate wave function overlaps in order to extract some parameters 
for a collaborator.

Since I want the overlap in a heterostructure of 2 monolayers for the wave 
functions of the single layers (i.e., I need 3 calculations), the easiest 
solution would be to look at the changes in the projection on the atomic states 
using projwfc. Yet, this local basis is not complete and I would like to know 
how "much" is missing.


I found the wfck2r.f90 code. Yet, I think there are several problems with using 
this and maybe someone can comment. I think I once read that one cannot easily 
compare the wave functions between different calculation because of an 
arbitrary phase shift. Is this correct? Because if I can't do this, then I 
don't even need to consider using the code. If I can do it, then am I right, 
that the wfck2r code does not include the SOC case for output in octave format? 
Because in line 248, only pol=1 is used... Another problem I noticed is a huge 
difference in sizes of the octave output and the binary output. The octave 
output was in my test case about 68 MB while the binary file was 6 GB.


Kind regards


Thomas

--
Dr. rer. nat. Thomas Brumme
Theoretical chemistry
TU Dresden - BAR / II49
Helmholtzstr. 18
01069 Dresden

Tel:  +49 (0)351 463 40844

email: thomas.bru...@tu-dresden.de
___
Quantum ESPRESSO is supported by MaX (www.max-centre.eu)
users mailing list users@lists.quantum-espresso.org
https://lists.quantum-espresso.org/mailman/listinfo/users

[QE-users] PhD position at the TU Dresden, Germany

2020-06-12 Thread Thomas Brumme

Dear all,

I am seeking a candidate for a PhD position at the TU Dresden, Germany.

The successful candidate will investigate the electronic, optical, and 
electrical transport properties of various two-dimensional materials and 
heterostructures under the influence of strain fields using 
density-functional based methods. More specifically, she/he will 
investigate mono- and multilayer systems which are strained such that 
wrinkles or ripples are formed. Understanding the formation of these 
wrinkles in detail and unravelling the interrelation between the 
structure/morphology and the electronic properties are the main 
objectives of this project. The work will be embedded in the 
Collaborative Research Centre 1415 “Chemistry of Synthetic 
Two-Dimensional Materials” (https://tu-dresden.de/mn/chemie/sfb1415) and 
involves close collaboration with experimental and other theory groups, 
and strong commitment to scientific networking.


Requirements: University degree (M. Sc. or equivalent) in chemistry or 
physics, and profound knowledge in computational and theoretical 
physics/chemistry. Capability of team work, in particular with 
experimental partners, is essential. Skills in high-performance 
computing, solid-state materials, topological properties, and 
density-functional theory (FHI-aims and/or QuantumEspresso in 
particular) are welcome.


Please submit your comprehensive application including the usual 
documents as soon as possible by mail to: TU Dresden, Faculty of 
Science, Chair of Theoretical Chemistry, Dr. Thomas Brumme, 01062 
Dresden or preferably via the TU Dresden SecureMail Portal 
(https://securemail.tu-dresden.de) by sending it as a single 
pdf-document to antje.voel...@tu-dresden.de or to 
thomas.brum...@mailbox.tu-dresden.de. Please submit copies only, as your 
application will not be returned to you. Expenses incurred in attending 
interviews cannot be reimbursed.


Regards

Thomas

--
Dr. rer. nat. Thomas Brumme
Wilhelm-Ostwald-Institute for Physical and Theoretical Chemistry
Leipzig University
Phillipp-Rosenthal-Strasse 31
04103 Leipzig

Tel:  +49 (0)341 97 36456

email: thomas.bru...@uni-leipzig.de

___
Quantum ESPRESSO is supported by MaX (www.max-centre.eu/quantum-espresso)
users mailing list users@lists.quantum-espresso.org
https://lists.quantum-espresso.org/mailman/listinfo/users

Re: [QE-users] Expectation values of total angular momentum in SOC case

2020-01-24 Thread Thomas Brumme

Dear Paolo, (or Dear all,)

could this be a problem of the specific pseudopotential?

I tested with US and PAW in the Hf system and didn't find any 
inconsistency. Yet, I even get in the monolayer WSe2 the wrong spin 
state at the conduction-band minimum. It should be - due to symmetry, at 
least what I remember from a lot of publications - either spin up or 
spin down. I get for the expectation value of Sz 0.45411. Since both Sx 
and Sy are 0 this can only mean that I have something which is a 
combination of spin up and down. Or the wave function is not properly 
normalized, but this should be detected by PW, right?


I ask for the pseudo since I remember that I had problems with 
relaxations when using the W.rel-...paw... of the pslibrary. Using a 
slightly different pseudo (which I created some time ago with Lorenzo 
when I was in Paris) the relaxations work and also the electronic 
structure and everything was fine... Could it be, that only the spin 
part is affected?


Or is it just wrong that everyone says that in WSe2 the conduction-band 
minimum at the K point is spin up? (question to the audience)


Regards

Thomas


On 1/23/20 5:10 PM, Paolo Giannozzi wrote:
On Thu, Jan 23, 2020 at 5:00 PM Thomas Brumme 
mailto:thomas.bru...@uni-leipzig.de>> 
wrote:


Or is there something missing in the PAW case which is not
important for US-PP?


if you regularly get 1/2 for norm-conserving / ultrasoft and  less 
than that with PAW, it is very likely that there is something missing 
in the PAW case


Paolo
--
Paolo Giannozzi, Dip. Scienze Matematiche Informatiche e Fisiche,
Univ. Udine, via delle Scienze 208, 33100 Udine, Italy
Phone +39-0432-558216, fax +39-0432-558222


___
Quantum ESPRESSO is supported by MaX (www.max-centre.eu/quantum-espresso)
users mailing list users@lists.quantum-espresso.org
https://lists.quantum-espresso.org/mailman/listinfo/users


--
Dr. rer. nat. Thomas Brumme
Wilhelm-Ostwald-Institute for Physical and Theoretical Chemistry
Leipzig University
Phillipp-Rosenthal-Strasse 31
04103 Leipzig

Tel:  +49 (0)341 97 36456

email: thomas.bru...@uni-leipzig.de

___
Quantum ESPRESSO is supported by MaX (www.max-centre.eu/quantum-espresso)
users mailing list users@lists.quantum-espresso.org
https://lists.quantum-espresso.org/mailman/listinfo/users

Re: [QE-users] Expectation values of total angular momentum in SOC case

2020-01-23 Thread Thomas Brumme
No. I found calculations on the Hf system I did some time ago with PAW 
and US. Both cases give the same value of Sz at the K point - 0.498. OK, 
it is not exactly 0.5 but it is not 0.47.


So, good news, apparently it is not a problem of using PAW. So, it is a 
system-specific feature!?


But I don't understand why...

Thomas

On 1/23/20 5:36 PM, Thomas Brumme wrote:

Dear Paolo,

I will test this on one system with US and PAW and then I will see if 
this might be a problem...
The 1/2 I got for the other system in which I used US - so the US and 
PAW cases I have are from different systems...


Thanks

Thomas

On 1/23/20 5:10 PM, Paolo Giannozzi wrote:
On Thu, Jan 23, 2020 at 5:00 PM Thomas Brumme 
mailto:thomas.bru...@uni-leipzig.de>> 
wrote:


Or is there something missing in the PAW case which is not
important for US-PP?


if you regularly get 1/2 for norm-conserving / ultrasoft and  less 
than that with PAW, it is very likely that there is something missing 
in the PAW case


Paolo
--
Paolo Giannozzi, Dip. Scienze Matematiche Informatiche e Fisiche,
Univ. Udine, via delle Scienze 208, 33100 Udine, Italy
Phone +39-0432-558216, fax +39-0432-558222


___
Quantum ESPRESSO is supported by MaX (www.max-centre.eu/quantum-espresso)
users mailing listus...@lists.quantum-espresso.org
https://lists.quantum-espresso.org/mailman/listinfo/users


--
Dr. rer. nat. Thomas Brumme
Wilhelm-Ostwald-Institute for Physical and Theoretical Chemistry
Leipzig University
Phillipp-Rosenthal-Strasse 31
04103 Leipzig

Tel:  +49 (0)341 97 36456

email:thomas.bru...@uni-leipzig.de

___
Quantum ESPRESSO is supported by MaX (www.max-centre.eu/quantum-espresso)
users mailing list users@lists.quantum-espresso.org
https://lists.quantum-espresso.org/mailman/listinfo/users


--
Dr. rer. nat. Thomas Brumme
Wilhelm-Ostwald-Institute for Physical and Theoretical Chemistry
Leipzig University
Phillipp-Rosenthal-Strasse 31
04103 Leipzig

Tel:  +49 (0)341 97 36456

email: thomas.bru...@uni-leipzig.de

___
Quantum ESPRESSO is supported by MaX (www.max-centre.eu/quantum-espresso)
users mailing list users@lists.quantum-espresso.org
https://lists.quantum-espresso.org/mailman/listinfo/users

Re: [QE-users] Expectation values of total angular momentum in SOC case

2020-01-23 Thread Thomas Brumme

Dear Paolo,

I will test this on one system with US and PAW and then I will see if 
this might be a problem...
The 1/2 I got for the other system in which I used US - so the US and 
PAW cases I have are from different systems...


Thanks

Thomas

On 1/23/20 5:10 PM, Paolo Giannozzi wrote:
On Thu, Jan 23, 2020 at 5:00 PM Thomas Brumme 
mailto:thomas.bru...@uni-leipzig.de>> 
wrote:


Or is there something missing in the PAW case which is not
important for US-PP?


if you regularly get 1/2 for norm-conserving / ultrasoft and  less 
than that with PAW, it is very likely that there is something missing 
in the PAW case


Paolo
--
Paolo Giannozzi, Dip. Scienze Matematiche Informatiche e Fisiche,
Univ. Udine, via delle Scienze 208, 33100 Udine, Italy
Phone +39-0432-558216, fax +39-0432-558222


___
Quantum ESPRESSO is supported by MaX (www.max-centre.eu/quantum-espresso)
users mailing list users@lists.quantum-espresso.org
https://lists.quantum-espresso.org/mailman/listinfo/users


--
Dr. rer. nat. Thomas Brumme
Wilhelm-Ostwald-Institute for Physical and Theoretical Chemistry
Leipzig University
Phillipp-Rosenthal-Strasse 31
04103 Leipzig

Tel:  +49 (0)341 97 36456

email: thomas.bru...@uni-leipzig.de

___
Quantum ESPRESSO is supported by MaX (www.max-centre.eu/quantum-espresso)
users mailing list users@lists.quantum-espresso.org
https://lists.quantum-espresso.org/mailman/listinfo/users

Re: [QE-users] Expectation values of total angular momentum in SOC case

2020-01-23 Thread Thomas Brumme

Hey Lorenzo,

There are definitely regions in the BZ where the spin is restricted to 
certain directions but away from these high-symmetry points (or lines) 
it is pointing in different directions. For example at the K point it 
needs to point in the z direction. And I see this in the results - 
expectation values of Sx and Sy are zero and Sz is 0.47. But 0.47 for an 
s=1/2 particle? This is what I don't understand but which might be due 
to SOC (I thought)... But then, why it is like this in a TMD 
heterostructure but not in HfNCl (same BZ) where I see 0.499?


Thomas

On 1/23/20 4:25 PM, Lorenzo Paulatto wrote:

    Yet, the DFT expectation values for Sx, Sy,
    Sz do not result in a spin of 1/2 (for the TMD heterostructure) 
but a little
    bit less, 0.468, and this value is too different from 1/2 to say 
it is
    numerical noise. 


I'm not an expert in this subject, but I see it like this: you want to 
isolate a quasi-particle band, which is more or less equivalent that 
getting its Wannier function.  When doing a non-collinear calculation, 
Wannier functions are spinors.


It could be possible, for your case, to compute this Wannier function 
and see if its spin angle turns out to be constant, e.g. because of 
symmetry reason or dimensionality. (See Wannier90 example 17 or 18)


cheers





--
Dr. rer. nat. Thomas Brumme
Wilhelm-Ostwald-Institute for Physical and Theoretical Chemistry
Leipzig University
Phillipp-Rosenthal-Strasse 31
04103 Leipzig

Tel:  +49 (0)341 97 36456

email: thomas.bru...@uni-leipzig.de

___
Quantum ESPRESSO is supported by MaX (www.max-centre.eu/quantum-espresso)
users mailing list users@lists.quantum-espresso.org
https://lists.quantum-espresso.org/mailman/listinfo/users

Re: [QE-users] Expectation values of total angular momentum in SOC case

2020-01-23 Thread Thomas Brumme

Dear Guido,

no problem! Input is welcome and I also wanted to know if my reasoning 
is wrong and in this case more people can help better - it's like the 
"Ask the Audience" joker in "Who wants to be a millionaire" :D


The spin expectation values can be calculated for each k point like in a 
text book. The expectation value of the Pauli matrices with the spinor 
wave functions.
And I use this to get a Hamiltonian consisting of the Pauli matrices and 
with k-dependent prefactors. But the length of the spin vector (i.e., 
the vector with the 3 expectation values Sx, Sy, Sz) is not 1/2 but 
0.468 in the TMD heterostructure at and close to the conduction-band 
minimum. I expected 1/2 like in the first system I calculated.


That's why I was thinking if only the length of the spin vector for the 
whole band is 1/2... How to do the averaging in this case? Or is it due 
to SOC and only the total angular momentum makes sense? How to define 
this for a band and not an atom?


Concerning the actual calculation: this is done in 
PP/src/compute_sigma_avg.f90 and seems to be correct... Or is there 
something missing in the PAW case which is not important for US-PP?


Thomas

On 1/23/20 3:37 PM, Guido Menichetti wrote:

Dear Thomas,

sorry if I intrude on the conversation.

How do you evaluate the DFT expectation values for Sx, Sy, Sz from QE?
Could the discrepancy arise from the way it is calculated?

Regards,
G.

Il giorno gio 23 gen 2020 alle ore 15:22 Thomas Brumme 
mailto:thomas.bru...@uni-leipzig.de>> 
ha scritto:


Hey Lorenzo,

the "problem" is actually more complex and it is not a real
problem but
something I thought about and maybe I'm just missing something.

I calculate the band structure for some 2D systems including SOC and
want to fit a model to the spin state such that I can extract SOC
parameters. First order would be Rashba-type SOC but 2nd and 3rd
order
is something else which also depends on the local symmetry. For one
system this works without problems. Then I wanted to transfer the
ideas
and my "code" to a heterobilayer of TMDs and there it sort of
works but
there is one problem:

In order to fit the model, I first fit a generic Pauli Hamiltonian
(to
which the model is fitted) - in this way the code can be easily
adapted
to other local symmetries because only the 2nd stage needs to be
changed. Anyways, in the Pauli Hamiltonian I assume that the spin
is 1/2
- an electron or hole. Yet, the DFT expectation values for Sx, Sy,
Sz do
not result in a spin of 1/2 (for the TMD heterostructure) but a
little
bit less, 0.468, and this value is too different from 1/2 to say
it is
numerical noise. And then I thought that, well, spin is not a good
quantum number and I would need the total angular momentum. Or do
I need
to calculate the spin expectation values for the whole BZ and then a
single band would add up to 1/2? Is it OK to just, lets say, use
S^2 =
0.468 instead of 1/2 and say that this is due to SOC?

Regards

Thomas

On 1/23/20 12:36 PM, Lorenzo Paulatto wrote:
> Hello Thomas,
> if I remember correctly, the fact that the spin does not commute
with
> the Hamiltonian mean that the spin can be:
> 1. k-point dependent, you do not have spin-up and spin-down bands
> which can be separated
> 2. aligned along any direction, instead of just Z
>
> I think, but am not 100% sure, that if J is a good quantum
number for
> isolated atoms with mean-field interacting electrons, this is
not true
> for bulk crystals (what is L in the bulk?)
>
> With the options of bands.x setting lsigma=.true. you can plot the
> spin projected over x y and z and do some kind of color-codes
plot of
    > the bands
>
> cheers
>
>
>
> On 22/01/2020 16:57, Thomas Brumme wrote:
>> Dear all,
>>
>> I tried to find something in the archive but was not successful.
>>
>> In noncollinear calculations I can plot the spin expectation
values
>> using bands.x.
>> Those are calculated using the standard Pauli matrices. Yet,
spin is
>> not a good
>> quantum number anymore once I have SOC. Thus, I actually have
to look
>> at the
>> total angular momentum, J. Is it possible to get the expectation
    >> values of J?
>> Does it make sense at all to think about implementing it?
>>
>> Regards
>>
>> Thomas
>>
>

-- 
Dr. rer. nat. Thomas Brumme

Wilhelm-Ostwald-Institute for Physical and Theoretical Chemistry
Leipzig University
Phillipp-Rosenthal-Strasse 31
04103 Leipzig

Tel:  +49 (0)341 97 36456

Re: [QE-users] Expectation values of total angular momentum in SOC case

2020-01-23 Thread Thomas Brumme

Hey Lorenzo,

the "problem" is actually more complex and it is not a real problem but 
something I thought about and maybe I'm just missing something.


I calculate the band structure for some 2D systems including SOC and 
want to fit a model to the spin state such that I can extract SOC 
parameters. First order would be Rashba-type SOC but 2nd and 3rd order 
is something else which also depends on the local symmetry. For one 
system this works without problems. Then I wanted to transfer the ideas 
and my "code" to a heterobilayer of TMDs and there it sort of works but 
there is one problem:


In order to fit the model, I first fit a generic Pauli Hamiltonian (to 
which the model is fitted) - in this way the code can be easily adapted 
to other local symmetries because only the 2nd stage needs to be 
changed. Anyways, in the Pauli Hamiltonian I assume that the spin is 1/2 
- an electron or hole. Yet, the DFT expectation values for Sx, Sy, Sz do 
not result in a spin of 1/2 (for the TMD heterostructure) but a little 
bit less, 0.468, and this value is too different from 1/2 to say it is 
numerical noise. And then I thought that, well, spin is not a good 
quantum number and I would need the total angular momentum. Or do I need 
to calculate the spin expectation values for the whole BZ and then a 
single band would add up to 1/2? Is it OK to just, lets say, use S^2 = 
0.468 instead of 1/2 and say that this is due to SOC?


Regards

Thomas

On 1/23/20 12:36 PM, Lorenzo Paulatto wrote:

Hello Thomas,
if I remember correctly, the fact that the spin does not commute with 
the Hamiltonian mean that the spin can be:
1. k-point dependent, you do not have spin-up and spin-down bands 
which can be separated

2. aligned along any direction, instead of just Z

I think, but am not 100% sure, that if J is a good quantum number for 
isolated atoms with mean-field interacting electrons, this is not true 
for bulk crystals (what is L in the bulk?)


With the options of bands.x setting lsigma=.true. you can plot the 
spin projected over x y and z and do some kind of color-codes plot of 
the bands


cheers



On 22/01/2020 16:57, Thomas Brumme wrote:

Dear all,

I tried to find something in the archive but was not successful.

In noncollinear calculations I can plot the spin expectation values 
using bands.x.
Those are calculated using the standard Pauli matrices. Yet, spin is 
not a good
quantum number anymore once I have SOC. Thus, I actually have to look 
at the
total angular momentum, J. Is it possible to get the expectation 
values of J?

Does it make sense at all to think about implementing it?

Regards

Thomas





--
Dr. rer. nat. Thomas Brumme
Wilhelm-Ostwald-Institute for Physical and Theoretical Chemistry
Leipzig University
Phillipp-Rosenthal-Strasse 31
04103 Leipzig

Tel:  +49 (0)341 97 36456

email: thomas.bru...@uni-leipzig.de

___
Quantum ESPRESSO is supported by MaX (www.max-centre.eu/quantum-espresso)
users mailing list users@lists.quantum-espresso.org
https://lists.quantum-espresso.org/mailman/listinfo/users


[QE-users] Expectation values of total angular momentum in SOC case

2020-01-22 Thread Thomas Brumme

Dear all,

I tried to find something in the archive but was not successful.

In noncollinear calculations I can plot the spin expectation values 
using bands.x.
Those are calculated using the standard Pauli matrices. Yet, spin is not 
a good

quantum number anymore once I have SOC. Thus, I actually have to look at the
total angular momentum, J. Is it possible to get the expectation values 
of J?

Does it make sense at all to think about implementing it?

Regards

Thomas

--
Dr. rer. nat. Thomas Brumme
Wilhelm-Ostwald-Institute for Physical and Theoretical Chemistry
Leipzig University
Phillipp-Rosenthal-Strasse 31
04103 Leipzig

Tel:  +49 (0)341 97 36456

email: thomas.bru...@uni-leipzig.de

___
Quantum ESPRESSO is supported by MaX (www.max-centre.eu/quantum-espresso)
users mailing list users@lists.quantum-espresso.org
https://lists.quantum-espresso.org/mailman/listinfo/users


Re: [QE-users] How to calculate the dI/dV (STS)

2020-01-07 Thread Thomas Brumme

Dear Chris,

one more thing: all these options are just a first approximation. 
Looking at the

STMpw tool you cited you'll see that there are more advanced option.
One step further would be including the STM tip, next could be the wave 
function

matching ("Bardeen" in that tool), next would be calculating the zero bias
conductance via the Landauer-Buttiker formula, e.g., with
PWCOND: http://iramis.cea.fr/Pisp/alexander.smogunov/PWCOND/pwcond.html
 or WanT: http://www.wannier-transport.org/wiki/index.php/Main_Page
and, last but not least, calculating the conductance including a real 
bias voltage.
The last option (I think) is not available in QE - there was a code 
called Smeagol
which is interfaced with Siesta 
(https://www.tcd.ie/Physics/Smeagol/index.html)

but I don't know if this still works or if it is now included in Transiesta.

Cheers

Thomas

On 1/7/20 3:03 AM, Christoph Wolf wrote:

Dear Thomas,

thank you very much for your detailed explanation, I will try and see 
how far I can get with plot_num 3,7 and 10. A lot more options here 
than expected ;)


Thanks again and with best regards,
Chris

On Sun, 5 Jan 2020 at 06:07, Dr. Thomas Brumme 
mailto:thomas.bru...@uni-leipzig.de>> 
wrote:


Dear Chris,

within the Tersoff-Hamann approximation the STM image is proportional
to the integral of the local density of states integrated from the
Fermi energy till the bias voltage:

https://journals.aps.org/prb/abstract/10.1103/PhysRevB.31.805

As far as I remember, the method implemented in PWscf uses this
approximation. Accordingly, the STS - which is just dI/dV - should be
proportional to the local density of states at the bias voltage.
Two things to remember here:

- STM tips can have apex atoms which have d orbitals and then Tersoff
Hamann breaks down
- unoccupied states are - from my experience - hardly ever at the
correct bias compared to experiments. This is due to the band-gap
problem but also the curvature (effective mass) can be wrong. Or the
Fermi energy is at a different position in the experiments. Thus,
depending on the exchange-correlation functional, agreement for states
in the unoccupied regime could be false positives...

So, for STM pictures, use the option 5 in pp.x. For STS, either plot
the closest eigenfunction in real space (option 7) or directly use
option 3 to plot the local density of states. OR integrate the LDOS
over a certain region at the specified bias - "simulating" an
experimental broadening... Option 10.

Hope that helps! Kind regards

Thomas


P.S.: Numerical derivative of the STM pictures should also work and
I also used this about 10 years ago during my Diploma :)


Zitat von Christoph Wolf :

> Dear all,
>
> I was wondering if there is a tool that is able to calculate the
dI/dV for
> output from PWSCF? I guess the way it is currently implemented
would be to
> calculate a set of STM images for different biases and then take the
> numerical derivative but for larger systems this is actually
really time
> consuming and since we have the wave functions at the end of a
calculation
> there might be a better way to do this. There is for example
this code:
> https://github.com/qphensurf/STMpw which unfortunately is
currently not
> interfaced with PWSCF.
>
> Any help is much appreciated!
>
> Happy new year everyone!
>
> Chris
>
> --
> Postdoctoral Researcher
> Center for Quantum Nanoscience, Institute for Basic Science
    > Ewha Womans University, Seoul, South Korea


--
Dr. rer. nat. Thomas Brumme
Wilhelm-Ostwald-Institute for Physical and Theoretical Chemistry
Leipzig University
Phillipp-Rosenthal-Strasse 31
04103 Leipzig
Tel: +49 (0)341 97 36456
email: thomas.bru...@uni-leipzig.de
<mailto:thomas.bru...@uni-leipzig.de>



--
Postdoctoral Researcher
Center for Quantum Nanoscience, Institute for Basic Science
Ewha Womans University, Seoul, South Korea


--
Dr. rer. nat. Thomas Brumme
Wilhelm-Ostwald-Institute for Physical and Theoretical Chemistry
Leipzig University
Phillipp-Rosenthal-Strasse 31
04103 Leipzig

Tel:  +49 (0)341 97 36456

email: thomas.bru...@uni-leipzig.de

___
Quantum ESPRESSO is supported by MaX (www.max-centre.eu/quantum-espresso)
users mailing list users@lists.quantum-espresso.org
https://lists.quantum-espresso.org/mailman/listinfo/users

Re: [QE-users] How to calculate the dI/dV (STS)

2020-01-04 Thread Dr. Thomas Brumme

Dear Chris,

within the Tersoff-Hamann approximation the STM image is proportional
to the integral of the local density of states integrated from the
Fermi energy till the bias voltage:

https://journals.aps.org/prb/abstract/10.1103/PhysRevB.31.805

As far as I remember, the method implemented in PWscf uses this
approximation. Accordingly, the STS - which is just dI/dV - should be
proportional to the local density of states at the bias voltage.
Two things to remember here:

- STM tips can have apex atoms which have d orbitals and then Tersoff
Hamann breaks down
- unoccupied states are - from my experience - hardly ever at the
correct bias compared to experiments. This is due to the band-gap
problem but also the curvature (effective mass) can be wrong. Or the
Fermi energy is at a different position in the experiments. Thus,
depending on the exchange-correlation functional, agreement for states
in the unoccupied regime could be false positives...

So, for STM pictures, use the option 5 in pp.x. For STS, either plot
the closest eigenfunction in real space (option 7) or directly use
option 3 to plot the local density of states. OR integrate the LDOS
over a certain region at the specified bias - "simulating" an
experimental broadening... Option 10.

Hope that helps! Kind regards

Thomas


P.S.: Numerical derivative of the STM pictures should also work and
I also used this about 10 years ago during my Diploma :)


Zitat von Christoph Wolf :


Dear all,

I was wondering if there is a tool that is able to calculate the dI/dV for
output from PWSCF? I guess the way it is currently implemented would be to
calculate a set of STM images for different biases and then take the
numerical derivative but for larger systems this is actually really time
consuming and since we have the wave functions at the end of a calculation
there might be a better way to do this. There is for example this code:
https://github.com/qphensurf/STMpw which unfortunately is currently not
interfaced with PWSCF.

Any help is much appreciated!

Happy new year everyone!

Chris

--
Postdoctoral Researcher
Center for Quantum Nanoscience, Institute for Basic Science
Ewha Womans University, Seoul, South Korea



--
Dr. rer. nat. Thomas Brumme
Wilhelm-Ostwald-Institute for Physical and Theoretical Chemistry
Leipzig University
Phillipp-Rosenthal-Strasse 31
04103 Leipzig
Tel: +49 (0)341 97 36456
email: thomas.bru...@uni-leipzig.de

___
Quantum ESPRESSO is supported by MaX (www.max-centre.eu/quantum-espresso)
users mailing list users@lists.quantum-espresso.org
https://lists.quantum-espresso.org/mailman/listinfo/users


Re: [QE-users] Problem with SCF including spin-orbit coupling

2019-12-21 Thread Dr. Thomas Brumme

Dear Pascal,

You can try the older pseudo and see whether it works - in the case of  
tungsten I created my own pseudo which is just slightly different from  
the one of the PS library. Yet, I don't know anymore what I changed  
and I'm already on Christmas vacation. It had to do with using a  
certain type of function (bessel or something like this?)... I could  
help you with that in January ;)


Regards

Thomas

Zitat von pboulet :


Dear Marko and Thomas,

I ran the tests you suggested, the problem comes from the Rh  
relativistic pseudopotential. The others are okay, even with the  
occupations fixed (there is a large gap anyway).


I may try the older psp of Rh available in the pslibrary, or do you  
have other suggestion(s)?


Thank you
Best regards
Pascal



Pascal Boulet
—
Professor in computational materials - DEPARTMENT OF CHEMISTRY
Aix-Marseille University - Avenue Escadrille Normandie Niemen -  
F-13013 Marseille - FRANCE

Tél: +33(0)4 13 55 18 10 - Fax : +33(0)4 13 55 18 50
Email : pascal.bou...@univ-amu.fr <mailto:pascal.bou...@univ-amu.fr>







Le 19 déc. 2019 à 20:53, Mladenovic Marko  
 a écrit :


I would also try non-fixed occupancies and different  
pseudopotentials (in line to what Thomas proposed).


Marko

From: users <mailto:users-boun...@lists.quantum-espresso.org>> on behalf of Dr.  
Thomas Brumme <mailto:thomas.bru...@uni-leipzig.de>>

Sent: Thursday, December 19, 2019 7:03:25 PM
To: pascal.bou...@univ-amu.fr <mailto:pascal.bou...@univ-amu.fr>;  
users@lists.quantum-espresso.org  
<mailto:users@lists.quantum-espresso.org>

Subject: Re: [QE-users] Problem with SCF including spin-orbit coupling

Dear Pascal,

If the non-SOC calculation converged, try to replace sthe pseudos one
by one with the relativistic ones - I know for example that the
relativistic tungsten potential of the PS library had (has?) some
problems. You could have the same problem. If it is a specific pseudo
we can try to change/improve it ;)

Thomas

Zitat von pboulet <mailto:pascal.bou...@univ-amu.fr>>:



Hello Marko,

Thank you for your prompt response.

I copy-pasted the wrong input file. The right one is a SCF one (the
nscf should be subsequent to scf):

 calculation   ='scf',
 nstep = 200,
 etot_conv_thr = 1.d-7,
!  forc_conv_thr = 1.d-4,
 wfcdir= './WFC' ,
!  tstress   = .true.,
 prefix= 'STe_scf_LS',
 pseudo_dir= '/scratch/cnt0022/pmc6881/paboulet/pseudo/',
 verbosity = 'high',
 restart_mode  = ‘from_scratch',
 wf_collect= .true.,
 disk_io   = 'high',

The rest is the same as in my previous email, except for the
k-points: 12x12x4 1 1 1, as in the non-SO optimisation.

Pascal



Le 19 déc. 2019 à 17:42, Mladenovic Marko
mailto:marko.mladeno...@epfl.ch>> a écrit :

Hello Pascal,

it seems that you are performing a non-scf calculation (calculation
 ='nscf') and you are restarting an other calculation
(restart_mode  = 'restart'). Is this what you want?

Best,
Marko
From: users <mailto:users-boun...@lists.quantum-espresso.org>> on behalf of

pboulet mailto:pascal.bou...@univ-amu.fr>>
Sent: Thursday, December 19, 2019 5:03:40 PM
To: Quantum Espresso users Forum
Subject: [QE-users] Problem with SCF including spin-orbit coupling

Dear All,

I have some troubles with a SCF calculation using SOC (QE 6.1).
I have optimised a structure with non-SOC pseudopotentials and now
I want to perform a SCF calculation by including SOC to get the
electronic band structure. The problem is that the SCF diverges
right away as you can see here:



Self-consistent Calculation

iteration #  1 ecut=35.00 Ry beta=0.10
Davidson diagonalization with overlap
ethr =  1.00E-05,  avg # of iterations = 14.2

negative rho (up, down):  8.197E+02 0.000E+00

total cpu time spent up to now is 2293.7 secs

total energy  =  -16093.90449193 Ry
Harris-Foulkes estimate   =  -18277.15379034 Ry
estimated scf accuracy< 5077678.52441197 Ry

iteration #  2 ecut=35.00 Ry beta=0.10
Davidson diagonalization with overlap
ethr =  1.00E-02,  avg # of iterations =  5.0

negative rho (up, down):  2.677E+03 0.000E+00

total cpu time spent up to now is 3119.6 secs

total energy  =   79024.56634461 Ry
Harris-Foulkes estimate   =  -17101.52000727 Ry
estimated scf accuracy< 4036754.22296032 Ry

iteration #  3 ecut=35.00 Ry beta=0.10
Davidson diagonalization with overlap
ethr =  1.00E-02,  avg # of iterations =  6.0

negative rho (up, down):  2.815E+03 0.000E+00

total cpu time spent up to now is 4658.4 secs

total energy  =  132135.46722241 Ry
Harris-Foulkes estimate   = -142275.32127673 Ry
estimated scf accuracy< 6116470.38137350 Ry
<<<<<

I am using PAW pseudopotentials (Ba, Rh, Ge and S) from the PSL
libra

Re: [QE-users] Problem with SCF including spin-orbit coupling

2019-12-19 Thread Dr. Thomas Brumme

Dear Pascal,

If the non-SOC calculation converged, try to replace the pseudos one  
by one with the relativistic ones - I know for example that the  
relativistic tungsten potential of the PS library had (has?) some  
problems. You could have the same problem. If it is a specific pseudo  
we can try to change/improve it ;)


Thomas

Zitat von pboulet :


Hello Marko,

Thank you for your prompt response.

I copy-pasted the wrong input file. The right one is a SCF one (the  
nscf should be subsequent to scf):


  calculation   ='scf',
  nstep = 200,
  etot_conv_thr = 1.d-7,
!  forc_conv_thr = 1.d-4,
  wfcdir= './WFC' ,
!  tstress   = .true.,
  prefix= 'STe_scf_LS',
  pseudo_dir= '/scratch/cnt0022/pmc6881/paboulet/pseudo/',
  verbosity = 'high',
  restart_mode  = ‘from_scratch',
  wf_collect= .true.,
  disk_io   = 'high',

The rest is the same as in my previous email, except for the  
k-points: 12x12x4 1 1 1, as in the non-SO optimisation.


Pascal


Le 19 déc. 2019 à 17:42, Mladenovic Marko  
 a écrit :


Hello Pascal,

it seems that you are performing a non-scf calculation (calculation  
  ='nscf') and you are restarting an other calculation  
(restart_mode  = 'restart'). Is this what you want?


Best,
Marko
From: users  on behalf of  
pboulet 

Sent: Thursday, December 19, 2019 5:03:40 PM
To: Quantum Espresso users Forum
Subject: [QE-users] Problem with SCF including spin-orbit coupling

Dear All,

I have some troubles with a SCF calculation using SOC (QE 6.1).
I have optimised a structure with non-SOC pseudopotentials and now  
I want to perform a SCF calculation by including SOC to get the  
electronic band structure. The problem is that the SCF diverges  
right away as you can see here:

>>>>
Self-consistent Calculation

 iteration #  1 ecut=35.00 Ry beta=0.10
 Davidson diagonalization with overlap
 ethr =  1.00E-05,  avg # of iterations = 14.2

 negative rho (up, down):  8.197E+02 0.000E+00

 total cpu time spent up to now is 2293.7 secs

 total energy  =  -16093.90449193 Ry
 Harris-Foulkes estimate   =  -18277.15379034 Ry
 estimated scf accuracy< 5077678.52441197 Ry

 iteration #  2 ecut=35.00 Ry beta=0.10
 Davidson diagonalization with overlap
 ethr =  1.00E-02,  avg # of iterations =  5.0

 negative rho (up, down):  2.677E+03 0.000E+00

 total cpu time spent up to now is 3119.6 secs

 total energy  =   79024.56634461 Ry
 Harris-Foulkes estimate   =  -17101.52000727 Ry
 estimated scf accuracy< 4036754.22296032 Ry

 iteration #  3 ecut=35.00 Ry beta=0.10
 Davidson diagonalization with overlap
 ethr =  1.00E-02,  avg # of iterations =  6.0

 negative rho (up, down):  2.815E+03 0.000E+00

 total cpu time spent up to now is 4658.4 secs

 total energy  =  132135.46722241 Ry
 Harris-Foulkes estimate   = -142275.32127673 Ry
 estimated scf accuracy< 6116470.38137350 Ry
<<<<<

I am using PAW pseudopotentials (Ba, Rh, Ge and S) from the PSL  
library.  The cg diagonalisation does not solve the problem.


Here is a piece of the input file:

  calculation   ='nscf',
  nstep = 300,
  etot_conv_thr = 1.d-7,
  forc_conv_thr = 1.d-4,
  wfcdir= './WFC' ,
  prefix= 'STe_nscf_LS',
  pseudo_dir= '/scratch/cnt0022/pmc6881/paboulet/pseudo/',
  verbosity = 'high',
  restart_mode  = 'restart',
  wf_collect= .true.,
  disk_io   = 'high',
/

!  celldm(1)   = 1.0,
  nat = 52,
  ntyp= 5,
  ibrav   = 0,
  ecutwfc = 35.d0,
  ecutrho = 350.d0,
  occupations = 'fixed',
  nbnd= 630,
  lspinorb= .true.,
  noncolin= .true.,
  nr1 = 72, nr2 = 72, nr3 = 360,
/

  electron_maxstep = 200,
  conv_thr = 1.d-10,
  mixing_beta  = 0.2d0,
  diagonalization  = 'david',
/

K_POINTS automatic
16 16 8 0 0 0

ATOMIC_SPECIES
Ba 137.3270   Ba.rel-pbe-spn-kjpaw_psl.1.0.0.UPF,
Rh 102.9055   Rh.rel-pbe-spn-kjpaw_psl.1.0.0.UPF,
Ge  72.6400   Ge.rel-pbe-dn-kjpaw_psl.1.0.0.UPF,
S   32.0650   S.rel-pbe-nl-kjpaw_psl.1.0.0.UPF,
Te 127.6000   Te.rel-pbe-dn-kjpaw_psl.0.2.2.UPF

etc…


Do you have a special recipe for this type of calculation?

Thank you,
Best regards

Pascal Boulet
—
Professor in computational materials - DEPARTMENT OF CHEMISTRY
Aix-Marseille University - Avenue Escadrille Normandie Niemen -  
F-13013 Marseille - FRANCE

Tél: +33(0)4 13 55 18 10 - Fax : +33(0)4 13 55 18 50
Email : pascal.bou...@univ-amu.fr <mailto:pascal.bou...@univ-amu.fr>







___
Quantum ESPRESSO is supported by MaX (www.max-centre.eu/quantum-espresso)
users mailing list users@lists.quantum-espresso.org
https://lists.quantum-espresso.org/mailman/listinfo/users



--
Dr. rer. nat. Thomas Brumme
Wilhelm-Ostwald-Institute for Physical and Theoretical Chemistry
Lei

Re: [QE-users] band convergece problem in charged system

2019-10-22 Thread Thomas Brumme

Dear Yang,

I have no clue how the Martyna-Tuckerman correction works but usually one
has problems with systems having such a large total charge since the total
cell needs to be neutral. Some terms are in fact calculated for a charge 
neutral
system resulting in a total potential which has a curvature as if there 
would be

a homogeneous background charge. This constant curvature leads to a
potential energy minimum in the vacuum space between the repeated cells.
If you increase the cell this minimum goes down in energy and, at some 
point,

crosses the Fermi energy and gets occupied.

Long story short:
I have no clue but are you sure that Martyna-Tuckerman correction works
correctly for charged systems?
Is the LUMO really a state of the molecule and not from a state in the 
vacuum?


You could check if everything works as intended for a neutral system...

Regards

Thomas


On 10/2/19 1:46 PM, Leo Yang wrote:


Dear QE users:

I try to use QE-6.4.1 to simulate as single molecule. The system 
is built in large super cell, and /total_charge=-4/  is added to input 
file leading to charged system. When I do the convergence test in term 
of the super cell size (20 Angs - 30 Angs ), the variation for total 
energy is small: -1038.6784 Ry for 20 Angs and 1038.7388 Ry for 30 
Angs. But the band energy and Fermi energy (5.23 ev to 3.08 ev) shift 
visibly. The problem is that the band gap between band 104 (occupied) 
and 105 (unoccupied) become smaller with the increase of size (about 
0.81 ev change to 0.11 ev). It seems the band gap is gradually 
disappear, which confuses me so much. The input file is attached with 
this email. It will be very appreciated if you can solve this for me. 
Thank you.



Best wishes

Yang Zhou


--

Yang Zhou

PhD student

University of Leeds

United Kingdom


___
Quantum ESPRESSO is supported by MaX (www.max-centre.eu/quantum-espresso)
users mailing list users@lists.quantum-espresso.org
https://lists.quantum-espresso.org/mailman/listinfo/users


--
Dr. rer. nat. Thomas Brumme
Wilhelm-Ostwald-Institute for Physical and Theoretical Chemistry
Leipzig University
Phillipp-Rosenthal-Strasse 31
04103 Leipzig

Tel:  +49 (0)341 97 36456

email: thomas.bru...@uni-leipzig.de

___
Quantum ESPRESSO is supported by MaX (www.max-centre.eu/quantum-espresso)
users mailing list users@lists.quantum-espresso.org
https://lists.quantum-espresso.org/mailman/listinfo/users

Re: [QE-users] Fermi Surface of 2D material

2019-09-28 Thread Dr. Thomas Brumme

Dear Hussain,

Remember that the Fermi surface of a 3D bulk material is a 2D surface
in 3D space (kx, ky, kz). Thus, the Fermi surface of a 2D system is
an 1D line/curve on a 2D plane (kx, ky). The bxsf file format only
works nicely for 3D systems. You can however trick a little bit.
Instead of specifying a grid like, e.g., "32 32 1" you specify it by
"32 32 2" and then you can use XCrysDen with the bxsf file format.
It's also kind of a test if you modeled your system correctly - you
should see no dispersion in the 3rd direction.

Regards

Thomas


Zitat von Hussain Ali :


Dear QE users,
I am new to QE. I have interest to plot the Fermi surface of 2D materials.
I have tried to reproduce the bulk Fermi surfaces of some materials, e.g.,
Ni following the example of QE.
Can someone guide me how to plot the Fermi surface of 2D material. I tried
but the .bxsf file shows no Fermi surface.
My 2D material is metal.

Hussain
MS student
QAU, Islamabad Pakistan.


--
Dr. rer. nat. Thomas Brumme
Wilhelm-Ostwald-Institute for Physical and Theoretical Chemistry
Leipzig University
Phillipp-Rosenthal-Strasse 31
04103 Leipzig

Tel:  +49 (0)341 97 36456

email: thomas.bru...@uni-leipzig.de

___
Quantum ESPRESSO is supported by MaX (www.max-centre.eu/quantum-espresso)
users mailing list users@lists.quantum-espresso.org
https://lists.quantum-espresso.org/mailman/listinfo/users


Re: [QE-users] bad convergence in field-effect configuration calculations

2019-09-16 Thread Dr. Thomas Brumme

Dear Bin,

I have no clue if it could be done but it is definitely not  
implemented. There are problems:
Firstly, changing the lattice also changes the doping charge per area  
and thus the dipole. Yes, this is calculated self-consistently anyways  
but in the total energy there're parts related to the dipole and the  
gate interacting with themselves - how to take them properly into  
account? One would need to go through the equations related to the  
gate and to stress in order to understand what needs to be done. We  
just assumed that the influence on the lattice due to the doping is  
small - which is certainly not true for all materials. If you find a  
way to implement this, please let me know ;)


Thomas

Zitat von Bin Shao :


Dear Thomas,



Thank you so much for your reply! I redo the calculation with a  
lower doping level and hide the gate in the barrier as you  
suggested. The calculation successfully gets converged. However, I  
have another question that if we relax the lattice constant of 2D  
materials, e.g. graphene, under the field-effect setup, i.e. the  
influence of gating on the lattice constant of 2D materials. Is this  
meaningful? If yes, can we do this by setting calculation =  
‘relax-vc’ or by calculating the curve of energy vs. lattice constant?




Best,

Bin






From: Dr. Thomas Brumme 
Sent: Monday, September 16, 2019 7:42:59 PM
To: Quantum ESPRESSO users Forum ;  
bshao...@outlook.com 
Subject: Re: [QE-users] bad convergence in field-effect  
configuration calculations


Dear Bin,

I can't check your input thoroughly as I'm on vacation, but there are
2 things I noticed:

1. The doping seems quite high - maybe you're close to the van Hove
singularity which could explain problems. Can you try with less doping
and then - if this converges - increase the doping till you get
problems? And plot the total potential - maybe the charge is already
spilling into the vacuum?! This is connected to the second problem:
2. It can help if you "hide" the gate in the barrier. A typical setup
I used recently:
  zgate= 0.589,
  emaxpos  = 0.59,
  eopreg   = 0.01,
  block= .true.,
  block_1  = 0.50,
  block_2  = 0.60,
  block_height = 2.50,
and the system is below 0.5... E.g. for your case, try
zgate = 0.131
block = .true.
block_1 = 0.13
block_2 = 0.23
block_height = 2.5
edir = 3
emaxpos = 0.13
eopreg = 0.01
The barrier starts with the dipole correction and the gate is right
behind the dipole... Compare with Fig. 2 of the PRB (2014). Also note
that 2.5 Ry for the barrier height is an arbitrarily chosen value - I
noticed other people now also use 2.5 Ry even if 2 Ry or even 1.5 Ry
should be enough. Yet, the higher the doping, the higher the dipole
correction the higher the barrier (if you "hide" everything in the
barrier)...

Regards & Greeting from the lake Garda in Italy

Thomas

Zitat von Bin Shao :


Dear all,

I would like to learn how to set up calculations of field-effect
configuration in quantum espresso. I read the paper "T. Brumme, M.
Calandra, F. Mauri; PRB 89, 245406 (2014)." and started with
graphene as an example. The input file is as follows. During the scf
calculation, there are a lot of warnings like " c_bands:  1
eigenvalues not converged". And the calculation could not get
converged after 800 iterations, could anyone help me with the input
file? Thanks in advance!

Best regards,
Bin



  calculation = 'scf'
  etot_conv_thr =   2.00d-05
  forc_conv_thr =   1.00d-04
  outdir = './out/'
  prefix = 'aiida'
  pseudo_dir = './pseudo/'
  tprnfor = .true.
  tstress = .true.
  gate = .true.
  dipfield = .true.
  tefield = .true.
  verbosity = 'high'
/

  degauss =   0.02
  ecutrho =   500
  ecutwfc =   45
  ibrav = 0
  nat = 2
  ntyp = 1
  occupations = 'smearing'
  smearing = 'cold'
  tot_charge = -0.2
  zgate = 0.1
  block = .true.
  block_1 = 0.13
  block_2 = 0.23
  block_height = 2.5
  edir = 3
  emaxpos = 0.005
  eopreg = 0.01
/

  conv_thr =   1.00d-9
  electron_maxstep = 800
  mixing_beta =  0.4
  mixing_mode = 'local-TF'
/
ATOMIC_SPECIES
C  12.011 C.pbe-n-kjpaw_psl.1.0.0.UPF
ATOMIC_POSITIONS crystal
C0.00   0.00   0.35
C0.333000   0.667000   0.35
K_POINTS automatic
65 65 1 0 0 0
CELL_PARAMETERS angstrom
  2.463800   0.00   0.00
 -1.231900   2.1336508000   0.00
  0.00   0.00  35.00
==




___
Quantum ESPRESSO is supported by MaX (www.max-centre.eu/quantum-espresso)
users mailing list users@lists.quantum-espresso.org
https://lists.quantum-espresso.org/mailman/listinfo/users

Re: [QE-users] bad convergence in field-effect configuration calculations

2019-09-16 Thread Dr. Thomas Brumme

Dear Bin,

I can't check your input thoroughly as I'm on vacation, but there are  
2 things I noticed:


1. The doping seems quite high - maybe you're close to the van Hove  
singularity which could explain problems. Can you try with less doping  
and then - if this converges - increase the doping till you get  
problems? And plot the total potential - maybe the charge is already  
spilling into the vacuum?! This is connected to the second problem:
2. It can help if you "hide" the gate in the barrier. A typical setup  
I used recently:

 zgate= 0.589,
 emaxpos  = 0.59,
 eopreg   = 0.01,
 block= .true.,
 block_1  = 0.50,
 block_2  = 0.60,
 block_height = 2.50,
and the system is below 0.5... E.g. for your case, try
   zgate = 0.131
   block = .true.
   block_1 = 0.13
   block_2 = 0.23
   block_height = 2.5
   edir = 3
   emaxpos = 0.13
   eopreg = 0.01
The barrier starts with the dipole correction and the gate is right  
behind the dipole... Compare with Fig. 2 of the PRB (2014). Also note  
that 2.5 Ry for the barrier height is an arbitrarily chosen value - I  
noticed other people now also use 2.5 Ry even if 2 Ry or even 1.5 Ry  
should be enough. Yet, the higher the doping, the higher the dipole  
correction the higher the barrier (if you "hide" everything in the  
barrier)...


Regards & Greeting from the lake Garda in Italy

Thomas

Zitat von Bin Shao :


Dear all,

I would like to learn how to set up calculations of field-effect  
configuration in quantum espresso. I read the paper "T. Brumme, M.  
Calandra, F. Mauri; PRB 89, 245406 (2014)." and started with  
graphene as an example. The input file is as follows. During the scf  
calculation, there are a lot of warnings like " c_bands:  1  
eigenvalues not converged". And the calculation could not get  
converged after 800 iterations, could anyone help me with the input  
file? Thanks in advance!


Best regards,
Bin



  calculation = 'scf'
  etot_conv_thr =   2.00d-05
  forc_conv_thr =   1.00d-04
  outdir = './out/'
  prefix = 'aiida'
  pseudo_dir = './pseudo/'
  tprnfor = .true.
  tstress = .true.
  gate = .true.
  dipfield = .true.
  tefield = .true.
  verbosity = 'high'
/

  degauss =   0.02
  ecutrho =   500
  ecutwfc =   45
  ibrav = 0
  nat = 2
  ntyp = 1
  occupations = 'smearing'
  smearing = 'cold'
  tot_charge = -0.2
  zgate = 0.1
  block = .true.
  block_1 = 0.13
  block_2 = 0.23
  block_height = 2.5
  edir = 3
  emaxpos = 0.005
  eopreg = 0.01
/

  conv_thr =   1.00d-9
  electron_maxstep = 800
  mixing_beta =  0.4
  mixing_mode = 'local-TF'
/
ATOMIC_SPECIES
C  12.011 C.pbe-n-kjpaw_psl.1.0.0.UPF
ATOMIC_POSITIONS crystal
C0.00   0.00   0.35
C0.333000   0.667000   0.35
K_POINTS automatic
65 65 1 0 0 0
CELL_PARAMETERS angstrom
  2.463800   0.00   0.00
 -1.231900   2.1336508000   0.00
  0.00   0.00  35.00
==




___
Quantum ESPRESSO is supported by MaX (www.max-centre.eu/quantum-espresso)
users mailing list users@lists.quantum-espresso.org
https://lists.quantum-espresso.org/mailman/listinfo/users


Re: [QE-users] Looking for systems that are difficult to converge with SCF

2019-08-14 Thread Thomas Brumme

Dear John,

I don't know if somebody already answered but I have some general input:
What I found really difficult to converge are magnetic systems with SOC 
(i.e., full-relativistic calculations with unpaired spins).
A typical example I was interested in (even if I cannot provide an input 
atm) are 2D transition-metal dichalcogenides with
adsorbed single-molecule magnets such as metal-phthalocyanines or 
porphyrins. I guess the problem is also due to the size
of the systems but you could also try a heavy atom on top of a chalcogen 
defect which reduces the system size.


Other system could be systems with 2 different stable charge states if 
you use DFT+U but there I don't have an example.


Regards

Thomas

On 8/12/19 8:35 PM, John McFarland wrote:


Dear all,


I'm trying to find systems that are difficult to converge with SCF.  
This includes systems where the energy will fluctuate with SCF 
iterations, or systems that converge to a local minimum or excited 
state.  Any info on where I could find or produce such systems, or 
even actual input files, would be most welcome.



Best regards,


John McFarland


___
Quantum ESPRESSO is supported by MaX (www.max-centre.eu/quantum-espresso)
users mailing list users@lists.quantum-espresso.org
https://lists.quantum-espresso.org/mailman/listinfo/users


--
Dr. rer. nat. Thomas Brumme
Wilhelm-Ostwald-Institute for Physical and Theoretical Chemistry
Leipzig University
Phillipp-Rosenthal-Strasse 31
04103 Leipzig

Tel:  +49 (0)341 97 36456

email: thomas.bru...@uni-leipzig.de

___
Quantum ESPRESSO is supported by MaX (www.max-centre.eu/quantum-espresso)
users mailing list users@lists.quantum-espresso.org
https://lists.quantum-espresso.org/mailman/listinfo/users

Re: [QE-users] missing f orbital - Sm atom reg

2019-08-06 Thread Thomas Brumme

Dear Muthu Vallinayagam,

please read the pdf provided with ld1.x (pseudo-gen.pdf in the folder 
atomic/Doc)

and some papers on pseudopotentials and their generation.

You will see, that the valence configuration used to create the pseudo 
has nothing
to do with the atomic configuration used in the calculation of the 
isolated atom.


Since the pseudo was created without f orbitals in valence (as apparent 
by the
missing f shell in the generation configuration) these states are not 
there. You
can't project onto something which is not in the calculation... Well, 
you can but

this would be meaningless.

Please note that this is already stated in the naming of the 
pseudopotential:


Sm.pbe-spdn-rrkjus_psl.1.0.0.UPF

spd means that semicore s, p, and d states were included. In order to 
include
f states as well, you either need to create your own pseudo or you need 
to find

another pseudo online.

Regards

Thomas


On 8/6/19 1:02 PM, Muthu V wrote:

Dear All,
i use Sm.pbe-spdn-rrkjus_psl.1.0.0.UPF, downloaded from QE web page, 
for my system which contains Sm atoms.
the electronic configuration of Sm contains f orbitals and the 
configuration is

*1s2 |   2s2 2p6 |  3s2 3p6 3d10 |  4s2 4p6 4d10   5s2 5p6 4f6 6s2*
when gone through this potential file i could see
  Valence configuration:
    nl pn  l   occ       Rcut      Rcut US       E pseu
    5S  1  0  2.00      1.400      1.700    -3.480513
    6S  2  0  1.50      1.400      1.700    -0.320378
    5P  2  1  6.00      1.400      1.800    -1.957179
    6P  3  1  0.50      1.400      1.800    -0.119872
    5D  3  2  1.00      1.400      2.000    -0.238636
    Generation configuration:
    5S  1  0  2.00      1.400      1.700    -3.480555
    6S  2  0  1.50      1.400      1.700    -0.320387
    5P  2  1  6.00      1.400      1.800    -1.957209
    6P  3  1  0.50      1.400      1.800    -0.119877
    5D  3  2  1.00      1.400      2.000    -0.238652
    5D  3  2  0.00      1.400      2.000     2.00
...
and
 input
 title='Sm',
   zed=62.0,
   rel=1,
config='[Xe] 4f5.0 5d1.0 6s1.5 6p0.5',
   iswitch=3,
   dft='PBE'
 /
...
when i did PDOS calculation, i do not see any f orbital contribution. 
"Valence configuration:"  and  " Generation configuration: " and the 
keyword "config" in input card are different from each other. my 
question is
whether electrons f orbitals  are treated as core electron ? if yes 
the why it is so ?


your explanation would be more help

best regards

Muthu Vallinayagam
PhD - student
HZDR, Germany


___
Quantum ESPRESSO is supported by MaX (www.max-centre.eu/quantum-espresso)
users mailing list users@lists.quantum-espresso.org
https://lists.quantum-espresso.org/mailman/listinfo/users


--
Dr. rer. nat. Thomas Brumme
Wilhelm-Ostwald-Institute for Physical and Theoretical Chemistry
Leipzig University
Phillipp-Rosenthal-Strasse 31
04103 Leipzig

Tel:  +49 (0)341 97 36456

email: thomas.bru...@uni-leipzig.de

___
Quantum ESPRESSO is supported by MaX (www.max-centre.eu/quantum-espresso)
users mailing list users@lists.quantum-espresso.org
https://lists.quantum-espresso.org/mailman/listinfo/users

Re: [QE-users] 2D CODE WILL NOT WORK, 2D MATERIAL NOT IN X-Y PLANE

2019-07-10 Thread Dr. Thomas Brumme

Dear  Julien,

Maybe it's because your cell vectors 1 and 2 are pointing
slightly in the z direction? I don't know but I think this
could be a problem. If it is just a rotation of the whole
cell you could try rotating it back... Otherwise search the
code for the message and look when it is triggered.

Regards

Thomas

Zitat von Julien Barbaud :


Dear users,



I am running into a new problem while trying to perform a calculation on a
2D slab consisting of 4 unit cells in the z-direction with an additional
molecule on top of an under-coordinated surface atom.

When I try to run the code (in serial mode just for checking), I get this
message:

2D CODE WILL NOT WORK, 2D MATERIAL NOT IN X-Y PLANE!!

2D CODE WILL NOT WORK, 2D MATERIAL NOT IN X-Y PLANE!!

(repeated twice as shown here)



I tried to search, but could not find any documentation on this error.



I thought that this might be due to a vacuum slab too small, but the
dimension should be ok according to previous calculations on similar
systems, and increasing the c lattice to very large values constant does not
solve the problem.

I made sure that the slab+molecule were centered around 0 in the z-direction
as I have been told on this list that this was important for the 2D code not
to cutoff some real coulombian interactions. This did not solve the problem.
I also tried to wrap all atoms in the cell, but this did not change anything
(plus it necessarily meant that the system was not centered around 0
anymore).



I do not understand this error message. What does "not in x-y plane" mean
for this 2D code? What confuses me is that I have been doing several very
similar calculations, and I never had this problem before. Any suggestion ?



I will join my input file



Thanks for reading,

Julien


--
Dr. rer. nat. Thomas Brumme
Wilhelm-Ostwald-Institute for Physical and Theoretical Chemistry
Leipzig University
Phillipp-Rosenthal-Strasse 31
04103 Leipzig

Tel:  +49 (0)341 97 36456

email: thomas.bru...@uni-leipzig.de

___
Quantum ESPRESSO is supported by MaX (www.max-centre.eu/quantum-espresso)
users mailing list users@lists.quantum-espresso.org
https://lists.quantum-espresso.org/mailman/listinfo/users


Re: [QE-users] dvscf file from a grid ph.x run

2019-07-10 Thread Dr. Thomas Brumme

Dear Ankit,

this discussion might help:

https://www.mail-archive.com/users@lists.quantum-espresso.org/msg33771.html

Regards

Thomas

Zitat von Ankit Jain :


Dear users,


I have been trying to collect and merge .dvscf files from a grid  
example of ph.x and combine these files into a single file (like it  
was created from serial single job).



So far I have little success on this. I searched on the mailing list  
and it seems that the similar question was asked earlier:



https://www.mail-archive.com/users@lists.quantum-espresso.org/msg29613.html


The suggestion from this earlier question was to write a small  
fortran script capable of reading and combining these binary files.  
I am wondering if such utility already exists or if not, if someone  
can point to piece of source code where these files are written so  
that I can attempt to read and combine these files.



Thank You,

Ankit Jain

Denmark Technical University


--
Dr. rer. nat. Thomas Brumme
Wilhelm-Ostwald-Institute for Physical and Theoretical Chemistry
Leipzig University
Phillipp-Rosenthal-Strasse 31
04103 Leipzig

Tel:  +49 (0)341 97 36456

email: thomas.bru...@uni-leipzig.de

___
Quantum ESPRESSO is supported by MaX (www.max-centre.eu/quantum-espresso)
users mailing list users@lists.quantum-espresso.org
https://lists.quantum-espresso.org/mailman/listinfo/users


Re: [QE-users] Freezing Intermediate Images in NEB Calculation

2019-07-01 Thread Thomas Brumme

Dear Achintya,

so, you want to simulate the layer sliding in SnSe in a phospherene-like 
structure.

And you get a barrier height of 1.25 eV. Does this make sense? I don't know.
Maybe should ask this specific question if you want to know the answer. 
Maybe

you can find some published data on this kind of transition? Maybe your
colleagues can help. The input looks ok. Only suggestion I have: use 
climbing

image NEB which usually gives better transition states.

Regards

Thomas

On 7/1/19 1:37 PM, Achintya Kumar wrote:

Dear Thomas
My input and output files are attached
BEGIN

BEGIN_PATH_INPUT

restart_mode='from_scratch'
string_method='neb',
ds=2.D0,
opt_scheme="broyden",
num_of_images=15,
nstep_path = 60,
k_max=0.3D0,
k_min=0.20D0,
path_thr=0.1D0,
/
END_PATH_INPUT
BEGIN_ENGINE_INPUT

prefix="snse"
outdir="./outdir",
pseudo_dir="/cad/QE/pseudopotentials/upf_files",
/

ibrav= 0,
celldm(1)=1.889725989,
nat= 4,
ntyp= 2,
ecutwfc= 40.0D0,
ecutrho= 200.0D0,
occupations= "fixed",
/

conv_thr    = 1.D-6,
mixing_beta = 0.5D0,
/

/
ATOMIC_SPECIES
Sn 118.71 Sn.pbe-dn-kjpaw_psl.0.2.UPF
Se 78.960 Se.pbe-n-kjpaw_psl.0.2.UPF
CELL_PARAMETERS (alat=  1.88972599)
   4.393133495   0.0   0.0
   0.0   4.245162285   0.0
   0.0   0.0  25.0

K_POINTS automatic
14 14 1 0 0 0
BEGIN_POSITIONS
FIRST_IMAGE
ATOMIC_POSITIONS angstrom
Sn       -0.27090369  0.0   11.151283650
Se        0.27090369  0.0   13.851931775
Sn       -2.467470438 2.122581143   13.848716350
Se        2.467470438 2.122581143   11.148068225
INTERMEDIATE_IMAGE
ATOMIC_POSITIONS angstrom
Sn       0.0 0.0   11.151283650
Se       0.0 0.0   13.851931775
Sn       2.196566748 2.122581143   13.848716350
Se       2.196566748 2.122581143   11.148068225
LAST_IMAGE
ATOMIC_POSITIONS angstrom
Sn       0.27090369  0.0   11.151283650
Se      -0.27090369  0.0   13.851931775
Sn       2.467470438 2.122581143   13.848716350
Se      -2.467470438 2.122581143   11.148068225
END_POSITIONS
END_ENGINE_INPUT

END
#
 activation energy (->) =   1.251588 eV
     activation energy (<-) =   1.251588 eV

     image        energy (eV)        error (eV/A)  frozen

         1     -17182.1943011            0.998235            T
         2     -17182.4628221            0.079139            F
         3     -17181.6722586            0.069784            F
         4     -17181.2913646            0.124247            F
         5     -17181.0750253            0.130701            F
         6     -17181.2203018            0.125583            F
         7     -17182.2031417            0.146372            F
         8     -17182.0989949            0.080510            F
         9     -17181.3352176            0.093461            F
        10     -17180.9427134            0.045147            F
        11     -17181.0428918            0.172851            F
        12     -17181.6282545            0.148683            F
        13     -17181.7506510            0.946514            F
        14     -17182.4631851            0.193101            F
        15     -17182.1943013            0.997393            T

     path length          = 32.289 bohr
     inter-image distance =  2.306 bohr

 ---


     neb: reached the maximum number of steps

     NEB          :     9h37m CPU       10h36m WALL


   This run was terminated on:  22:39:42  14Jun2019

=--=
   JOB DONE.
=------=

On Mon, Jul 1, 2019 at 4:58 PM Thomas Brumme 
mailto:thomas.bru...@uni-leipzig.de>> 
wrote:


Dear Achintya,

without any info on how you made your NEB calculation, which
system using this or that
parameter, we cannot judge if it is correct or not. If you don't
want to or can't provide these
informations, then you need to discuss this with some locals. Your
supervisor?
In general, if you want to know if an NEB makes sense you need to
have an idea of the
potential energy surface and look at the final path...

Thomas

On 7/1/19 12:02 PM, Achintya Kumar wrote:

Dear Thomas
How can I make sure that my neb calculation is correct or not.

On Mon, Jul 1, 2019 at 3:29 PM Achintya Kumar
mailto:kumar.achinty...@gmail.com>>
wrote:

Dear Thomas
I have done the same thing by splitting my path in two
separate paths, but I was just interested to know whether it
is possible or not to fix an intermediate images.
Thanks for your answer.

On Mon, Jul 1, 2019 at 1:55 PM Thomas Brumme
mailto:thomas.bru...@uni-leipzig.de>> wrote:

Dear Achintya,

I think t

Re: [QE-users] Freezing Intermediate Images in NEB Calculation

2019-07-01 Thread Thomas Brumme

Dear Achintya,

without any info on how you made your NEB calculation, which system 
using this or that
parameter, we cannot judge if it is correct or not. If you don't want to 
or can't provide these
informations, then you need to discuss this with some locals. Your 
supervisor?
In general, if you want to know if an NEB makes sense you need to have 
an idea of the

potential energy surface and look at the final path...

Thomas

On 7/1/19 12:02 PM, Achintya Kumar wrote:

Dear Thomas
How can I make sure that my neb calculation is correct or not.

On Mon, Jul 1, 2019 at 3:29 PM Achintya Kumar 
mailto:kumar.achinty...@gmail.com>> wrote:


Dear Thomas
I have done the same thing by splitting my path in two separate
paths, but I was just interested to know whether it is possible or
not to fix an intermediate images.
Thanks for your answer.

On Mon, Jul 1, 2019 at 1:55 PM Thomas Brumme
mailto:thomas.bru...@uni-leipzig.de>> wrote:

Dear Achintya,

I think that fixing an intermediate image is not possible as
this makes no sense.
If an image is fixed, it is also an end point of an elastic
band... So, if you just
split your path in two separate paths you can fix this
"intermediate" image.

Cheerio

Thomas

On 7/1/19 8:19 AM, Achintya Kumar wrote:

Hello
How can I kept frozen intermediate images in NEB calculation?

-- 
=

Achintya Priydarshi
Research Scholar
IIT Kanpur.
===

___
Quantum ESPRESSO is supported by MaX (www.max-centre.eu/quantum-espresso  
<http://www.max-centre.eu/quantum-espresso>)
users mailing listus...@lists.quantum-espresso.org  
<mailto:users@lists.quantum-espresso.org>
https://lists.quantum-espresso.org/mailman/listinfo/users


    -- 
    Dr. rer. nat. Thomas Brumme

Wilhelm-Ostwald-Institute for Physical and Theoretical Chemistry
Leipzig University
Phillipp-Rosenthal-Strasse 31
04103 Leipzig

Tel:  +49 (0)341 97 36456

email:thomas.bru...@uni-leipzig.de  
<mailto:thomas.bru...@uni-leipzig.de>



-- 
=

Achintya Priydarshi
Research Scholar
IIT Kanpur.
===



--
=
Achintya Priydarshi
Research Scholar
IIT Kanpur.
===========


--
Dr. rer. nat. Thomas Brumme
Wilhelm-Ostwald-Institute for Physical and Theoretical Chemistry
Leipzig University
Phillipp-Rosenthal-Strasse 31
04103 Leipzig

Tel:  +49 (0)341 97 36456

email: thomas.bru...@uni-leipzig.de

___
Quantum ESPRESSO is supported by MaX (www.max-centre.eu/quantum-espresso)
users mailing list users@lists.quantum-espresso.org
https://lists.quantum-espresso.org/mailman/listinfo/users

Re: [QE-users] Freezing Intermediate Images in NEB Calculation

2019-07-01 Thread Thomas Brumme

Dear Achintya,

I wrote:

"I think that fixing an intermediate image is not possible as this makes 
no sense."


and additionally I didn't find anything in the input description. Thus, 
I would change

my "I think" to "I believe"

Cheerio

Thomas

On 7/1/19 11:59 AM, Achintya Kumar wrote:

Dear Thomas
I have done the same thing by splitting my path in two separate paths, 
but I was just interested to know whether it is possible or not to fix 
an intermediate images.

Thanks for your answer.

On Mon, Jul 1, 2019 at 1:55 PM Thomas Brumme 
mailto:thomas.bru...@uni-leipzig.de>> 
wrote:


Dear Achintya,

I think that fixing an intermediate image is not possible as this
makes no sense.
If an image is fixed, it is also an end point of an elastic
band... So, if you just
split your path in two separate paths you can fix this
"intermediate" image.

Cheerio

Thomas

On 7/1/19 8:19 AM, Achintya Kumar wrote:

Hello
How can I kept frozen intermediate images in NEB calculation?

-- 
=

Achintya Priydarshi
Research Scholar
IIT Kanpur.
===

___
Quantum ESPRESSO is supported by MaX (www.max-centre.eu/quantum-espresso  
<http://www.max-centre.eu/quantum-espresso>)
users mailing listus...@lists.quantum-espresso.org  
<mailto:users@lists.quantum-espresso.org>
https://lists.quantum-espresso.org/mailman/listinfo/users


-- 
Dr. rer. nat. Thomas Brumme

Wilhelm-Ostwald-Institute for Physical and Theoretical Chemistry
Leipzig University
Phillipp-Rosenthal-Strasse 31
04103 Leipzig

Tel:  +49 (0)341 97 36456

email:thomas.bru...@uni-leipzig.de  <mailto:thomas.bru...@uni-leipzig.de>



--
=
Achintya Priydarshi
Research Scholar
IIT Kanpur.
===


--
Dr. rer. nat. Thomas Brumme
Wilhelm-Ostwald-Institute for Physical and Theoretical Chemistry
Leipzig University
Phillipp-Rosenthal-Strasse 31
04103 Leipzig

Tel:  +49 (0)341 97 36456

email: thomas.bru...@uni-leipzig.de

___
Quantum ESPRESSO is supported by MaX (www.max-centre.eu/quantum-espresso)
users mailing list users@lists.quantum-espresso.org
https://lists.quantum-espresso.org/mailman/listinfo/users

Re: [QE-users] Freezing Intermediate Images in NEB Calculation

2019-07-01 Thread Thomas Brumme

Dear Achintya,

I think that fixing an intermediate image is not possible as this makes 
no sense.
If an image is fixed, it is also an end point of an elastic band... So, 
if you just

split your path in two separate paths you can fix this "intermediate" image.

Cheerio

Thomas

On 7/1/19 8:19 AM, Achintya Kumar wrote:

Hello
How can I kept frozen intermediate images in NEB calculation?

--
=
Achintya Priydarshi
Research Scholar
IIT Kanpur.
===

___
Quantum ESPRESSO is supported by MaX (www.max-centre.eu/quantum-espresso)
users mailing list users@lists.quantum-espresso.org
https://lists.quantum-espresso.org/mailman/listinfo/users


--
Dr. rer. nat. Thomas Brumme
Wilhelm-Ostwald-Institute for Physical and Theoretical Chemistry
Leipzig University
Phillipp-Rosenthal-Strasse 31
04103 Leipzig

Tel:  +49 (0)341 97 36456

email: thomas.bru...@uni-leipzig.de

___
Quantum ESPRESSO is supported by MaX (www.max-centre.eu/quantum-espresso)
users mailing list users@lists.quantum-espresso.org
https://lists.quantum-espresso.org/mailman/listinfo/users

Re: [QE-users] fft order too large

2019-04-25 Thread Thomas Brumme

Dear Paolo,

thanks. The patch is working. Yet, I don't understand why. Usually 
activating tefield disables symmetries,

so the fractional translations shouldn't be important. Anyway, it works :)

Thomas

On 4/25/19 3:37 PM, Paolo Giannozzi wrote:
Very funny. I think I know why: it's a highly nonlocal effect of a 
small change I recently did to get rid of an old and annoying problem 
with fractional translations not commensurate with FFT grid. 
Apparently the code found a fractional translation 1/13 and tried to 
make the FFT grid commensurate with it. Please try the attached patch


Paolo

On Thu, Apr 25, 2019 at 1:04 AM Dr. Thomas Brumme 
mailto:thomas.bru...@uni-leipzig.de>> 
wrote:


OK, sorry, my bad. First of all, the number of states has of
course nothing to
do with the fft dimension.

But I now found a different weird thing:
Another input, with the very same structure, except the position
of the 2 Pb
atoms, is working - well, it gets past the point and at least prints
the memory
requirements and then I stop it...

This:

  Pb       0.       0.       3.210853292
  Pb       3.9323       0.       3.210853292

is now

  Pb      -0.14214  -0.75210   3.369160329
  Pb      23.593818848  -0.74795   3.369161551

This makes no sense.

Thomas

Zitat von "Dr. Thomas Brumme" mailto:thomas.bru...@uni-leipzig.de>>:

> Dear Paolo,
>
> thanks for the suggestion which I also found in the mail archive
but I'm also
> wondering why it was working with older versions of the code but
not with
> newer. Also, I don't understand what should be the problem with
the input.
> Sure, it is a large system, but on the other hand I already
calculated even
> larger systems. Is it just the combination of large cell+a lot
of electrons
> and SOC?
>
> Regards
>
> Thomas
>
>
> Zitat von Paolo Giannozzi mailto:p.gianno...@gmail.com>>:
>
>> There is a check on FFT dimensions exceeding a maximum value of
2049. There
>> is no deep reason for that, just the assumption that such a
large value
>> cannot be due to other than a mistake in the input data. You
may easily
>> increase that limit (if you really need more than 2049: do you?
sure?) by
    >> editing FFTXlib/fft_param.f90
>>
>> Paolo
>>
>> On Wed, Apr 24, 2019 at 4:48 PM Thomas Brumme
mailto:thomas.bru...@uni-leipzig.de>>
>> wrote:
>>
>>> Dear all,
>>>
>>> I have a problem running some old input with newer version of QE.
>>> I'm always getting, nearly at the beginning of the run, right
after:
>>>
>>>      Subspace diagonalization in iterative solution of the
eigenvalue
>>> problem:
>>>      a serial algorithm will be used
>>>
>>> the error:
>>>
>>>
>>>
>>>

%%
>>>      Error in routine  good_fft_order (2050):
>>>       fft order too large
>>>
>>>
>>>

%%
>>>
>>> The very same input is running with versions 6.1, 6.2.1, 6.2.2.
>>> It stops working with some dev version after 6.3 and is not
working
>>> with version 6.4.1.
>>>
>>> I have absolutely no clue why it is not working. I'll paste
the input
>>> below. Even if it is a large system I would call it a MWE as the
>>> calculation
>>> is not starting.
>>>
>>> Um, and all versions I tested where compiled with the ifort
version 18.0.3
>>> using the intel mkl and scalapack libraries.
>>>
>>> Thanks for your help!
>>>
>>> Thomas
>>>
>>> Input:
>>>
>>> 
>>>  calculation   = 'relax',
>>>  restart_mode  = 'from_scratch',
>>>  prefix        = '2Pb_line_1',
>>>  wf_collect    = .true.,
>>>  pseudo_dir    = '/home/tbrumme/structures/IrTe2/',
>>>  outdir        = '/scratch/tbrumme/',
>>>  verbosity     = 'high',
>>>  nstep         = 300,
>>>  tefield       = .true.,
>>>  dipfield      = .true.,
>>> /
>>> 
>>>  ibrav       = 0,
>>>  nat         = 158,
>>>  ntyp        

Re: [QE-users] fft order too large

2019-04-24 Thread Dr. Thomas Brumme

OK, sorry, my bad. First of all, the number of states has of course nothing to
do with the fft dimension.

But I now found a different weird thing:
Another input, with the very same structure, except the position of the 2 Pb
atoms, is working - well, it gets past the point and at least prints  
the memory

requirements and then I stop it...

This:

 Pb   0.   0.   3.210853292
 Pb   3.9323   0.   3.210853292

is now

 Pb  -0.14214  -0.75210   3.369160329
 Pb  23.593818848  -0.74795   3.369161551

This makes no sense.

Thomas

Zitat von "Dr. Thomas Brumme" :


Dear Paolo,

thanks for the suggestion which I also found in the mail archive but I'm also
wondering why it was working with older versions of the code but not with
newer. Also, I don't understand what should be the problem with the input.
Sure, it is a large system, but on the other hand I already calculated even
larger systems. Is it just the combination of large cell+a lot of electrons
and SOC?

Regards

Thomas


Zitat von Paolo Giannozzi :


There is a check on FFT dimensions exceeding a maximum value of 2049. There
is no deep reason for that, just the assumption that such a large value
cannot be due to other than a mistake in the input data. You may easily
increase that limit (if you really need more than 2049: do you? sure?) by
editing FFTXlib/fft_param.f90

Paolo

On Wed, Apr 24, 2019 at 4:48 PM Thomas Brumme 
wrote:


Dear all,

I have a problem running some old input with newer version of QE.
I'm always getting, nearly at the beginning of the run, right after:

 Subspace diagonalization in iterative solution of the eigenvalue
problem:
 a serial algorithm will be used

the error:


  
%%

 Error in routine  good_fft_order (2050):
  fft order too large

  
%%


The very same input is running with versions 6.1, 6.2.1, 6.2.2.
It stops working with some dev version after 6.3 and is not working
with version 6.4.1.

I have absolutely no clue why it is not working. I'll paste the input
below. Even if it is a large system I would call it a MWE as the
calculation
is not starting.

Um, and all versions I tested where compiled with the ifort version 18.0.3
using the intel mkl and scalapack libraries.

Thanks for your help!

Thomas

Input:


 calculation   = 'relax',
 restart_mode  = 'from_scratch',
 prefix= '2Pb_line_1',
 wf_collect= .true.,
 pseudo_dir= '/home/tbrumme/structures/IrTe2/',
 outdir= '/scratch/tbrumme/',
 verbosity = 'high',
 nstep = 300,
 tefield   = .true.,
 dipfield  = .true.,
/

 ibrav   = 0,
 nat = 158,
 ntyp= 3,
 ecutwfc = 60,
 ecutrho = 480,
 occupations = 'smearing',
 smearing= 'gauss',
 degauss = 0.05,
 vdw_corr= 'DFT-D3',
lspinorb=.true.
noncolin=.TRUE.
 emaxpos = 0.5,
 eopreg  = 0.03,
 edir= 3,
 eamp= 0.0,
/

 electron_maxstep = 250,
 diagonalization  = 'david',
 mixing_mode  = 'plain',
 mixing_beta  = 0.2,
 conv_thr = 1.0d-8,
/

 ion_dynamics='bfgs',
/
CELL_PARAMETERS angstrom
51.1199   0.   0.
 0.  13.62188678   0.
 0.   0.  20.
ATOMIC_SPECIES
 Ir  192.2173 Ir.rel-pbe-spn-kjpaw_psl.1.0.0.UPF
 Te  127.603  Te.rel-pbe-n-kjpaw_psl.1.0.0.UPF
 Pb  207.2Pb.rel-pbe-dn-kjpaw_psl.1.0.0.UPF
ATOMIC_POSITIONS angstrom
Ir   0.   0.   0.
Te   1.96615000   1.13515723   1.34927500
Te   1.96615000  -1.13515723  -1.34927500
Ir  -1.96615000   3.40547170   0.
Te   0.  -2.27031446   1.34927500
Te   0.   2.27031446  -1.34927500
Ir   0.  -6.81094339   0.
Te   1.96615000  -5.67578616   1.34927500
Te   1.96615000   5.67578616  -1.34927500
Ir  -1.96615000  -3.40547170   0.
Te   0.   4.54062893   1.34927500
Te   0.  -4.54062893  -1.34927500
Ir   3.9323  -6.81094339   0.
Te   5.89845000  -5.67578616   1.34927500
Te   5.89845000   5.67578616  -1.34927500
Ir   1.96615000  -3.40547170   0.
Te   3.9323   4.54062893   1.34927500
Te   3.9323  -4.54062893  -1.34927500
Ir   3.9323   0.   0.
Te   5.89845000   1.13515723   1.34927500
Te   5.89845000  -1.13515723  -1.34927500
Ir   1.96615000   3.40547170   0.
Te   3.9323  -2.27031446   1.34927500
Te   3.9323   2.27031446  -1.34927500
Ir   7.8646  -6.81094339   0.
Te   9.83075000  -5.67578616   

Re: [QE-users] fft order too large

2019-04-24 Thread Dr. Thomas Brumme

Dear Paolo,

thanks for the suggestion which I also found in the mail archive but I'm also
wondering why it was working with older versions of the code but not with
newer. Also, I don't understand what should be the problem with the input.
Sure, it is a large system, but on the other hand I already calculated even
larger systems. Is it just the combination of large cell+a lot of electrons
and SOC?

Regards

Thomas


Zitat von Paolo Giannozzi :


There is a check on FFT dimensions exceeding a maximum value of 2049. There
is no deep reason for that, just the assumption that such a large value
cannot be due to other than a mistake in the input data. You may easily
increase that limit (if you really need more than 2049: do you? sure?) by
editing FFTXlib/fft_param.f90

Paolo

On Wed, Apr 24, 2019 at 4:48 PM Thomas Brumme 
wrote:


Dear all,

I have a problem running some old input with newer version of QE.
I'm always getting, nearly at the beginning of the run, right after:

  Subspace diagonalization in iterative solution of the eigenvalue
problem:
  a serial algorithm will be used

the error:


   
%%

  Error in routine  good_fft_order (2050):
   fft order too large

   
%%


The very same input is running with versions 6.1, 6.2.1, 6.2.2.
It stops working with some dev version after 6.3 and is not working
with version 6.4.1.

I have absolutely no clue why it is not working. I'll paste the input
below. Even if it is a large system I would call it a MWE as the
calculation
is not starting.

Um, and all versions I tested where compiled with the ifort version 18.0.3
using the intel mkl and scalapack libraries.

Thanks for your help!

Thomas

Input:


  calculation   = 'relax',
  restart_mode  = 'from_scratch',
  prefix= '2Pb_line_1',
  wf_collect= .true.,
  pseudo_dir= '/home/tbrumme/structures/IrTe2/',
  outdir= '/scratch/tbrumme/',
  verbosity = 'high',
  nstep = 300,
  tefield   = .true.,
  dipfield  = .true.,
/

  ibrav   = 0,
  nat = 158,
  ntyp= 3,
  ecutwfc = 60,
  ecutrho = 480,
  occupations = 'smearing',
  smearing= 'gauss',
  degauss = 0.05,
  vdw_corr= 'DFT-D3',
lspinorb=.true.
noncolin=.TRUE.
  emaxpos = 0.5,
  eopreg  = 0.03,
  edir= 3,
  eamp= 0.0,
/

  electron_maxstep = 250,
  diagonalization  = 'david',
  mixing_mode  = 'plain',
  mixing_beta  = 0.2,
  conv_thr = 1.0d-8,
/

  ion_dynamics='bfgs',
/
CELL_PARAMETERS angstrom
51.1199   0.   0.
  0.  13.62188678   0.
  0.   0.  20.
ATOMIC_SPECIES
  Ir  192.2173 Ir.rel-pbe-spn-kjpaw_psl.1.0.0.UPF
  Te  127.603  Te.rel-pbe-n-kjpaw_psl.1.0.0.UPF
  Pb  207.2Pb.rel-pbe-dn-kjpaw_psl.1.0.0.UPF
ATOMIC_POSITIONS angstrom
Ir   0.   0.   0.
Te   1.96615000   1.13515723   1.34927500
Te   1.96615000  -1.13515723  -1.34927500
Ir  -1.96615000   3.40547170   0.
Te   0.  -2.27031446   1.34927500
Te   0.   2.27031446  -1.34927500
Ir   0.  -6.81094339   0.
Te   1.96615000  -5.67578616   1.34927500
Te   1.96615000   5.67578616  -1.34927500
Ir  -1.96615000  -3.40547170   0.
Te   0.   4.54062893   1.34927500
Te   0.  -4.54062893  -1.34927500
Ir   3.9323  -6.81094339   0.
Te   5.89845000  -5.67578616   1.34927500
Te   5.89845000   5.67578616  -1.34927500
Ir   1.96615000  -3.40547170   0.
Te   3.9323   4.54062893   1.34927500
Te   3.9323  -4.54062893  -1.34927500
Ir   3.9323   0.   0.
Te   5.89845000   1.13515723   1.34927500
Te   5.89845000  -1.13515723  -1.34927500
Ir   1.96615000   3.40547170   0.
Te   3.9323  -2.27031446   1.34927500
Te   3.9323   2.27031446  -1.34927500
Ir   7.8646  -6.81094339   0.
Te   9.83075000  -5.67578616   1.34927500
Te   9.83075000   5.67578616  -1.34927500
Ir   5.89845000  -3.40547170   0.
Te   7.8646   4.54062893   1.34927500
Te   7.8646  -4.54062893  -1.34927500
Ir   7.8646   0.   0.
Te   9.83075000   1.13515723   1.34927500
Te   9.83075000  -1.13515723  -1.34927500
Ir   5.89845000   3.40547170   0.
Te   7.8646  -2.27031446   1.34927500
Te   7.8646   2.27031446  -1.34927500
Ir  11.7969  -6.81094339   0.
Te

[QE-users] fft order too large

2019-04-24 Thread Thomas Brumme
Te  -1.96615000   5.67578616  -1.34927500
Ir  -5.89845000  -3.40547170   0.
Te  -3.9323   4.54062893   1.34927500
Te  -3.9323  -4.54062893  -1.34927500
Ir  -3.9323   0.   0.
Te  -1.96615000   1.13515723   1.34927500
Te  -1.96615000  -1.13515723  -1.34927500
Ir  -5.89845000   3.40547170   0.
Te  -3.9323  -2.27031446   1.34927500
Te  -3.9323   2.27031446  -1.34927500
Pb   0.   0.   3.210853292
Pb   3.9323   0.   3.210853292
K_POINTS automatic
2 8 1 0 0 0

--
Dr. rer. nat. Thomas Brumme
Wilhelm-Ostwald-Institute for Physical and Theoretical Chemistry
Leipzig University
Phillipp-Rosenthal-Strasse 31
04103 Leipzig

Tel:  +49 (0)341 97 36456

email: thomas.bru...@uni-leipzig.de

___
Quantum Espresso is supported by MaX (www.max-centre.eu/quantum-espresso)
users mailing list users@lists.quantum-espresso.org
https://lists.quantum-espresso.org/mailman/listinfo/users

Re: [QE-users] unefficient parallelization of scf calculation

2019-04-12 Thread Thomas Brumme

Dear Julien,

it's actually described indirectly in the paper - in all figures you can 
see that the
system is centered a z = 0. Otherwise, the cutoff will cut off some of 
the real
Coulomb interactions. Probably this should be added to the description, 
but this
should be done by the original author. If he doesn't answer here, I will 
contact

him.

The way you center should be more like (zh+zl)=0 - I don't understand why
you want to divide by 2 (zero divided by 2?!). That the slab isn't 
centered should
not be problematic. From your input I also see that the vacuum region 
should be
large enough but maybe you can try increasing it a bit more - maybe 
increase the

cell dimension in z to 45 or 50 Angstrom?

Cheerio

Thomas


On 4/12/19 11:30 AM, JULIEN, CLAUDE, PIERRE BARBAUD wrote:


Well actually, this remark may be much closer to solve my problem than 
you might think !


I tried to run the simulation again on the cluster without the 
assume_isolated=’2D’ flag, and the parallelization was efficient this 
time. It seemed to scale well with growing number of procs. So thank 
you very much for this suggestion.


However, this raised other questions that I was not able to find an 
answer to:


·Where does the information about the need to center the system come 
from? I could not find track of it in the INPUT_PW documentation. I 
only quickly glanced over the paper mentioned as reference in the ‘2D’ 
flag, but I don’t suppose that such technical details would be 
explained there ( as I imagine that it is more relevant to the 
particular coding implementation in quantum espresso than the general 
method described in the article)


·I tried to correct the assume_isolated, but I might have 
misunderstood how to center the system. I just took the atom with 
“highest z” zh and the one with “lowest z” zl, and made sure to shift 
the whole so that the average (zh+zl)/2=0. This did not work. However, 
this particular example being made of a slab and a 2d layer over it, 
this means, among others, that the slab itself is not centered, and I 
am unsure whether this is the correct way to do it


Best,

Julien

*From:*Thomas Brumme [mailto:thomas.bru...@uni-leipzig.de]
*Sent:* mercredi 10 avril 2019 18:19
*To:* Quantum Espresso users Forum; Julien Barbaud
*Subject:* Re: [QE-users] unefficient parallelization of scf calculation

Dear Julien,

I can't give any valuable input for your question regarding the 
parallelization, but I think your
input is wrong. Using assume_isolated needs the system to be centered 
around z=0.


Regards

Thomas

On 4/10/19 11:36 AM, Julien Barbaud wrote:

I am starting to use a hpc cluster of my university, but I am very
green on parallel computation.

I have made a first test (test #1) on a very small-scale
simulation (relaxation of a GO sheet with 19 atoms, with respect
to the gamma point). The calculation took 3m20s to run on 1 proc
on my personal computer. On the cluster with 4 proc and default
parallel options, it took 1m5s, and on 8 proc it took 44s. This
seems like a reasonable behavior, and at least shows that raising
the number of procs does reduce computation time in this case
(with obvious limitations if too many procs for the job).

However I tried with another test, a bit bigger (test #2). This
example is a scf calculation with 120 atoms (still with respect to
the gamma point). In this case, the parallelization brings
absolutely no improvement. In fact, although the /outfile/
confirms that the code is running on N procs, it has similar
performances as if it was running on 1 proc (sometimes even worse
actually, but probably not in a significant manner, as the times
are fluctuating a bit from 1 run to another)

I tried to run this same input file on my personal computer both
on 1 and 2 cores. Turns out that it takes 10376s to run 10
iterations on 1 core, while it takes 6777s on two cores, so it
seems that the parallelization is doing ok on my computer.

I have tried to run with different number of cores on the hpc, and
different parallelization options (like for instance –nb 4), but
nothing seems to improve the time

Basically, I am stuck with those 2 seemingly conflicting facts:

  * Parallelization seems to have no particular problem on the hpc
cluster because test #1 gives good results
  * Parallelization seems to have no particular problem with the
particular input file #2 because it seems to scale reasonably
with proc number on my individual computer

However, combining both and running this file in parallel on the
hpc cluster ends up not working correctly…

I included below the input file and output file of test #2. I also
included as well as the slurm script that I use to submit the
calculation to the job manager, in case it helps (test2.scf.slurm.txt)

Any suggestion on what is going wrong would be very welcome

Re: [QE-users] unefficient parallelization of scf calculation

2019-04-10 Thread Thomas Brumme
  119.18s CPU    120.33s WALL (   1 calls)
 electrons    :   1961.71s CPU   1969.12s WALL (   1 calls)

 Called by init_run:
 wfcinit  : 52.26s CPU 52.44s WALL (   1 calls)
 potinit  : 19.26s CPU 19.33s WALL (   1 calls)
 hinit0   : 36.63s CPU 36.68s WALL (   1 calls)

 Called by electrons:
 c_bands  :   1919.78s CPU   1923.97s WALL (   1 calls)
 sum_band : 28.22s CPU 30.08s WALL (   1 calls)
 v_of_rho :  2.26s CPU  2.35s WALL (   2 calls)
 newd : 20.58s CPU 22.50s WALL (   2 calls)
 PAW_pot  :  4.00s CPU  4.00s WALL (   2 calls)
 mix_rho  :  0.23s CPU  0.24s WALL (   1 calls)

 Called by c_bands:
 init_us_2    :  0.22s CPU  0.27s WALL (   3 calls)
 regterg  :   1919.41s CPU   1923.60s WALL (   2 calls)

 Called by sum_band:
 sum_band:bec :  0.00s CPU  0.00s WALL (   1 calls)
 addusdens    : 16.57s CPU 17.94s WALL (   1 calls)

 Called by *egterg:
 h_psi    :    680.38s CPU    682.69s WALL (  43 calls)
 s_psi    :    259.57s CPU    259.75s WALL (  43 calls)
 g_psi    :  0.93s CPU  0.94s WALL (  40 calls)
 rdiaghg  : 52.76s CPU 52.86s WALL (  41 calls)

 Called by h_psi:
 h_psi:pot    :    679.62s CPU    681.90s WALL (  43 calls)
 h_psi:calbec :    255.27s CPU    255.54s WALL (  43 calls)
 vloc_psi :    164.42s CPU    166.01s WALL (  43 calls)
 add_vuspsi   :    259.93s CPU    260.35s WALL (  43 calls)

 General routines
 calbec   :    263.20s CPU    263.88s WALL (  44 calls)
 fft  :  2.33s CPU  2.43s WALL (  23 calls)
 ffts :  0.09s CPU  0.09s WALL (   3 calls)
 fftw :    128.50s CPU    130.07s WALL (   10237 calls)
 interpolate  :  0.25s CPU  0.26s WALL (   2 calls)
 davcio   :  0.00s CPU  0.10s WALL (   3 calls)

 Parallel routines
 fft_scatt_xy : 23.50s CPU 23.55s WALL (   10263 calls)
 fft_scatt_yz : 10.98s CPU 12.22s WALL (   10263 calls)

 PWSCF    : 34m45.53s CPU    34m55.12s WALL


   This run was terminated on:  16:10:30  10Apr2019

=--=
   JOB DONE.
=--=






*-SLURM 
command-*


*
*

#!/bin/bash

#SBATCH --job-name=QE_GO-Cl_bonding_scf
#SBATCH --partition=cpu
#SBATCH --mail-type=end
#SBATCH --mail-user=julien_barb...@sjtu.edu.cn
#SBATCH --output=bonding.scf.slurm.out
#SBATCH --error=bonding.scf.slurm.err
#SBATCH -p cpu
#SBATCH -n 8
#SBATCH --ntasks-per-node=8

ulimit -l unlimited
ulimit -s unlimited

INPUT=$HOME/QE/GO-Cl/FAPBI3_bonding/scf/1x2x3_matching/bonding.scf.in
EXEC=$HOME/QE/qe-6.3/bin/pw.x

srun --mpi=pmi2 $EXEC -in $INPUT


___
users mailing list
users@lists.quantum-espresso.org
https://lists.quantum-espresso.org/mailman/listinfo/users


--
Dr. rer. nat. Thomas Brumme
Wilhelm-Ostwald-Institute for Physical and Theoretical Chemistry
Leipzig University
Phillipp-Rosenthal-Strasse 31
04103 Leipzig

Tel:  +49 (0)341 97 36456

email: thomas.bru...@uni-leipzig.de

___
users mailing list
users@lists.quantum-espresso.org
https://lists.quantum-espresso.org/mailman/listinfo/users

Re: [QE-users] Optimization does not converge

2019-04-04 Thread Thomas Brumme
   0.2164
   C   0.3124999725702590   0.082967603452   0.2164
   C   0.249725702590   0.332967603450   0.2164
   C   0.1874999725702590   0.582967603448   0.2164
   C   0.124725702590   0.832967603447   0.2164
   C   0.4374999725702590   0.082967603452   0.2164
   C   0.374725702590   0.332967603450   0.2164
   C   0.3124999725702590   0.582967603448   0.2164
   C   0.249725702590   0.832967603447   0.2164
   C   0.5624999725702590   0.082967603451   0.2164
   C   0.499725702591   0.332967603449   0.2164
   C   0.4374999725702590   0.582967603438   0.2164
   C   0.374725702590   0.832967603447   0.2164
   C   0.6874999725702590   0.082967603451   0.2164
   C   0.624725702590   0.332967603450   0.2164
   C   0.5624999725702590   0.582967603447   0.2164
   C   0.499725702591   0.832967603448   0.2164
   C   0.8124999725702590   0.082967603451   0.2164
   C   0.749725702590   0.332967603450   0.2164
   C   0.6874999725702600   0.582967603440   0.2164
   C   0.624725702590   0.832967603438   0.2164
   C   0.9374999725702590   0.082967603450   0.2164
   C   0.874725702590   0.332967603450   0.2164
   C   0.8124999725702600   0.582967603438   0.2164
   C   0.749725702590   0.832967603438   0.2164
   C   0.9374999725702590   0.582967603438   0.2164
   C   0.874725702601   0.832967603448   0.2164
   O   0.5008325506691220   0.4975899051684459   0.2743438844378600
   H   0.4505865611121930   0.4983293698599460   0.2875531708238770
K_POINTS automatic
2  2  1  0  0  0


Wei gui
School of Mechanical Engineering?Chongqing University, China



___
users mailing list
users@lists.quantum-espresso.org
https://lists.quantum-espresso.org/mailman/listinfo/users


___
users mailing list
users@lists.quantum-espresso.org
https://lists.quantum-espresso.org/mailman/listinfo/users



--
Dr. rer. nat. Thomas Brumme
Wilhelm-Ostwald-Institute for Physical and Theoretical Chemistry
Leipzig University
Phillipp-Rosenthal-Strasse 31
04103 Leipzig

Tel:  +49 (0)341 97 36456

email: thomas.bru...@uni-leipzig.de

___
users mailing list
users@lists.quantum-espresso.org
https://lists.quantum-espresso.org/mailman/listinfo/users

Re: [QE-users] BoltzTraP imbedded in quantum espresso

2019-03-11 Thread Thomas Brumme

Dear Timothy,

from this error I would guess that there is some problem with your 
lapack installation.
Yet, without input or some more info we cannot help. Also, since when is 
BoltzTraP

embedded in QE?
Furthermore, BoltzTraP (old) does not work with the new QE 6.* but only 
5.4 as far as
I remember. BoltzTraP2 is not yet linked with QE... Maybe you want to do 
this and

share your work?

Regards

Thomas

On 3/10/19 3:58 PM, TIMOTHY UTO wrote:

Hi QE users,
Please has anybody  encounter this problem while using BoltzTrap 
imbedded with QE to calculate transport properties? and  how can I 
solve this problem please? The error is shown below.




error while loading shared libraries: liblapack.so.3gf: cannot
open shared object file: No such file or directory

Uto Timothy
PhD student
FUNAAB
Sent from Yahoo Mail on Android

<https://go.onelink.me/107872968?pid=InProduct=Global_Internal_YGrowth_AndroidEmailSig__AndroidUsers_wl=ym_sub1=Internal_sub2=Global_YGrowth_sub3=EmailSignature>


___
users mailing list
users@lists.quantum-espresso.org
https://lists.quantum-espresso.org/mailman/listinfo/users


--
Dr. rer. nat. Thomas Brumme
Wilhelm-Ostwald-Institute for Physical and Theoretical Chemistry
Leipzig University
Phillipp-Rosenthal-Strasse 31
04103 Leipzig

Tel:  +49 (0)341 97 36456

email: thomas.bru...@uni-leipzig.de

___
users mailing list
users@lists.quantum-espresso.org
https://lists.quantum-espresso.org/mailman/listinfo/users

Re: [QE-users] Total force does not coverge

2019-03-06 Thread Thomas Brumme
19.68       0.011  0.000
0.000       8.521689973238880  0.000
0.000       0.000  25.00
ATOMIC_SPECIES
Fe  55.847   Fe.pbe-nd-rrkjus.UPF
C  12.0107   C.pbe-rrkjus.UPF
O   16.00    O.pbe-rrkjus.UPF
ATOMIC_POSITIONS  crystal
..
K_POINTS automatic
2  2  1  0  0  0





Wei gui
School of Mechanical Engineering,Chongqing University, China



___
users mailing list
users@lists.quantum-espresso.org
https://lists.quantum-espresso.org/mailman/listinfo/users


--
Dr. rer. nat. Thomas Brumme
Wilhelm-Ostwald-Institute for Physical and Theoretical Chemistry
Leipzig University
Phillipp-Rosenthal-Strasse 31
04103 Leipzig

Tel:  +49 (0)341 97 36456

email: thomas.bru...@uni-leipzig.de

___
users mailing list
users@lists.quantum-espresso.org
https://lists.quantum-espresso.org/mailman/listinfo/users

Re: [QE-users] Convergence in Hexagonal Systems

2019-02-09 Thread Dr. Thomas Brumme

Dear Dr. Anuja Chanana,

without more information, we (everyone here in this users list) can't  
help you.
What do you mean with "difficult to reach on convergence"? How do you  
test convergence?


Is an scf cycle not converging? Assuming you calculate a "meaningful"  
structure, this
could either mean you need to increase the wave-function or the  
charge-density cutoff
or both. Or the k-point sampling. Again, I don't think it is a problem  
of the pseudo.
For both N and B, creating pseudos is pretty straightforward and I  
don't think the
pseudos of the PSlibrary are wrong or bad. On the other hand, looking  
at this site:


https://www.materialscloud.org/discover/sssp/plot/efficiency/N

I see that for a nitrogen PBE US pseudo of the PSLibrary a cutoff of  
40 Ry might be
just too small, especially when you want to relax the cell. For  
accurate forces and
pressure you always need a higher cutoff. (PBE is nearly PW91 but for  
the latter, I

guess, you need a higher cutoff)

What do you mean with "its difficult to reach on convergence"?  
Furthermore, why do

you use a smearing if the system is an insulator?

Regards

Thomas

P.S.: Please, always reply to "All", i.e., also to the list, so that  
others can

find the answer if they have the same problem.

Zitat von Anuja Chanana :


Dear Dr. Thomas Brumme,
Thanks for the information.
I tested the pseudopotential for bulk BN hexagonal system as well. But its
difficult to reach on convergence.
PFA the input file.


Thanks and Regards
Dr. Anuja Chanana


[image: Mailtrack]
<https://mailtrack.io?utm_source=gmail_medium=signature_campaign=signaturevirality5;>
Sender
notified by
Mailtrack
<https://mailtrack.io?utm_source=gmail_medium=signature_campaign=signaturevirality5;>
02/09/19,
11:08:44 PM

On Sat, Feb 9, 2019 at 10:19 PM Dr. Thomas Brumme <
thomas.bru...@uni-leipzig.de> wrote:


Dear Anuja Chanana,

I don't think it is the pseudo which gives problems in one case and none
in the other. Looking at the structure I guess that you want to
simulate an isolated
layer in the hexagonal system, don't you? Otherwise it would make no
sense to have a vacuum of 6 Angstrom. But then, why do you have only
6 Angstrom? And why do you use k-point sampling in a non-periodic
direction?
Furthermore, your in-plane k-point sampling is different in both systems
which
could explain other differences!? If you really want to simulate a 2D
system have a look at the input variable

assume_isolated='2D'

Otherwise, if you don't want to simulate a 2D system, please check your
structures with, e.g., XCrysDen.

Regards

Thomas Brumme


Zitat von Anuja Chanana :

> Hello all,
> I am trying to converge a hexagonal and cubic system of BN. The cubic
> system shows a convergence while the hexagonal system doesn't, though the
> parameters are the same. I guess there might be some issue in the
Nitrogen
> pseudopotential. Can you please look into the issue?
> PFA the input files for both the systems.
>
>
>
> Thanks and Regards
> Dr. Anuja Chanana
>
>
>
> [image: Mailtrack]
> <
https://mailtrack.io?utm_source=gmail_medium=signature_campaign=signaturevirality5;
>
> Sender
> notified by
> Mailtrack
> <
https://mailtrack.io?utm_source=gmail_medium=signature_campaign=signaturevirality5;
>
> 02/09/19,
> 2:41:41 PM

--
Dr. rer. nat. Thomas Brumme
Wilhelm-Ostwald-Institute for Physical and Theoretical Chemistry
Leipzig University
Phillipp-Rosenthal-Strasse 31
04103 Leipzig

Tel:  +49 (0)341 97 36456

email: thomas.bru...@uni-leipzig.de






___
users mailing list
users@lists.quantum-espresso.org
https://lists.quantum-espresso.org/mailman/listinfo/users


Re: [QE-users] Convergence in Hexagonal Systems

2019-02-09 Thread Dr. Thomas Brumme

Dear Anuja Chanana,

I don't think it is the pseudo which gives problems in one case and none
in the other. Looking at the structure I guess that you want to  
simulate an isolated

layer in the hexagonal system, don't you? Otherwise it would make no
sense to have a vacuum of 6 Angstrom. But then, why do you have only
6 Angstrom? And why do you use k-point sampling in a non-periodic direction?
Furthermore, your in-plane k-point sampling is different in both systems which
could explain other differences!? If you really want to simulate a 2D
system have a look at the input variable

assume_isolated='2D'

Otherwise, if you don't want to simulate a 2D system, please check your
structures with, e.g., XCrysDen.

Regards

Thomas Brumme


Zitat von Anuja Chanana :


Hello all,
I am trying to converge a hexagonal and cubic system of BN. The cubic
system shows a convergence while the hexagonal system doesn't, though the
parameters are the same. I guess there might be some issue in the Nitrogen
pseudopotential. Can you please look into the issue?
PFA the input files for both the systems.



Thanks and Regards
Dr. Anuja Chanana



[image: Mailtrack]
<https://mailtrack.io?utm_source=gmail_medium=signature_campaign=signaturevirality5;>
Sender
notified by
Mailtrack
<https://mailtrack.io?utm_source=gmail_medium=signature_campaign=signaturevirality5;>
02/09/19,
2:41:41 PM


--
Dr. rer. nat. Thomas Brumme
Wilhelm-Ostwald-Institute for Physical and Theoretical Chemistry
Leipzig University
Phillipp-Rosenthal-Strasse 31
04103 Leipzig

Tel:  +49 (0)341 97 36456

email: thomas.bru...@uni-leipzig.de

___
users mailing list
users@lists.quantum-espresso.org
https://lists.quantum-espresso.org/mailman/listinfo/users


Re: [QE-users] Spin Orbit Coupling with PAW PseudoPotentials (Full Relativistic PBESOL from pslib) and gaupbe xc

2019-01-15 Thread Thomas Brumme
s.f90
pw.x   0056F479 run_pwscf_    132  
run_pwscf.f90
pw.x   004091A0 MAIN__ 77  
pwscf.f90

pw.x   0040901E Unknown   Unknown  Unknown
libc-2.12.so <http://libc-2.12.so> 00328741ED5D  
__libc_start_main Unknown Unknown

pw.x   00408F29 Unknown   Unknown  Unknown

The questions I have are
1) Why is is_ultrasoft="true" set for PAW type pseudopotentials?
2) Would it be possible to run hybrid xc calculation including spin 
orbit coupling in Quantum Espresso? In particular the above shown 
calculation that included Full Relativistic PBESOL with GauPBE(or HSE06)

3) If it is possible please suggest how to resolve it.

By the way I am trying to study defects in CdTe including spin orbit 
coupling following the paper

PHYSICAL REVIEW B 98, 054108 (2018)

Regards
Abdul Rawoof
Phd Student in ECEE
Arizona State University



___
users mailing list
users@lists.quantum-espresso.org
https://lists.quantum-espresso.org/mailman/listinfo/users


--
Dr. rer. nat. Thomas Brumme
Wilhelm-Ostwald-Institute for Physical and Theoretical Chemistry
Leipzig University
Phillipp-Rosenthal-Strasse 31
04103 Leipzig

Tel:  +49 (0)341 97 36456

email: thomas.bru...@uni-leipzig.de

___
users mailing list
users@lists.quantum-espresso.org
https://lists.quantum-espresso.org/mailman/listinfo/users

Re: [QE-users] QE_2_BoltzTrap

2019-01-07 Thread Thomas Brumme
or: local variable 'nsym' referenced before
assignment.
 I am unable to sort out the error but I can say it may be
due nscf input.
So please guide me how to sort out this error?
Thanks and regards
*Mohammad Ubaid*
*PhD Research Scholar*
*Department of Physics*
*Jamia Millia Islamia University*
*New Delhi - 110025*
___
users mailing list
users@lists.quantum-espresso.org
<mailto:users@lists.quantum-espresso.org>
https://lists.quantum-espresso.org/mailman/listinfo/users



___
users mailing list
users@lists.quantum-espresso.org
https://lists.quantum-espresso.org/mailman/listinfo/users


--
Dr. rer. nat. Thomas Brumme
Wilhelm-Ostwald-Institute for Physical and Theoretical Chemistry
Leipzig University
Phillipp-Rosenthal-Strasse 31
04103 Leipzig

Tel:  +49 (0)341 97 36456

email: thomas.bru...@uni-leipzig.de

56a57,58
> !test
> !  write(*,*)'deltaef = ',deltaef
57a60
> !  write(*,*)'deltaef = ',deltaef
81c84,88
<  
---
>   !TB 2014
>   !OPEN(666,FILE='minv_sigxy', STATUS='unknown', FORM='formatted')
>   !OPEN(667,FILE='fermifunc', STATUS='unknown', FORM='formatted')
>   !write(*,*)'spinorb',spinorbit
>   !TB 2014
122a130,135
> 
>  !TB 2014
>  !factor1=fermi(efermi,-1.8*RYDBERG,temp)*spinorbit
>  !factor2=dfermide(efermi,-1.8*RYDBERG,temp)*spinorbit
>  !WRITE(667,*)efermi/RYDBERG,factor1,factor2
> 
204a218,221
>   !TB 2014
>   !CLOSE(666)
>   !CLOSE(667)
>   !TB 2014
239c256
<  INTEGER:: i,j,k
---
>  INTEGER:: i,j,k, tmp
250c267
<  REAL(8):: minv_sigxy(3,3),seebeck(3,3),thermal(3,3),hall(3,3,3)
---
>  REAL(8):: minv_sigxy(2,2),seebeck(3,3),thermal(3,3),hall(3,3,3)
254a272,274
>  !TB 2014
>  minv_sigxy=ZERO
>  !TB 2014
303a324
> 
304a326
> ! write(*,*)'test: spinorbit = ',spinorbit,' lifetime = ',lifetime
307,317c329,342
< cond(1:3,1:3) = -cond(1:3,1:3)*deltaef &
<  * OHM *METER*SECOND
< nu(1:3,1:3)   =   -nu(1:3,1:3)*deltaef/temp &
<  * METER * SECOND / AMPERE
< kappa(1:3,1:3)=-kappa(1:3,1:3)*deltaef/temp &
<  * METER * SECOND / WATT
< sigxyz(1:3,1:3,1:3)=-sigxyz*deltaef &
<  * OHM**2 * SECOND**3 * AMPERE / METER
< specheat  = specheat*deltaef & 
<  * MOL / JOULE
< CALL invert_3x3(cond,minv_sigxy,i)
---
>cond(1:3,1:3) = -cond(1:3,1:3)*deltaef &
> * OHM *METER*SECOND
>nu(1:3,1:3)   =   -nu(1:3,1:3)*deltaef/temp &
> * METER * SECOND / AMPERE
>kappa(1:3,1:3)=-kappa(1:3,1:3)*deltaef/temp &
> * METER * SECOND / WATT
>sigxyz(1:3,1:3,1:3)=-sigxyz*deltaef &
> * OHM**2 * SECOND**3 * AMPERE / METER
>specheat  = specheat*deltaef & 
> * MOL / JOULE
> !TB 2014: reduce halltensor to 2D
> !changed below invert_3x3 to invert_2x2
> minv_sigxy(1:2,1:2)=cond(1:2,1:2)
> CALL invert_2x2(minv_sigxy,minv_sigxy,i)
321,323c346,350
< DO i=1,3
<DO j=1,3
<   DO ialp=1,3
---
> !TB 2014: reduce seebeck to 2D
> ! changed below everything from 1,3 to 1,2
> DO i=1,2
>DO j=1,2
>   DO ialp=1,2
330,334c357,366
< DO i=1,3
<DO j=1,3
<   DO k=1,3
<  DO ialp=1,3
< DO ibet=1,3
---
> !TB 2014: reduce halltensor to 2D
> ! changed below k FROM going from 1 to 3 TO 3
> ! i,j from 1,3 to 1,2
> ! ialp,ibet from 1,3 to 1,2
> !WRITE(666,*)efermi/RYDBERG,minv_sigxy(1:2,1:2), lifetime
> DO i=1,2
>DO j=1,2
>   DO k=3,3
>  DO ialp=1,2
> DO ibet=1,2
85a86,102
> !TB 2014
> SUBROUTINE invert_2x2(mat1,inv_mat1,idet)
>   USE defs
>   IMPLICIT NONE
>   REAL(8):: mat1(2,2),inv_mat1(2,2),det,determ1
>   INTEGER:: idet
>   inv_mat1(1,1)=mat1(2,2) 
>   inv_mat1(1,2)=-mat1(1,2) 
>   inv_mat1(2,1)=-mat1(2,1) 
>   inv_mat1(2,2)=mat1(1,1) 
>   det=determ1(mat1)
>   idet=1
>   IF(det < test) idet=0
>   inv_mat1(1:2,1:2)=inv_mat1(1:2,1:2)/det
> END SUBROUTINE invert_2x2
> !TB 2014
> 
104a122,130
> 
> !TB 2014
> REAL(8) FUNCTION determ1(a)
>   ! return the determinante of a 2x2 matrix
>   IMPLICIT NONE
>   REAL(8)  :: a(2,2)
>   determ1=a(1,1)*a(2,2)-a(1,2)*a(2,1)
> END FUNCTION determ1
> !TB 2014
___
users mailing list
users@lists.quantum-espresso.org
https://lists.quantum-espresso.org/mailman/listinfo/users

Re: [QE-users] QE_2_BoltzTrap

2019-01-03 Thread Dr. Thomas Brumme

Dear Mohammad,

first of all: if you have problems/questions regarding BoltzTraP it might be a
good idea to ask the authors or in their google users group. See also the
web page of the new BoltzTraP2 program:

https://www.imc.tuwien.ac.at/index.php?id=21094

For the specific question using BoltzTraP for 2D materials:
I used it and remember that one had to change some lines for the call of the
calculation of, e.g., the conductivity tensor otherwise the results  
were wrong.
I can't check thoroughly at the moment as I'm still on vacation and  
can't switch

on the hard drive remotely which contains the data.

You'll have to search the respective calls for the calculation of the  
different

tensors by yourself or wait till next week.

Regards

Thomas


Zitat von Ubaid Mohd :


Dear users,
 Is there any need to change the any *.F90 file in the BoltzTrap code to
make the code for 2D materials, if yes, which file to be edit for using the
code in 2D materials?
Thanks and regards

*Mohammad Ubaid*
*PhD Research Scholar*
*Department of Physics*
*Jamia Millia Islamia University*
*New Delhi - 110025*


On Thu, Dec 27, 2018 at 10:48 AM Ubaid Mohd  wrote:


Hi sir,
 I tried with
" print(nsym)
except:
nsym=1"
after code " i += 1 " in line 115.
But still I am getting the error;
  File "./qe2boltz.py", line 117
print(nsym)
^
IndentationError: unindent does not match any outer indentation level
Here is my edited script and nscf output. Please have a look.
Thanks and Regards
*Mohammad Ubaid*
*PhD Research Scholar*
*Department of Physics*
*Jamia Millia Islamia University*
*New Delhi - 110025*


On Wed, Dec 26, 2018 at 5:36 PM Ubaid Mohd  wrote:


I am not able to fix because I don't know python. Here is my script, so
could you please fix that and send the script.
 Regards
*Mohammad Ubaid*
*PhD Research Scholar*
*Department of Physics*
*Jamia Millia Islamia University*
*New Delhi - 110025*


On Wed, Dec 26, 2018 at 5:28 PM DongKyu Lee 
wrote:


Dear Ubaid Mohd,

The python script can not find symmetry-information when "nosym =
.true.".
It will be fixed by adding code to handle error exception.
I solved the problem as adding
"
try:
print(nsym)
except:
nsym=1
"
after code " i += 1 " in line 115.

Sincerely,

/


Dongkyu Lee  / Master Candidate
Computational Many-Body Physics Group
Department of Physics and Photon Science
Gwangju Institute of Science and Technology [GIST]
Gwangju 61005, South Korea

Office.  +82-62-715-2855
Mobile. +82-10-5000-0016
Email. physdk...@gist.ac.kr


**/

--

Message: 4
Date: Wed, 26 Dec 2018 13:42:08 +0530
From: Ubaid Mohd 
To: users@lists.quantum-espresso.org
Subject: [QE-users] QE_2_BoltzTrap
Message-ID:
<
cacgvgwgbqb7zfv+ihuhehjupvznmz0vc-epovpzgy+xqsea...@mail.gmail.com>
Content-Type: text/plain; charset="utf-8"

Dear users,
  I am using BoltzTrap for thermoelectric properties. I am following the
steps like  scf and nscf calculation, then using the script qe2boltz.py
to
convert the data for BoltzTrap. Some times script converts the data
without
any error and some times I got this error:
Traceback (most recent call last):
File "/pkg/chem/boltztrap125/util/qe2boltz.py", line 253, in 
sys.exit(main())
File "/pkg/chem/boltztrap125/util/qe2boltz.py", line 130, in main
for ir in range(nsym):
UnboundLocalError: local variable 'nsym' referenced before assignment.
 I am unable to sort out the error but I can say it may be due nscf
input.
So please guide me how to sort out this error?
Thanks and regards
*Mohammad Ubaid*
*PhD Research Scholar*
*Department of Physics*
*Jamia Millia Islamia University*
*New Delhi - 110025*
___
users mailing list
users@lists.quantum-espresso.org
https://lists.quantum-espresso.org/mailman/listinfo/users





--
Dr. rer. nat. Thomas Brumme
Wilhelm-Ostwald-Institute for Physical and Theoretical Chemistry
Leipzig University
Phillipp-Rosenthal-Strasse 31
04103 Leipzig

Tel:  +49 (0)341 97 36456

email: thomas.bru...@uni-leipzig.de

___
users mailing list
users@lists.quantum-espresso.org
https://lists.quantum-espresso.org/mailman/listinfo/users


Re: [QE-users] read ions position from previous runs

2018-12-16 Thread Dr. Thomas Brumme
One can use restart_mode='restart' together with startingpot='atomic'  
and startingwfc='atomic+random'... This will trick the code to use the  
positions but starting with the standard starting potential and wave  
functions. But be careful: the code also reads in other variables such  
as electric field or similar... Give it a try :)


Thomas

Zitat von Paolo Giannozzi :


There is a variable "ion_positions" that could be used for the purpose you
mention, but this is not currently implemented

Paolo

On Sat, Dec 15, 2018 at 10:05 PM Shenli Zhang  wrote:


Dear QE users,

I'm wondering if there is a way to just import atom's positions from a
previous run without rewriting the charge density and wavefunction
information at the same time?

Thank you so much!
--
Shenli Zhang

Graduate Student of Materials Science
University of California, Davis
___
users mailing list
users@lists.quantum-espresso.org
https://lists.quantum-espresso.org/mailman/listinfo/users




--
Paolo Giannozzi, Dip. Scienze Matematiche Informatiche e Fisiche,
Univ. Udine, via delle Scienze 208, 33100 Udine, Italy
Phone +39-0432-558216, fax +39-0432-558222




___
users mailing list
users@lists.quantum-espresso.org
https://lists.quantum-espresso.org/mailman/listinfo/users


Re: [QE-users] Superposition of 2Pz orbital's states

2018-12-12 Thread Thomas Brumme

Dear Gui Wei,

why do you need to use sumpdos.x? You can also use other tools. In fact, 
you only want to sum the same column
of many different files, right? Google helps. Something like this should 
work:


https://unix.stackexchange.com/questions/182758/sum-two-columns-from-different-files

So, you can use:

paste file1 file2 | awk '{ print $3 + $13; }'

but doing this for 64 files might be hard. What about:

https://askubuntu.com/questions/963011/adding-columns-from-multiple-files-with-numbered-names

Thus, something like this:

awk '{a[FNR]=$1;b[FNR]+=$2;c[FNR]+=$3;d[FNR]+=$4;e[FNR]+=$5} END{for 
(i=1;i<=FNR;i++) print a[i], b[i], c[i], d[i], e[i]}' 
C.pdos_atm#{1..64}(\C\)_wfc#2\(p\) >> total_C_pdos.dat


Now you have in the 3rd column of total_C_pdos.dat the total pz pDOS.

Regards

Thomas


On 12/12/18 14:40, IORI, Federico wrote:

To me this information is not directly given in QE.
Or you change your basis set for a localised basis set (not QE code) 
or you try a wannier expansion in case


https://www.quantum-espresso.org/Doc/pp_user_guide/node8.html

http://ermes.unt.edu/Tutorial_Proj.pdf



On Wed, Dec 12, 2018 at 2:23 PM Gui Wei <201707021...@cqu.edu.cn 
<mailto:201707021...@cqu.edu.cn>> wrote:


Hi,
Thanks for your reply.When I use your method,I got the same result
as before.That is two columns of data:E (eV)  pdos(E),Where
pdos(E)contains the sum of three items:Pz,Px,Py.But my purpose is
to only sum the Pz orbital's states of each atom.

Gui Wei
School of Mechanical Engineering,Chongqing University, China


-原始邮件-
*发件人:*"IORI, Federico" mailto:federico.i...@airliquide.com>>
*发送时间:*2018-12-12 18:18:06 (星期三)
*收件人:* users@lists.quantum-espresso.org
<mailto:users@lists.quantum-espresso.org>
*抄送:*
*主题:* Re: [QE-users] Superposition of 2Pz orbital's states

Hi.

My quick and dirty homemade way

ls -1 pdos_atm* > list.dat
grep "(p" list.dat > list_2p.dat
sumpdos.x -f list_2p.dat > pdos_2p.dat

HTH

ff

On Wed, Dec 12, 2018 at 10:39 AM Gui Wei
<201707021...@cqu.edu.cn <mailto:201707021...@cqu.edu.cn>> wrote:

Hi,
When I sum the pdos for 2p orbitals of 64 carbon atoms
with sumpdos.x : ./sumpdos.x C.pdos_atm#1(\C\)_wfc#2\(p\)
C.pdos_atm#2(\C\)_wfc#2\(p\)……C.pdos_atm#64(\C\)_wfc#2\(p\).I
obtain the result that contains only two columns of
data:E(eV)and pdos(E) rather than four columns:E(eV)
pdos(E) pdos(E) pdos(E).But what I care about is the
result of superposition of 2Pz orbital's states.Is there
any easy way to quickly count the data I want?


Gui Wei
School of Mechanical Engineering,Chongqing University, China
___
users mailing list
users@lists.quantum-espresso.org
<mailto:users@lists.quantum-espresso.org>
https://lists.quantum-espresso.org/mailman/listinfo/users



-- 
Federico IORI


Computational material scientist

Paris-Saclay Research Center

1 chemin de la Porte des Loges

<https://www.google.com/maps/place/Air+Liquide/@48.8297381,2.2016685,11.75z/data=%214m5%213m4%211s0x47e67e61a4fbbdc7:0xca3bea9e80059880%218m2%213d48.7630415%214d2.1333045>
Les Loges en Josas – 78354 Jouy en Josas cedex Mail:
federico.i...@airliquide.com
<mailto:federico.i...@airliquide.com>

Phone: +33 7 621 605 15


___
users mailing list
users@lists.quantum-espresso.org
<mailto:users@lists.quantum-espresso.org>
https://lists.quantum-espresso.org/mailman/listinfo/users



--
Federico IORI

Computational material scientist

Paris-Saclay Research Center

1 chemin de la Porte des Loges 
<https://www.google.com/maps/place/Air+Liquide/@48.8297381,2.2016685,11.75z/data=%214m5%213m4%211s0x47e67e61a4fbbdc7:0xca3bea9e80059880%218m2%213d48.7630415%214d2.1333045> 
Les Loges en Josas – 78354 Jouy en Josas cedex Mail: 
federico.i...@airliquide.com <mailto:federico.i...@airliquide.com>


Phone: +33 7 621 605 15




___
users mailing list
users@lists.quantum-espresso.org
https://lists.quantum-espresso.org/mailman/listinfo/users


--
Dr. rer. nat. Thomas Brumme
Wilhelm-Ostwald-Institute for Physical and Theoretical Chemistry
Leipzig University
Phillipp-Rosenthal-Strasse 31
04103 Leipzig

Tel:  +49 (0)341 97 36456

email: thomas.bru...@uni-leipzig.de

___
users mailing list
users@lists.quantum-espresso.org
https://lists.quantum-espresso.org/mailman/listinfo/users

Re: [QE-users] shared library error

2018-12-04 Thread Thomas Brumme

Dear Haider Abbas,

do all the nodes have access to the same filesystem? I remember getting 
such (or similar) errors
when I wanted to run a job on nodes with separate filesystem... In this 
case be sure that each node
has access to the temporary directory. See also input variables outdir 
and wfcdir


Regards

Thomas Brumme


P.S.: Please add your affiliation.
See also the posting guidelines here: 
http://www.quantum-espresso.org/forum#1.0



On 12/04/18 09:41, Haider Abbas wrote:


Dear all,

I have prepared a 10 node cluster (each having 4 core). I have 
installed GAMESS and QUANTUM ESPRESSO. Gamess is running fine, but 
when I run quantum espresso with the command


mpiexec -n 4 ./pw.x -i relax2.in <http://relax2.in> > out.out &

I get no error and everything is fine. And the job run on all 4 core a 
single node.


But when I run the same job on the 10 node with the command

mpiexec -f hosts -n 40 ./pw.x -i relax2.in <http://relax2.in> > out.out &

I get the error


mpiuser@master:~/qe-29-11/bin$ ./pw.x: error while loading shared 
libraries: liblapack.so.3: cannot open shared object file: No such 
file or directory


./pw.x: error while loading shared libraries: liblapack.so.3: cannot 
open shared object file: No such file or directory


./pw.x: error while loading shared libraries: liblapack.so.3: cannot 
open shared object file: No such file or directory




However, I have "liblapack.so.3" in the same directory from where I am 
running the job.


I sought your help.
Thanking in advance.
with regards


___
users mailing list
users@lists.quantum-espresso.org
https://lists.quantum-espresso.org/mailman/listinfo/users


--
Dr. rer. nat. Thomas Brumme
Wilhelm-Ostwald-Institute for Physical and Theoretical Chemistry
Leipzig University
Phillipp-Rosenthal-Strasse 31
04103 Leipzig

Tel:  +49 (0)341 97 36456

email: thomas.bru...@uni-leipzig.de

___
users mailing list
users@lists.quantum-espresso.org
https://lists.quantum-espresso.org/mailman/listinfo/users

Re: [QE-users] dipfield

2018-11-13 Thread Dr. Thomas Brumme
Both methods should give the same result - they did at least when I  
tried. But you can't calculate phonons with dipole correction so if  
you want those start with assume_isolated = '2D'
But read all infos given in input_pw and the related papers - it helps  
to understand how to setup the system ;)


Thomas

Zitat von Asad Mahmood :


Thank you Thomas, I will try both flag one by one and see what change do
each of them brings separately.

On Tue, Nov 13, 2018 at 3:52 PM Thomas Brumme 
wrote:


Dear Asad,

please also answer to the list ;) ("reply all")

No! It's either dipfield = .true. or assume_isolated ='2d'. If you want to
simulate "a real" 2D system using
the Coulomb cutoff you don't need to correct due to the systems dipole.
And if you correct with the artificial
dipole, cutting of the Coulomb will result in something bad/weird...
Actually, I don't know if this will work at all.

So, either dipfield = .true. or assume_isolated ='2d'

Thomas

On 11/13/18 11:45, Asad Mahmood wrote:

Thanks Thomas for reply. So I need to include both flags(i.e dipfielf =
.true and assume_isolated ='2d' ) in relax or structure optimization
calcations ?

On Tue, Nov 13, 2018, 3:11 PM Thomas Brumme 
Dear Asad,

What is the benefit of using dipfield=.true.? Well, the benefit is that
you get the correct electronic structure,
energetics, and so on. As you want to simulate a 2D system with 3D
boundary conditions this is the only
correct way if you have an asymmetric system (in the z-direction,
assuming the 2D system is in xy). Well,
not the only correct way. Recently a Coulomb cutoff technique was
implemented in QE, check input flag
assume_isolated='2D'

Regards

Thomas

P.S.: If you have a dipole in your system you'll see that properties
(especially the total energy) will change
with increasing vacuum region. If you increase it, you'll minimize the
eigenfunction overlap between the
repeated layers but the Coulomb interaction will never be zero and thus
you can get wrong results - and
this will be worse for larger dipole, e.g., the flourinated graphene.

On 11/13/18 08:38, Asad Mahmood wrote:

Hi,

I have a hexagonal graphene(or any other) monolayer and I hydrogenate(or
flourinate, etc.) on the top surface only (single sided). Is there need to
use flag dipfield = .true ? What is benefit or disadvantage of using this
flag for optimization of L.C or bond length?
I am using 20 angstrom vacuum between two layers to avoid mutual
influence.


___
users mailing  
listusers@lists.quantum-espresso.orghttps://lists.quantum-espresso.org/mailman/listinfo/users



--
Dr. rer. nat. Thomas Brumme
Wilhelm-Ostwald-Institute for Physical and Theoretical Chemistry
Leipzig University
Phillipp-Rosenthal-Strasse 31
04103 Leipzig

Tel:  +49 (0)341 97 36456

email: thomas.bru...@uni-leipzig.de



--
Dr. rer. nat. Thomas Brumme
Wilhelm-Ostwald-Institute for Physical and Theoretical Chemistry
Leipzig University
Phillipp-Rosenthal-Strasse 31
04103 Leipzig

Tel:  +49 (0)341 97 36456

email: thomas.bru...@uni-leipzig.de






___
users mailing list
users@lists.quantum-espresso.org
https://lists.quantum-espresso.org/mailman/listinfo/users


Re: [QE-users] dipfield

2018-11-13 Thread Thomas Brumme

Dear Asad,

please also answer to the list ;) ("reply all")

No! It's either dipfield = .true. or assume_isolated ='2d'. If you want 
to simulate "a real" 2D system using
the Coulomb cutoff you don't need to correct due to the systems dipole. 
And if you correct with the artificial
dipole, cutting of the Coulomb will result in something bad/weird... 
Actually, I don't know if this will work at all.


So, either dipfield = .true. or assume_isolated ='2d'

Thomas


On 11/13/18 11:45, Asad Mahmood wrote:
Thanks Thomas for reply. So I need to include both flags(i.e dipfielf 
= .true and assume_isolated ='2d' ) in relax or structure optimization 
calcations ?


On Tue, Nov 13, 2018, 3:11 PM Thomas Brumme 
mailto:thomas.bru...@uni-leipzig.de> wrote:


Dear Asad,

What is the benefit of using dipfield=.true.? Well, the benefit is
that you get the correct electronic structure,
energetics, and so on. As you want to simulate a 2D system with 3D
boundary conditions this is the only
correct way if you have an asymmetric system (in the z-direction,
assuming the 2D system is in xy). Well,
not the only correct way. Recently a Coulomb cutoff technique was
implemented in QE, check input flag
assume_isolated='2D'

Regards

Thomas

P.S.: If you have a dipole in your system you'll see that
properties (especially the total energy) will change
with increasing vacuum region. If you increase it, you'll minimize
the eigenfunction overlap between the
repeated layers but the Coulomb interaction will never be zero and
thus you can get wrong results - and
this will be worse for larger dipole, e.g., the flourinated graphene.

On 11/13/18 08:38, Asad Mahmood wrote:

Hi,

I have a hexagonal graphene(or any other) monolayer and I
hydrogenate(or flourinate, etc.) on the top surface only (single
sided). Is there need to use flag dipfield = .true ? What is
benefit or disadvantage of using this flag for optimization of
L.C or bond length?
I am using 20 angstrom vacuum between two layers to avoid mutual
influence.


___
users mailing list
users@lists.quantum-espresso.org
<mailto:users@lists.quantum-espresso.org>
https://lists.quantum-espresso.org/mailman/listinfo/users


    -- 
    Dr. rer. nat. Thomas Brumme

Wilhelm-Ostwald-Institute for Physical and Theoretical Chemistry
Leipzig University
Phillipp-Rosenthal-Strasse 31
04103 Leipzig

Tel:  +49 (0)341 97 36456

email:thomas.bru...@uni-leipzig.de <mailto:thomas.bru...@uni-leipzig.de>



--
Dr. rer. nat. Thomas Brumme
Wilhelm-Ostwald-Institute for Physical and Theoretical Chemistry
Leipzig University
Phillipp-Rosenthal-Strasse 31
04103 Leipzig

Tel:  +49 (0)341 97 36456

email: thomas.bru...@uni-leipzig.de

___
users mailing list
users@lists.quantum-espresso.org
https://lists.quantum-espresso.org/mailman/listinfo/users

Re: [QE-users] dipfield

2018-11-13 Thread Thomas Brumme

Dear Asad,

What is the benefit of using dipfield=.true.? Well, the benefit is that 
you get the correct electronic structure,
energetics, and so on. As you want to simulate a 2D system with 3D 
boundary conditions this is the only
correct way if you have an asymmetric system (in the z-direction, 
assuming the 2D system is in xy). Well,
not the only correct way. Recently a Coulomb cutoff technique was 
implemented in QE, check input flag

assume_isolated='2D'

Regards

Thomas

P.S.: If you have a dipole in your system you'll see that properties 
(especially the total energy) will change
with increasing vacuum region. If you increase it, you'll minimize the 
eigenfunction overlap between the
repeated layers but the Coulomb interaction will never be zero and thus 
you can get wrong results - and

this will be worse for larger dipole, e.g., the flourinated graphene.

On 11/13/18 08:38, Asad Mahmood wrote:

Hi,

I have a hexagonal graphene(or any other) monolayer and I 
hydrogenate(or flourinate, etc.) on the top surface only (single 
sided). Is there need to use flag dipfield = .true ? What is benefit 
or disadvantage of using this flag for optimization of L.C or bond length?
I am using 20 angstrom vacuum between two layers to avoid mutual 
influence.



___
users mailing list
users@lists.quantum-espresso.org
https://lists.quantum-espresso.org/mailman/listinfo/users


--
Dr. rer. nat. Thomas Brumme
Wilhelm-Ostwald-Institute for Physical and Theoretical Chemistry
Leipzig University
Phillipp-Rosenthal-Strasse 31
04103 Leipzig

Tel:  +49 (0)341 97 36456

email: thomas.bru...@uni-leipzig.de

___
users mailing list
users@lists.quantum-espresso.org
https://lists.quantum-espresso.org/mailman/listinfo/users

Re: [QE-users] Error in routine read_namelists (5010):, reading namelist system

2018-11-01 Thread Thomas Brumme

Dear Maxim,

please answer also to the list as others also might be able to help.
Maybe even better than me. Space group 33 is not implemented.
If you check the file 'Modules/wypos.f90' you'll see that the case
33 is missing. I have no idea why, but maybe there is an equivalent
space group? Another simple solution is to use the tools of the
Bilbao side:

http://www.cryst.ehu.es/cgi-bin/cryst/programs/mcif2vesta/index.php

Upload your cif and get the VASP output which you can easily use
as an input for QE.

Thomas


On 11/01/18 18:09, Максим Арсентьев wrote:

Dear Thomas,

I tried PWSCF v.5.4.0 and now the error is:

     Error in routine wypos (1):
     group not recognized

--
Best wishes,
Maxim Arsent'ev, Ph.D. (Chemistry)
Laboratory of research of nanostructures
Institute of Silicate Chemistry of RAS



--
Dr. rer. nat. Thomas Brumme
Wilhelm-Ostwald-Institute for Physical and Theoretical Chemistry
Leipzig University
Phillipp-Rosenthal-Strasse 31
04103 Leipzig

Tel:  +49 (0)341 97 36456

email: thomas.bru...@uni-leipzig.de

___
users mailing list
users@lists.quantum-espresso.org
https://lists.quantum-espresso.org/mailman/listinfo/users

Re: [QE-users] Error in routine read_namelists (5010):, reading namelist system

2018-11-01 Thread Thomas Brumme

Dear Maxim,

the option to enter the coordinates using the space group and Wyckoff 
positions
was introduced in version 5.1.1 - so, you either need to update QE or 
use a different

format for your coordinates input.

Regards

Thomas


On 11/01/18 14:49, Максим Арсентьев wrote:

Dear Thomas,

Thank you for the response, my version is PWSCF v.5.0.2 (svn rev. 9656).
I changed Wyckoff position into "4a" and celldm as you said, but still 
the same problem(


--
Best wishes,
Maxim Arsent'ev, Ph.D. (Chemistry)
Laboratory of research of nanostructures
Institute of Silicate Chemistry of RAS


--
Dr. rer. nat. Thomas Brumme
Wilhelm-Ostwald-Institute for Physical and Theoretical Chemistry
Leipzig University
Phillipp-Rosenthal-Strasse 31
04103 Leipzig

Tel:  +49 (0)341 97 36456

email: thomas.bru...@uni-leipzig.de

___
users mailing list
users@lists.quantum-espresso.org
https://lists.quantum-espresso.org/mailman/listinfo/users

Re: [QE-users] Error in routine read_namelists (5010):, reading namelist system

2018-11-01 Thread Thomas Brumme

Dear Maxim,

space group 33 does not have a Wyckoff position 4d but only 4a.
Furthermore, we would need to know the version of QE you're
using. Additionally, if you set

    A=24.98444525,
    B=0.588411037,
    C=1.372938916,

it looks like you actually want to use celldm(1) ... celldm(3) or is
your lattice really just 0.588 Angstrom in the y direction ;)

Cheerio

Thomas


On 11/01/18 13:11, Maxim Arsentev wrote:


Dear  QE users,

When I run my input with "space_group" variable activated, I got whis 
error:


     Error in routine  read_namelists (5010):
      reading namelist system

--
Best wishes,
Maxim Arsent'ev, Ph.D. (Chemistry)
Laboratory of research of nanostructures
Institute of Silicate Chemistry of RAS


___
users mailing list
users@lists.quantum-espresso.org
https://lists.quantum-espresso.org/mailman/listinfo/users


--
Dr. rer. nat. Thomas Brumme
Wilhelm-Ostwald-Institute for Physical and Theoretical Chemistry
Leipzig University
Phillipp-Rosenthal-Strasse 31
04103 Leipzig

Tel:  +49 (0)341 97 36456

email: thomas.bru...@uni-leipzig.de

___
users mailing list
users@lists.quantum-espresso.org
https://lists.quantum-espresso.org/mailman/listinfo/users

Re: [QE-users] Gamma_only case not implemented CRASH

2018-10-10 Thread Dr. Thomas Brumme

Dear Lucas,

please have a look at the input description for the k-points list
and then look at your input for the bands calculation. I guess the
code assumes 0 lines for the bands as the first number in the first
line is a "0".
If you want a path along (G M K G) you should have something like:

K_POINTS crystal_b
4
0.00.00.0 60
0.50.00.0 35
0. 0. 0.0 69
0.00.00.0 1


Cheerio

Thomas

--
Dr. rer. nat. Thomas Brumme
Wilhelm-Ostwald-Institute for Physical and Theoretical Chemistry
Leipzig University
Phillipp-Rosenthal-Strasse 31
04103 Leipzig

Tel:  +49 (0)341 97 36456

email: thomas.bru...@uni-leipzig.de


Zitat von Lucas Dória :


Paolo, thanks for replying.


v.5.4.0.

Regards.
Lucas.


De: users  em nome de  
Paolo Giannozzi 

Enviado: quarta-feira, 10 de outubro de 2018 09:30
Para: Quantum Espresso users Forum
Assunto: Re: [QE-users] Gamma_only case not implemented CRASH

QE version?

On Wed, Oct 10, 2018 at 2:01 PM Lucas Dória  
mailto:lucasdoriacarva...@hotmail.com>>  
wrote:


Thanks for your reply, Lorenzo.

The first round of calculations is an scf calculation where I use an  
automatic grid of k-points:




celldm(1) =  6.04253156,
degauss   =  1.0e-02,
ecutrho   =  148.,
ecutwfc   =  37.,
ibrav = 0,
nat   = 7,
nspin = 2,
ntyp  = 3,
occupations   = 'smearing',
smearing  = 'gaussian',
/



conv_thr = 1.0e-10,
electron_maxstep = 200,
mixing_beta  = 0.7000,
diagonalization  = 'cg',
startingpot  = 'atomic',
startingwfc  = 'atomic+random',
/

K_POINTS {automatic}
7 7 1 0 0 0

The second one is a band calculation with the k-path calculated with  
XCrysDen (Gg M K Gg):



celldm(1) = 6.04253156,
degauss   =  1.0e-02,
ecutrho   =  148.,
ecutwfc   =  37.,
ibrav = 0,
nat   = 7,
nspin = 2,
ntyp  = 3,
occupations   = 'smearing',
smearing  = 'gaussian',
/


diagonalization  = 'cg',
conv_thr =  1.0e-08,
electron_maxstep =  200,
mixing_beta  =  4.0e-01,
startingpot  = 'atomic',
startingwfc  = 'atomic+random',
/

KPOINTS {tpiba_b}
0.00 0.00 0.00 1 1 1
0.50 0.00 0.00 6 1 1
0.33 0.33 0.00 6 6 1
0.00 0.00 0.00 1 1 1


The third and last one is just an input for bands.x:



filband = 'bands.dat',
lsym= .false.,
prefix  = 'bands_F',
outdir  = '/tmp/scratch/ldoria/saida',
/


Best regards.
Lucas.


De: users  
mailto:users-boun...@lists.quantum-espresso.org>> em nome de Lorenzo Paulatto  
mailto:paul...@gmail.com>>

Enviado: quarta-feira, 10 de outubro de 2018 04:39
Para:  
users@lists.quantum-espresso.org<mailto:users@lists.quantum-espresso.org>

Assunto: Re: [QE-users] Gamma_only case not implemented CRASH

Dear Lucas,
please provide you input example (to the list, not to me) because:

1. running bands.x after a calculation at the gamma point only does not
make sense

-or-

2. if you did SCF with gamma only, NSCF with more k-points (which is
questionable but legit) and bands.x afterwards, then this error should
not occur, may be a bug

kind regards



--
Lorenzo Paulatto - Paris
___
users mailing list
users@lists.quantum-espresso.org<mailto:users@lists.quantum-espresso.org>
https://lists.quantum-espresso.org/mailman/listinfo/users
users Info Page -  
lists.quantum-espresso.org<https://lists.quantum-espresso.org/mailman/listinfo/users>

lists.quantum-espresso.org<http://lists.quantum-espresso.org>
This is the mailing list for discussions about the Quantum ESPRESSO  
distribution. Only registered users can post. To see the collection  
of prior postings to the list, visit the users Archives.. Using users



___
users mailing list
users@lists.quantum-espresso.org<mailto:users@lists.quantum-espresso.org>
https://lists.quantum-espresso.org/mailman/listinfo/users


--
Paolo Giannozzi, Dip. Scienze Matematiche Informatiche e Fisiche,
Univ. Udine, via delle Scienze 208, 33100 Udine, Italy
Phone +39-0432-558216, fax +39-0432-558222




___
users mailing list
users@lists.quantum-espresso.org
https://lists.quantum-espresso.org/mailman/listinfo/users

Re: [QE-users] Projected DOS with Spin-Orbit Coupling

2018-10-03 Thread Dr. Thomas Brumme

Dear Asad,

about the order I'm not sure, but with SOC you cannot think of px or  
py or dxy anymore.

For more details, see e.g. this paper:

https://journals.aps.org/prb/abstract/10.1103/PhysRevB.71.115106

working through the math you'll see that every, e.g., d-state can and  
will couple to
a lot or even all other d-states... With SOC, there is nothing like px  
or py. You can
however plot the eigenfunction itself and then have a look whether it  
resembles such

a state.

Regards

Thomas


--
Dr. rer. nat. Thomas Brumme
Wilhelm-Ostwald-Institute for Physical and Theoretical Chemistry
Leipzig University
Phillipp-Rosenthal-Strasse 31
04103 Leipzig

Zitat von Asad Mahmood :


Hi all,
I did scf calculation (using pw.x) with flags lspinorb = .true and noncolin
= .true.
Then I run projwfc.x (for pdos). Following files were generated:
1) *pdos_atom#1(C)_wfc#1(s)*
2)* pdos_atom#1(C)_wfc#1(s_j0.5)*
3) *pdos_atom#1(C)_wfc#2(p)*
4)* pdos_atom#1(C)_wfc#2(p_j0.5)*
5) *pdos_atom#1(C)_wfc#3(p_j1.5)*
where #4 file had 4 columns which were shown as:

*# E(eV)   ldos(E)   pdos(E)_1   pdos(E)_2 *
and #5 file had 6 columns as shown below:

*# E(eV)   ldos(E)   pdos(E)_1   pdos(E)_2   pdos(E)_3   pdos(E)_4  *

*Q#1: *Can someone explain what does these suffix (_1, _2, _3, etc) mean?
Clearly , j1.5 and j0.5 are due to L+/S and L-S respectively.
*Q#2:* How can we think of px, py and pz from these new labelled files?

Regards,
Asad Mahmood,
Research Assistant
Department of Physics
Q.A.U, Islamabad,
Pakistan




___
users mailing list
users@lists.quantum-espresso.org
https://lists.quantum-espresso.org/mailman/listinfo/users


Re: [QE-users] Difficulty with convergence using tefield

2018-09-30 Thread Dr. Thomas Brumme

Dear Alex,

if your is non-periodic in one direction (with the vacuum and the
artificial dip due to the field and dipole correction) there is no
reason to put k-points in this direction - you just increase the
computation time to get bands with no dispersion at all in this
direction. At least I think that there shouldn't be any dispersion,
otherwise something is wrong... The system is only periodic in 2D.

Anyway, another info: the main parallelization scheme in QE is
over the points in real space in z direction. So, it's always
good to have the largest unit cell dimension along z.

Cheers

Thomas


--
Dr. rer. nat. Thomas Brumme
Wilhelm-Ostwald-Institute for Physical and Theoretical Chemistry
Leipzig University
Phillipp-Rosenthal-Strasse 31
04103 Leipzig

Tel:  +49 (0)341 97 36456

email: thomas.bru...@uni-leipzig.de

Zitat von "Alex.Durie" :


Dear Lorenzo,


Thank you for your response, yes I got convergence without the field  
in about 23 iterations, but the estimated error was much better at  
the earlier iterations than with the field.



I take it from your response you believe the issue lies with the  
convergence? My k-point grid was defined so that the mesh was  
equally spaced in reciprocal space, however I understand the system  
is highly inhomogeneous in the field direction so I will try  
something like 16x16x16.



I understand what you say about edir, I believe also that the system  
runs more efficiently with the magnetisation along the z-axis. Do  
you think overall the code is more efficient with say edir = 3,


nspin = 4, angle1(1) = 90, angle2(1) = 0? Provided that's the  
correct way to set magnetisation along the x-axis?


Many thanks,

Alex


Subject: Re: [QE-users] Difficulty with convergence using tefield

Message-ID:

Content-Type: text/plain; charset="utf-8"

edir=1 and 2 are extremely inefficient in parallel. Use edir=3, rotate your
slab. Does it converge without field? Also, your choic is a of k-points
looks the opposite of correct: you need many points along the field
direction.

--
Lorenzo Paulatto
Written on a virtual keyboard with real fingers

On Sat, 29 Sep 2018, 20:43 Alex.Durie,  wrote:


Dear experts,

I am attempting to perform an scf calculation on a semi-infinite slab of
ferromagnetic cobalt surrounded by a vacuum with an electric field applied
perpendicular to the magnetisation, out-of-plane.

I am struggling to get convergence in the scf cycle. I achieved
satisfactory results using the same geometry and k-points without the field.

I have attempted to run it with ecutwfc = 30, and ecutrho at its default
value. Now I am attempting ecutwfc = 46, ecutrho = 238 (as recommended in
the header of the pseudopotential file).

I find in both cases the total energy and Harris-Foulkes estimate are
usually negative, and the estimated scf accuracy is very large, for each
cycle.

The runtime for larger ecutwfc and ecutrho, is much longer.

Is there something I'm doing wrong, or do I need to alter my expectations
of runtime? Please see input file below.

I am running the code with MPI, on 4 cores. At 12 iterations, the CPU time
is 24737.9 secs, the total energy is -377.46347734 Ry, the Harris-Foulkes
estimate is -1293.19368730 Ry and the estimated scf accuracy is <
3322.28583637 Ry. I have also noticed the "Adding external electric field"
block is duplicated on each cycle, though I suspect that's normal.

Many thanks

Alex Durie
PhD student
The Open University
United Kingdom


calculation='scf'
restart_mode='from_scratch',
pseudo_dir = '/home/alex/QE/pslibrary.1.0.0/pz/PSEUDOPOTENTIALS/',
outdir='./'
prefix='co'
tefield = .true.
dipfield = .true.
/

ibrav = 8,
celldm(1) =4.82388,
celldm(2) =8.,
celldm(3) =1.,
nat= 5,
ntyp= 1,
ecutwfc = 46.0
ecutrho = 238.0
nspin = 2
occupations='smearing', smearing='cold', degauss=0.02
starting_magnetization = -1
nbnd = 45
edir = 2
emaxpos = 0.41
eopreg = 0.59
eamp = 0.000243
/

electron_maxstep = 500
diagonalization='cg'
conv_thr = 1.0e-6
mixing_beta = 0.3
/
ATOMIC_SPECIES
Co 58.933195 Co.pz-n-kjpaw_psl.1.0.0.UPF
ATOMIC_POSITIONS {bohr}
Co   0.000  0.000  0.00
Co   2.4119400  3.411  2.411940
Co   0.000  6.822  0.00
Co   2.4119400 10.233  2.411940
Co   0.000 13.644  0.00
K_POINTS {automatic}
16 2 16  0 0 0




___
users mailing list
users@lists.quantum-espresso.org
https://lists.quantum-espresso.org/mailman/listinfo/users


Re: [QE-users] Relax Calculation anomalous result

2018-09-17 Thread Thomas Brumme

Hi Asad,

did you consider to include a dipole correction? Or maybe using the new 
2D cutoff technique?
I think that this might be important for a 2D system in which you only 
hydrogenated one side.


Regards

Thomas

P.S.: Please add your affiliation as also stated in the Posting 
Guidelines (http://www.quantum-espresso.org/forum#1.0)



On 09/17/18 09:49, Asad Mahmood wrote:

Hi,

I did relax calculation for monolayer Germanene (honeycomb) with about 
20 angstrom vacuum between two layers. I did it for single sided 
hydrogenation. Final relaxed coordinates gave me different values of 
Ge-H bond length. One was 1.49 angstrom while other was 1.52 angstrom. 
Buckling length also decreased, while it should not decrease according 
to previous literature.

Is there any error?

Asad


___
users mailing list
users@lists.quantum-espresso.org
https://lists.quantum-espresso.org/mailman/listinfo/users


--
Dr. rer. nat. Thomas Brumme
Wilhelm-Ostwald-Institute for Physical and Theoretical Chemistry
Leipzig University
Phillipp-Rosenthal-Strasse 31
04103 Leipzig

Tel:  +49 (0)341 97 36456

email: thomas.bru...@uni-leipzig.de

___
users mailing list
users@lists.quantum-espresso.org
https://lists.quantum-espresso.org/mailman/listinfo/users

Re: [QE-users] Memory requirements of projwfc.x in k-resolved case

2018-08-14 Thread Thomas Brumme

Dear all,

OK, I did some small test using a modified example 4 from the PP examples.
Essentially, instead of using the band path given there I used (random):

K_POINTS crystal_b
8
0 0 0 10
1 0 0 10
1 1 0 10
1 1 1 10
0 0 0 10
0 1 0 10
0 1 1 10
0 0 0 1

And I changed the pseudo between

Pt.pz-n-rrkjus_psl.0.1.UPF
Pt.pz-n-kjpaw_psl.0.1.UPF

Pt.rel-pz-n-rrkjus_psl.0.1.UPF
Pt.rel-pz-n-kjpaw_psl.0.1.UPF

Finally, I calculated the projected wave functions by using projwfc.x with:


    prefix='Pt',
    outdir='$TMP_DIR/'
    ngauss = 0,
    degauss = 0.01,
    Emin = 8,
    Emax = 40,
    DeltaE = 0.01,
    lsym = .false.,
    kresolveddos = .true.,
    filproj = 'pt.band.dat.proj',
/

Or without the kresolveddos flag set to true, i.e., deleting the last 2 
lines above.


In the case of kresolveddos = .true. I always observe that the memory 
used by one process (4 in total) increases to nearly twice the value of 
the others. For example:


for paw (logged with top)
28710 tbrumme   20   0  86  42480  22016 R 242.0  0.3   0:21.21 
projwfc.x
28710 tbrumme   20   0  909748  74836  22592 R  98.0  0.5   0:25.11 
projwfc.x


for rel-paw
28921 tbrumme   20   0  44  52388  21380 R 227.5  0.3   0:35.94 
projwfc.x
28921 tbrumme   20   0  920608  86028  22476 R 100.0  0.5   0:40.07 
projwfc.x


for us
29285 tbrumme   20   0  870516  34372  21304 R 219.6  0.2   0:23.30 
projwfc.x
29285 tbrumme   20   0  906888  71848  22272 R  98.0  0.4   0:25.95 
projwfc.x


for rel-us
29102 tbrumme   20   0  878620  43500  21980 R 223.5  0.3   0:34.56 
projwfc.x
29102 tbrumme   20   0  914472  79604  22324 R 102.0  0.5   0:39.10 
projwfc.x


This also happens in a serial calculation, but does not happen when 
calculating with kresolveddos=.false. For the bands calculation I can 
see a maximum memory usage like (for rel paw):

28850 tbrumme   20   0  892820  51972  21256 R 178.4  0.3   1:55.15 pw.x

which is comparable to the memory usage before the sudden increase. The 
output of the estimated memory usage in the bands run tells me that I 
will need a maximum of 7.72 MB per process and 30.89 MB total for us 
potentials. The 34 MB given above (before the increase) is already more 
than the estimate - but OK, I know that it's just an estimate and the 
estimation of the usage was improved in a recent commit. Yet, at the end 
the one task uses even twice this estimate. Judging from the PID I think 
it is the master process (ionode ?!).


In my large calculation of MoS2 on MoS2 projwfc.x does not even reach 
the point of writing DOS per atom, i.e., *.pdos_atm#* and thus the crash 
must be before. So, one way of reducing the memory usage would obviously 
be to reduce the number of k points and apparently also reducing the 
number of energy points does help. And it turns out that this DeltaE 
crucially affects the used memory by one process... So while writing 
this email I found a solution - more or less.


To cut a long story short:

If someone experience the same problem, i.e., memory problems for 
projwfc.x, try reducing the deltae


Cheerio

Thomas Brumme

On 08/14/18 12:03, Thomas Brumme wrote:

Dear all,

I'm struggling to project the wave functions on atoms in the 
k-resolved case.
The job always crashes because of the memory limit. The system itself 
is quite
large - 2 layers of MoS2 but rotated, total of 138 atoms. The band 
structure

calculation for 151 k points finished without problems using 1.72 GB RAM
maximum per core (100 cores in total). Starting the projwfc.x run with 
the

same settings (100 cores, 2 GB RAM per core) the job is killed because it
exceeds the memory. Increasing to 8 GB per core does not solve the 
problem.


What are the exact memory requirements for projwfc.x for the k-resolved
case? I read in the forums that it shouldn't be more than the 
corresponding
scf or bands run, should it? Then why does those runs finish and the 
projwfc.x
not? I'm using version 6.2.1 compiled with the old xml format (as I 
started the
calculation when the new XML was not there yet and had to stop in 
between)
Furthermore, the normal (scf and bands) run are parallelized via the 
standard
R & G space devision on 100 cores. Um, and I'm using the relativistic 
PBE paw

pseudos of the pslibrary, 55 Ry and 440 Ry cutoffs.

Is the code reading in the wave functions of all k points at once, 
i.e., would

it help to reduce the number of k points?

Regards

Thomas



--
Dr. rer. nat. Thomas Brumme
Wilhelm-Ostwald-Institute for Physical and Theoretical Chemistry
Leipzig University
Phillipp-Rosenthal-Strasse 31
04103 Leipzig

Tel:  +49 (0)341 97 36456

email: thomas.bru...@uni-leipzig.de

___
users mailing list
users@lists.quantum-espresso.org
https://lists.quantum-espresso.org/mailman/listinfo/users

[QE-users] Memory requirements of projwfc.x in k-resolved case

2018-08-14 Thread Thomas Brumme

Dear all,

I'm struggling to project the wave functions on atoms in the k-resolved 
case.
The job always crashes because of the memory limit. The system itself is 
quite

large - 2 layers of MoS2 but rotated, total of 138 atoms. The band structure
calculation for 151 k points finished without problems using 1.72 GB RAM
maximum per core (100 cores in total). Starting the projwfc.x run with the
same settings (100 cores, 2 GB RAM per core) the job is killed because it
exceeds the memory. Increasing to 8 GB per core does not solve the problem.

What are the exact memory requirements for projwfc.x for the k-resolved
case? I read in the forums that it shouldn't be more than the corresponding
scf or bands run, should it? Then why does those runs finish and the 
projwfc.x
not? I'm using version 6.2.1 compiled with the old xml format (as I 
started the

calculation when the new XML was not there yet and had to stop in between)
Furthermore, the normal (scf and bands) run are parallelized via the 
standard
R & G space devision on 100 cores. Um, and I'm using the relativistic 
PBE paw

pseudos of the pslibrary, 55 Ry and 440 Ry cutoffs.

Is the code reading in the wave functions of all k points at once, i.e., 
would

it help to reduce the number of k points?

Regards

Thomas

--
Dr. rer. nat. Thomas Brumme
Wilhelm-Ostwald-Institute for Physical and Theoretical Chemistry
Leipzig University
Phillipp-Rosenthal-Strasse 31
04103 Leipzig

Tel:  +49 (0)341 97 36456

email: thomas.bru...@uni-leipzig.de

___
users mailing list
users@lists.quantum-espresso.org
https://lists.quantum-espresso.org/mailman/listinfo/users


Re: [QE-users] dipole correction - "saggy" electrostatic potential?

2018-07-01 Thread Dr. Thomas Brumme

Dear Chris,

adding to my last reply: in the end, it all, of course, depends on what
you want to simulate. If you're aiming for a charged species on a surface
then assume_isolated='2D' might be wrong if you don't increase the slab
(mimicking the surface of a bulk material) such that the electric field
at the bottom does not influence the top. It might be worth to also look
at my implementation of a gate setup (gate=.true.) or to wait till the
author of assume_isolated='2D', Thibault Sohier, releases his implementation
including the gate setup. Gate? Maybe check this paper to see why this
could be useful:

https://www.sciencedirect.com/science/article/pii/S0169433218315022

As I said, it all depends on the situation you want to simulate. But a
flat potential with the same absolute value on both sides of the system
is rarely correct for a charged system.

Regards

Thomas

Zitat von Christoph Wolf :


Dear Thomas,

I played a bit with "assume_isolated='2D'" but I do not think that this can
correct the electrostatic potential of charged sytems (in the sense that
the potential becomes "flat") unless I am interpreting the output
(attached) wrong.

One way that gives me a flat vacuum potential is to use the M-P scheme but
that only works for cubic systems. After reading about the implementation
in VASP a bit I also think that is what they recommend.

Best,
Chris

On Thu, Jun 28, 2018 at 7:04 PM, Dr. Thomas Brumme <
thomas.bru...@uni-leipzig.de> wrote:


Dear Chris,

The potential shows the typical quadratic dependence on z since you're
calculating a charged system - there is a homogeneous background charge
since the 3D pbc system is assumed to be neutral. This has nothing to do
with the dipole correction. Depending on what you want to do next it might
be useful to set the flag assume_isolated='2D'

Regards

Thomas

Zitat von Christoph Wolf :


Dear all,


I am still observing something strange in my slab + dipole correction
calculation that I do not fully understand.

When using dipfield+tefield (eopreg and emaxpos well within the vacuum
region) I encounter a "saggy" electrostatic potential (plot_num=11)
despite
the sawtooth efield potential (plot_num=12) looking as usual. Maybe
someone
can give it a look and confirm if this is due to the excess charge in the
system (this does not happen when running the same system in VASP)?

I attach input and the plot of the potential for 2 and 4 layers of vacuum
(more vacuum does seem to improve the situation).

Thank you in advance for your time and assistance!

Chris

PS: I am using a QE version with dipole bug fix:

 Program PWSCF v.6.2.2 starts on 25Jun2018 at 10:39:24

--
Postdoctoral Researcher
Center for Quantum Nanoscience, Institute for Basic Science
Ewha Womans University, Seoul, South Korea



--
Dr. rer. nat. Thomas Brumme
Wilhelm-Ostwald-Institute for Physical and Theoretical Chemistry
Leipzig University
Phillipp-Rosenthal-Strasse 31
04103 Leipzig

Tel:  +49 (0)341 97 36456

email: thomas.bru...@uni-leipzig.de





--
Postdoctoral Researcher
Center for Quantum Nanoscience, Institute for Basic Science
Ewha Womans University, Seoul, South Korea




___
users mailing list
users@lists.quantum-espresso.org
https://lists.quantum-espresso.org/mailman/listinfo/users


Re: [QE-users] dipole correction - "saggy" electrostatic potential?

2018-07-01 Thread Dr. Thomas Brumme

Dear Chris,

The result of assume_isolated='2D' is correct, also physically:
The potential of a charged plate increases linearly with the distance  
from this plate. Check physics books on electrostatics. The wiggles in  
the center of the vacuum are due to the implementation - since they're  
in the "nonphysical" region (wave functions are zero) they don't matter.


Cheers

Thomas

Zitat von Christoph Wolf :


Dear Thomas,

I played a bit with "assume_isolated='2D'" but I do not think that this can
correct the electrostatic potential of charged sytems (in the sense that
the potential becomes "flat") unless I am interpreting the output
(attached) wrong.

One way that gives me a flat vacuum potential is to use the M-P scheme but
that only works for cubic systems. After reading about the implementation
in VASP a bit I also think that is what they recommend.

Best,
Chris

On Thu, Jun 28, 2018 at 7:04 PM, Dr. Thomas Brumme <
thomas.bru...@uni-leipzig.de> wrote:


Dear Chris,

The potential shows the typical quadratic dependence on z since you're
calculating a charged system - there is a homogeneous background charge
since the 3D pbc system is assumed to be neutral. This has nothing to do
with the dipole correction. Depending on what you want to do next it might
be useful to set the flag assume_isolated='2D'

Regards

Thomas

Zitat von Christoph Wolf :


Dear all,


I am still observing something strange in my slab + dipole correction
calculation that I do not fully understand.

When using dipfield+tefield (eopreg and emaxpos well within the vacuum
region) I encounter a "saggy" electrostatic potential (plot_num=11)
despite
the sawtooth efield potential (plot_num=12) looking as usual. Maybe
someone
can give it a look and confirm if this is due to the excess charge in the
system (this does not happen when running the same system in VASP)?

I attach input and the plot of the potential for 2 and 4 layers of vacuum
(more vacuum does seem to improve the situation).

Thank you in advance for your time and assistance!

Chris

PS: I am using a QE version with dipole bug fix:

 Program PWSCF v.6.2.2 starts on 25Jun2018 at 10:39:24

--
Postdoctoral Researcher
Center for Quantum Nanoscience, Institute for Basic Science
Ewha Womans University, Seoul, South Korea



--
Dr. rer. nat. Thomas Brumme
Wilhelm-Ostwald-Institute for Physical and Theoretical Chemistry
Leipzig University
Phillipp-Rosenthal-Strasse 31
04103 Leipzig

Tel:  +49 (0)341 97 36456

email: thomas.bru...@uni-leipzig.de





--
Postdoctoral Researcher
Center for Quantum Nanoscience, Institute for Basic Science
Ewha Womans University, Seoul, South Korea




___
users mailing list
users@lists.quantum-espresso.org
https://lists.quantum-espresso.org/mailman/listinfo/users


Re: [QE-users] dipole correction - "saggy" electrostatic potential?

2018-06-28 Thread Dr. Thomas Brumme

Dear Chris,

The potential shows the typical quadratic dependence on z since you're  
calculating a charged system - there is a homogeneous background  
charge since the 3D pbc system is assumed to be neutral. This has  
nothing to do with the dipole correction. Depending on what you want  
to do next it might be useful to set the flag assume_isolated='2D'


Regards

Thomas

Zitat von Christoph Wolf :


Dear all,

I am still observing something strange in my slab + dipole correction
calculation that I do not fully understand.

When using dipfield+tefield (eopreg and emaxpos well within the vacuum
region) I encounter a "saggy" electrostatic potential (plot_num=11) despite
the sawtooth efield potential (plot_num=12) looking as usual. Maybe someone
can give it a look and confirm if this is due to the excess charge in the
system (this does not happen when running the same system in VASP)?

I attach input and the plot of the potential for 2 and 4 layers of vacuum
(more vacuum does seem to improve the situation).

Thank you in advance for your time and assistance!

Chris

PS: I am using a QE version with dipole bug fix:

 Program PWSCF v.6.2.2 starts on 25Jun2018 at 10:39:24

--
Postdoctoral Researcher
Center for Quantum Nanoscience, Institute for Basic Science
Ewha Womans University, Seoul, South Korea


--
Dr. rer. nat. Thomas Brumme
Wilhelm-Ostwald-Institute for Physical and Theoretical Chemistry
Leipzig University
Phillipp-Rosenthal-Strasse 31
04103 Leipzig

Tel:  +49 (0)341 97 36456

email: thomas.bru...@uni-leipzig.de

___
users mailing list
users@lists.quantum-espresso.org
https://lists.quantum-espresso.org/mailman/listinfo/users

Re: [QE-users] Dipole correction: should there be a "dip" in the electrostatic potential?

2018-05-18 Thread Thomas Brumme

Dear Christoph,

which version of QE are you using? Was it compiled with the old XML 
format or the new one?
The new XML format had a bug in the way the input related to the dipole 
correction was saved.

This was solved about a month ago:

https://gitlab.com/QEF/q-e/commit/81a946e15b80972f629c88e47e2545596350f31c

see also the related issue (closed)

https://gitlab.com/QEF/q-e/issues/5

Maybe your problem is due to this. If you use 6.2.1 either download the 
develop branch or

compile using the old XML implementation.

Regards

Thomas

On 18.05.2018 12:50, Christoph Wolf wrote:

Dear all,

I am (again...) fighting with the dipole correction. I have a system 
consisting of Ag and MgO centered in the cell around z=0.5


ATOMIC_POSITIONS (crystal)
Ag   0.0   0.0   0.323271580
Ag   0.0   0.5   0.385512833

Mg  -0.0   0.0   0.721978968
O    0.5   0.0   0.723372273
O   -0.0   0.5   0.723371161
Mg   0.5   0.5   0.721975961

Then I apply tefield and dipfield with

  eamp    = 0.00
  edir    = 3
  emaxpos = 0.95
  eopreg  = 0.10

which means my system has no discontinuity anywhere in the slab and 
the field is "physical" between 0.95+0.1-1=0.05 and 0.95 of the z 
direction.


The SCF cycle converges but the pot_num=11 never becomes horizontal, 
rather it has a very small slope. I tried to increase the dipole 
length to see if that makes a difference but apparently it does not 
help either. With above input the potential amplitude at scf 
convergence is


 Adding external electric field

 Computed dipole along edir(3) :
    Elec. dipole  0.4894 Ry au,  1.2440 Debye
    Ion. dipole   0.4902 Ry au,  1.2460 Debye
    Dipole    0.2274 Ry au,  0.5779 Debye
    Dipole field  0.0008 Ry au,

    Potential amp.   -0.0855 Ry
    Total length 55.5874 bohr


Is there a scenario where the dipole correction could fail if the 
potential is "almost flat" in the vacuum region? If I run the same 
with VASP I get a small "jump" in the potential making it flat in the 
vacuum region, so I was wondering why this does not happen (probably 
different approach in PWSCF)?


Input and resulting potential is attached - any help is welcome!

Thanks in advance and with best wishes,
Chris
--
Postdoctoral Researcher
Center for Quantum Nanoscience, Institute for Basic Science
Ewha Womans University, Seoul, South Korea


___
users mailing list
users@lists.quantum-espresso.org
https://lists.quantum-espresso.org/mailman/listinfo/users


--
Dr. rer. nat. Thomas Brumme
Wilhelm-Ostwald-Institute for Physical and Theoretical Chemistry
Leipzig University
Phillipp-Rosenthal-Strasse 31
04103 Leipzig

Tel:  +49 (0)341 97 36456

email: thomas.bru...@uni-leipzig.de

___
users mailing list
users@lists.quantum-espresso.org
https://lists.quantum-espresso.org/mailman/listinfo/users

Re: [QE-users] Bandstructure with HSE provides wrong eigenvalues

2018-05-07 Thread Thomas Brumme

Didn't see this ;)

Well, anyway, there are 2 flaws in the input of Stefan.
First the different nqx as noticed by Layla and furthermore
the k-point list in the "bands" calculation is wrong.
One has to give the k points along the path in the Brillouin
zone after the list of the regular grid and everything needs
to be in the scf calculation. I.e.

K_POINTS {tpiba}
1778
  0.000   0.000   0.000 1
 -0.083   0.083  -0.083 1
 -0.167   0.167  -0.167 1
 -0.250   0.250  -0.250 1
 -0.333   0.333  -0.333 1
 -0.417   0.417  -0.417 1
  0.500  -0.500   0.500 1
  0.417  -0.417   0.417 1
  0.333  -0.333   0.333 1
  0.250  -0.250   0.250 1
  0.167  -0.167   0.167 1
  0.083  -0.083   0.083 1
  0.083   0.083   0.083 1
  0.000   0.167   0.000 1
 -0.083   0.250  -0.083 1
 -0.167   0.333  -0.167 1
...
   0.5000  0.5000  0.5000 0
   0.  0.  0. 0
   0.3889  0.3889  0.3889 0
   0.  0.  0. 0
   0.2778  0.2778  0.2778 0
   0.  0.  0. 0
   0.1667  0.1667  0.1667 0
   0.  0.  0. 0
   0.0556  0.0556  0.0556 0
   0.  0.  0. 0
   0.  0.  0. 0

The zero weight in the last column is important.
This is what I understand from your (Lorenzos) answer.
The interpolation is however much faster. For well behaved
bands (i.e., no complicated crossings or kinks) BoltzTraP works
and it is really fast ;) For those band structures with crossings
it is more involved as you need to sort the eigenenergies more
or less by hand - or maybe bands.x can help...

Cheerio

Thomas


On 07.05.2018 09:18, Lorenzo Paulatto wrote:

On 06/05/18 17:34, Dr. Thomas Brumme wrote:

Paulatto and I cite his answer at the end of the mail - you can find it
under the following link:


I'm flattered that my badly-spelled answer is taken in such a 
consideration, although it is important to notice that it was actually 
to Stefan himself that the answer was given! And he was actually using 
the extra point with zero weight trick. I told him to ask here about 
the eigenvalues discrepancy, because I did not know what could be the 
cause.


cheers



--
Dr. rer. nat. Thomas Brumme
Wilhelm-Ostwald-Institute for Physical and Theoretical Chemistry
Leipzig University
Phillipp-Rosenthal-Strasse 31
04103 Leipzig

Tel:  +49 (0)341 97 36456

email: thomas.bru...@uni-leipzig.de

___
users mailing list
users@lists.quantum-espresso.org
https://lists.quantum-espresso.org/mailman/listinfo/users

Re: [QE-users] Bandstructure with HSE provides wrong eigenvalues

2018-05-06 Thread Dr. Thomas Brumme
  0.3889  0.3889  0.3889 3
   0.  0.  0. 4
   0.2778  0.2778  0.2778 5
   0.  0.  0. 6
   0.1667  0.1667  0.1667 7
   0.  0.  0. 8
   0.0556  0.0556  0.0556 9
   0.  0.  0. 10
   0.  0.  0. 11
 ...



The eigenvalues from my 1st calculation si.scf.out are correct !
But the eigenvalues of the 2nd calculation si.bands.out are wrong !
E.g. the HOMO-LUMO distance at the Gamma point is just too big  
(compared to VASP results which are proven to be right)!


Where is the mistake in my input file si.bands.in for the 2nd calculation ??

Thanks a lot, best
Stefan


--
Dr. rer. nat. Thomas Brumme
Wilhelm-Ostwald-Institute for Physical and Theoretical Chemistry
Leipzig University
Phillipp-Rosenthal-Strasse 31
04103 Leipzig

Tel:  +49 (0)341 97 36456

email: thomas.bru...@uni-leipzig.de

___
users mailing list
users@lists.quantum-espresso.org
https://lists.quantum-espresso.org/mailman/listinfo/users


Re: [QE-users] Plotband.x .ps output only display dot

2018-04-29 Thread Dr. Thomas Brumme

Now that you mention it:

Erland, the K_POINTS card makes no sense.
Either you want to give a path from one point to another, than you
have to use K_POINTS {crystal_b} or you specify them using
K_POINTS {crystal} but then you have to give the points explicitly.
In the former case, the last point gets a "weight" of 1 as there is
no additional point.

Regards

Thomas

Zitat von Manu Hegde <mhe...@uwaterloo.ca>:


looks like K-point units, please double check it.
Manu

On Sun, Apr 29, 2018 at 1:58 PM, Dr. Thomas Brumme <
thomas.bru...@uni-leipzig.de> wrote:


Dear Erland (?)

In the last line of your Cu.bands.in it should be "0 0 0 1" as
there is no additional point, but I don't know if this is the
origin of your problem. If not, we need more information, e.g.,
the version you're using, maybe part of the output (especially
with respect to the point above).

And please, read the posting guidelines:

http://www.quantum-espresso.org/forum#1.0

Especially these points:

"Remember that even experts cannot guess where a problem lies in
the absence of sufficient information. One piece of information
that must always be provided is the version number you are using."

"Sign your post with your name and affiliation."

Cheerio

Thomas

Zitat von Erland Rachmad <erland.rach...@gmail.com>:


Dear users,


I tried to do some band calculations of Copper for learning. I have ran
scf
and then bands calculation but whenever I tried plotband it always display
dot without any line (I attach the .ps output file). How do I fix this?
For
the K_POINTS, I specify G-X-W-K-G-L-U-W-L-K|U-X for FCC path.




--
Dr. rer. nat. Thomas Brumme
Wilhelm-Ostwald-Institute for Physical and Theoretical Chemistry
Leipzig University
Phillipp-Rosenthal-Strasse 31
04103 Leipzig

Tel:  +49 (0)341 97 36456

email: thomas.bru...@uni-leipzig.de


___
users mailing list
users@lists.quantum-espresso.org
https://lists.quantum-espresso.org/mailman/listinfo/users





___
users mailing list
users@lists.quantum-espresso.org
https://lists.quantum-espresso.org/mailman/listinfo/users


Re: [QE-users] Plotband.x .ps output only display dot

2018-04-29 Thread Dr. Thomas Brumme

Dear Erland (?)

In the last line of your Cu.bands.in it should be "0 0 0 1" as
there is no additional point, but I don't know if this is the
origin of your problem. If not, we need more information, e.g.,
the version you're using, maybe part of the output (especially
with respect to the point above).

And please, read the posting guidelines:

http://www.quantum-espresso.org/forum#1.0

Especially these points:

"Remember that even experts cannot guess where a problem lies in
the absence of sufficient information. One piece of information
that must always be provided is the version number you are using."

"Sign your post with your name and affiliation."

Cheerio

Thomas

Zitat von Erland Rachmad <erland.rach...@gmail.com>:


Dear users,

I tried to do some band calculations of Copper for learning. I have ran scf
and then bands calculation but whenever I tried plotband it always display
dot without any line (I attach the .ps output file). How do I fix this? For
the K_POINTS, I specify G-X-W-K-G-L-U-W-L-K|U-X for FCC path.



--
Dr. rer. nat. Thomas Brumme
Wilhelm-Ostwald-Institute for Physical and Theoretical Chemistry
Leipzig University
Phillipp-Rosenthal-Strasse 31
04103 Leipzig

Tel:  +49 (0)341 97 36456

email: thomas.bru...@uni-leipzig.de

___
users mailing list
users@lists.quantum-espresso.org
https://lists.quantum-espresso.org/mailman/listinfo/users


Re: [QE-users] Error in routine ggen too many g-vectors

2018-04-27 Thread Thomas Brumme
 Dense  grid:   226331 G-vectors FFT dimensions: ( 45,  45, 243)

 Smooth grid:   165041 G-vectors FFT dimensions: ( 45,  45, 216)

 Estimated max dynamical RAM per process > 104.83Mb

 Estimated total allocated dynamical RAM > 6709.07Mb

 %%
 Error in routine ggen 1 (226332):
 too many g-vectors
 %%

 stopping ...

-

*Sohail Ahmad*
Department of Physics
King Khalid University
Abha, Saudi Arabia



___
users mailing list
users@lists.quantum-espresso.org
https://lists.quantum-espresso.org/mailman/listinfo/users


--
Dr. rer. nat. Thomas Brumme
Wilhelm-Ostwald-Institute for Physical and Theoretical Chemistry
Leipzig University
Phillipp-Rosenthal-Strasse 31
04103 Leipzig

Tel:  +49 (0)341 97 36456

email: thomas.bru...@uni-leipzig.de

___
users mailing list
users@lists.quantum-espresso.org
https://lists.quantum-espresso.org/mailman/listinfo/users

Re: [QE-users] Dipole corrections (dipfield) and the position of the slab

2018-04-27 Thread Thomas Brumme

Dear Chris,

in the end it (of course) doesn't (and shouldn't) matter where you 
center what.
You can also center the system at z=0 and the dipole at z=0.5. Just take 
care

that the distance between system and dipole is large enough.

Regards

Thomas

On 27.04.2018 12:06, Christoph Wolf wrote:

Dear Thomas,

thank you for your detailed reply!

If I understood this correctly, the ideal situation would be to have 
the slab in the center of the cell


Atom xx yy 0.5

(in crystal coordinates)

to center the dipole at 0% (=100% due to PBC) a reasonable choice 
would be emaxpos=0.95 and eopreg=0.10. In the dipole example they 
located the atoms around z=0 of the cells and put the dipole close to 
the center of the cell.


Muchas Gracias/Vielen Dank from Spain,

Christoph




On Fri, Apr 27, 2018 at 10:59 AM, Thomas Brumme 
<thomas.bru...@uni-leipzig.de <mailto:thomas.bru...@uni-leipzig.de>> 
wrote:


Dear Chris,

both planes of the dipole (the one at emaxpos and the one with the
opposite charge at emaxpos+eopreg) have to be in the vacuum region.
In fact, there should be enough space such that the wavefunctions are
essentially zero at the dipole planes. However, if the dipole is
too large,
charge can spill into the vacuum region as plane waves are not
localized
on the system and the charge could be in a lower energy state at the
dipole. In other words. don't use 50 Angstrom of vacuum as this
will lead
to a very low minimum in the total potential at the dipole.
See also this paper:

https://journals.aps.org/prb/abstract/10.1103/PhysRevB.85.045121
<https://journals.aps.org/prb/abstract/10.1103/PhysRevB.85.045121>

In this paper charged systems are discussed but similar things
apply to
the dipole correction. Thus, if your system is centered at 50% of the
cell, center the dipole at zero and converge things with
increasing the
size along z.

Regards

Thomas


On 26.04.2018 14:00, Christoph Wolf wrote:

Dear all,

After trying for a few days I am still a bit puzzled by the
"proper application" of the dipole correction. To test this I
have made a sheet of graphene added hydrogen below and fluorine
above. I then apply the following corrections:

    tefield = .true.
    dipfield =.true.

and

  eamp    = 0.00
  edir    = 3
  emaxpos = 0.80 !(=16 Angstrom)
  eopreg  = 0.10 ! (=2 Angstrom)

The cell is 20 A in total. As I shift the layer from 0% of the
cell to 50% cell (whilst keeping above emaxpos at 80% and eopreg
at 10% of the cell) the Fermi level shifts slightly (~0.2-0.5 eV
difference) and the electrostatic potential (pp.x plot num 11 and
then planar average using average.x as in the work-function
example) is only "flat" in the vacuum region when the sample is
about 3A from the bottom of the cell (i.e. the z coordinate of
atoms has to be larger than 3 A).

Reading the pw.x input I was under the impression that only
emaxpos has to fall into the vacuum but is there also a "rule of
thumb" for eopreg?

Thanks in advance for your help!

Best,
Chris


PS: I saw the related discussion, but it does not really answer
this I think...
http://qe-forge.org/pipermail/pw_forum/2009-December/089951.html
<http://qe-forge.org/pipermail/pw_forum/2009-December/089951.html>
-- 
Postdoctoral Researcher

Center for Quantum Nanoscience, Institute for Basic Science
Ewha Womans University, Seoul, South Korea


___
users mailing list
users@lists.quantum-espresso.org
<mailto:users@lists.quantum-espresso.org>
https://lists.quantum-espresso.org/mailman/listinfo/users
<https://lists.quantum-espresso.org/mailman/listinfo/users>


-- 
Dr. rer. nat. Thomas Brumme

Wilhelm-Ostwald-Institute for Physical and Theoretical Chemistry
Leipzig University
Phillipp-Rosenthal-Strasse 31
04103 Leipzig

Tel:  +49 (0)341 97 36456

email:thomas.bru...@uni-leipzig.de <mailto:thomas.bru...@uni-leipzig.de>




--
Postdoctoral Researcher
Center for Quantum Nanoscience, Institute for Basic Science
Ewha Womans University, Seoul, South Korea


--
Dr. rer. nat. Thomas Brumme
Wilhelm-Ostwald-Institute for Physical and Theoretical Chemistry
Leipzig University
Phillipp-Rosenthal-Strasse 31
04103 Leipzig

Tel:  +49 (0)341 97 36456

email: thomas.bru...@uni-leipzig.de

___
users mailing list
users@lists.quantum-espresso.org
https://lists.quantum-espresso.org/mailman/listinfo/users

Re: [QE-users] Dipole corrections (dipfield) and the position of the slab

2018-04-27 Thread Thomas Brumme

Dear Chris,

both planes of the dipole (the one at emaxpos and the one with the
opposite charge at emaxpos+eopreg) have to be in the vacuum region.
In fact, there should be enough space such that the wavefunctions are
essentially zero at the dipole planes. However, if the dipole is too large,
charge can spill into the vacuum region as plane waves are not localized
on the system and the charge could be in a lower energy state at the
dipole. In other words. don't use 50 Angstrom of vacuum as this will lead
to a very low minimum in the total potential at the dipole.
See also this paper:

https://journals.aps.org/prb/abstract/10.1103/PhysRevB.85.045121

In this paper charged systems are discussed but similar things apply to
the dipole correction. Thus, if your system is centered at 50% of the
cell, center the dipole at zero and converge things with increasing the
size along z.

Regards

Thomas

On 26.04.2018 14:00, Christoph Wolf wrote:

Dear all,

After trying for a few days I am still a bit puzzled by the "proper 
application" of the dipole correction. To test this I have made a 
sheet of graphene added hydrogen below and fluorine above. I then 
apply the following corrections:


    tefield = .true.
    dipfield =.true.

and

  eamp    = 0.00
  edir    = 3
  emaxpos = 0.80 !(=16 Angstrom)
  eopreg  = 0.10 ! (=2 Angstrom)

The cell is 20 A in total. As I shift the layer from 0% of the cell to 
50% cell (whilst keeping above emaxpos at 80% and eopreg at 10% of the 
cell) the Fermi level shifts slightly (~0.2-0.5 eV difference) and the 
electrostatic potential (pp.x plot num 11 and then planar average 
using average.x as in the work-function example) is only "flat" in the 
vacuum region when the sample is about 3A from the bottom of the cell 
(i.e. the z coordinate of atoms has to be larger than 3 A).


Reading the pw.x input I was under the impression that only emaxpos 
has to fall into the vacuum but is there also a "rule of thumb" for 
eopreg?


Thanks in advance for your help!

Best,
Chris


PS: I saw the related discussion, but it does not really answer this I 
think... http://qe-forge.org/pipermail/pw_forum/2009-December/089951.html

--
Postdoctoral Researcher
Center for Quantum Nanoscience, Institute for Basic Science
Ewha Womans University, Seoul, South Korea


___
users mailing list
users@lists.quantum-espresso.org
https://lists.quantum-espresso.org/mailman/listinfo/users


--
Dr. rer. nat. Thomas Brumme
Wilhelm-Ostwald-Institute for Physical and Theoretical Chemistry
Leipzig University
Phillipp-Rosenthal-Strasse 31
04103 Leipzig

Tel:  +49 (0)341 97 36456

email: thomas.bru...@uni-leipzig.de

___
users mailing list
users@lists.quantum-espresso.org
https://lists.quantum-espresso.org/mailman/listinfo/users

Re: [QE-users] Why does the difference between VASP and QE, even though they are BOMD theory

2018-04-11 Thread Dr. Thomas Brumme
IT Madras, India.


--
Dr. rer. nat. Thomas Brumme
Wilhelm-Ostwald-Institute for Physical and Theoretical Chemistry
Leipzig University
Phillipp-Rosenthal-Strasse 31
04103 Leipzig

Tel:  +49 (0)341 97 36456

email: thomas.bru...@uni-leipzig.de


___
users mailing list
users@lists.quantum-espresso.org
https://lists.quantum-espresso.org/mailman/listinfo/users


Re: [QE-users] phonon calculations on polar slabs - imaginary modes

2018-04-06 Thread Thomas Brumme

Dear Chris,

I think that the dipole correction is not working in ph.x, but
you could try the new Coulomb cutoff technique implemented
in both pw.x and ph.x by Sohier and coworkers, see
assume_isolated = '2D'

Regards

Thomas

On 06.04.2018 15:57, Christoph Wolf wrote:

Dear all,

I am curious if someone could shed some light onto the following:

I am currently calculating phonons of MgO using QE 6.2.1 and NC PPs 
from the ONCV library (requirement of post-processing with YAMBO). 
Bulk properties (lattice and vibrational) are very nicely reproduced.


When moving to slabs (1,2,3 layers with ca 20 A vacuum, dipfield 
correction applied) phonons become considerably "ugly" (very high 
energy imaginary modes). I was wondering if there is a "trick" to get 
better phonons in slabs with polar surfaces either during ph.x (I 
tried tighter convergence down to 10e-16, higher k grids in pw.x, 
higher q grids in ph.x without much success) or post-processing (not 
sure if the usual ASR are applicable in slabs)?


Since calculations on the slabs are rather time consuming any 
suggestion is very welcome!


A nice weekend to everyone from San Sebastian,

Chris

--
Postdoctoral Researcher
Center for Quantum Nanoscience, Institute for Basic Science
Ewha Womans University, Seoul, South Korea


___
users mailing list
users@lists.quantum-espresso.org
https://lists.quantum-espresso.org/mailman/listinfo/users


--
Dr. rer. nat. Thomas Brumme
Wilhelm-Ostwald-Institute for Physical and Theoretical Chemistry
Leipzig University
Phillipp-Rosenthal-Strasse 31
04103 Leipzig

Tel:  +49 (0)341 97 36456

email: thomas.bru...@uni-leipzig.de

___
users mailing list
users@lists.quantum-espresso.org
https://lists.quantum-espresso.org/mailman/listinfo/users

Re: [QE-users] convergence problem

2018-03-06 Thread Dr. Thomas Brumme
Well, the error says what's the problem... You can't use the option  
lefield for

a metallic system. Depending on what you want to investigate you can try with
the sawtooth approach using tefield/dipfield.

Regards

Thomas


Zitat von Sohail Ahmad <sohailphys...@yahoo.co.in>:

Thanks Dr Thomas BrummeAs suggested,when i changed occuptaions =  
'fixed' to 'smearning'

it gives following error
%%
 Error in routine iosys (1):
 Berry Phase/electric fields only for insulators!
%%


 calculation = 'relax',
 restart_mode = 'from_scratch',
 pseudo_dir= '/home/sohail/pseudo',
 outdir='./OUT',
 prefix='PdS2bAAel02',
 lelfield = .true.,
 gdir = 3,
 etot_conv_thr = 1.0d-5,
 forc_conv_thr = 1.0d-4,
/

 ibrav = 4, a = 3.53, b = 3.53, c = 20.0, cosAB = -0.5, cosAC  
= 0.0, cosBC = 0.0,

 nat = 6, ntyp = 2,
 ecutwfc = 80,
 ecutrho = 400,
 nbnd = 40,
 occupations = 'smearing',smearing = 'gaussian', degauss = 0.001,
/

 mixing_beta = 0.3,
 conv_thr = 1.0d-9,
 electron_maxstep = 900,
efield_cart(1) = 0.0d0,
efield_cart(2) = 0.0d0,
efield_cart(3) = 0.0054d0,
/

ion_dynamics = 'bfgs',
/
ATOMIC_SPECIES
Pd 106.42 Pd.pbe-mt_fhi.UPF
S 32.06 S.pbe-mt_fhi.UPF
ATOMIC_POSITIONS angstrom
Pd 0.  
0. 0.
S 1.76627922  
1.01975766 1.28191684
S 1.76627922  
-1.01975766 -1.28191684
Pd 0.  
0. 6.6000
S 1.76627922  
1.01975766 7.88191684
S 1.76627922  
-1.01975766 5.31808316

K_POINTS AUTOMATIC
12 12 4 0 0 0



On Tuesday, 6 March 2018 6:11 PM, Dr. Thomas Brumme  
<thomas.bru...@uni-leipzig.de> wrote:



 Dear Sohail,

bilayer PdS2 becomes metallic and thus you need to change from
occupations='fixed' to 'smearing'...
See also:

http://onlinelibrary.wiley.com/doi/10.1002/anie.201309280/full

Regards

Thomas Brumme

Zitat von Sohail Ahmad <sohailphys...@yahoo.co.in>:


I wish to apply electric field in Z direction to bilayers but it
didnot get converge aven after 5 dayswhile in case of monolayer
its fine...
Please rectify the mistakes in the following
input--

 calculation = 'relax',
 restart_mode = 'from_scratch',
 pseudo_dir= '/home/sohail/pseudo',
 outdir='./OUT',
 prefix='PdS2bAAel04',
 lelfield = .true.,
 gdir = 3,
 etot_conv_thr = 1.0d-5,
 forc_conv_thr = 1.0d-4,
/

 ibrav = 4, a = 3.53, b = 3.53, c = 20.0, cosAB = -0.5, cosAC =
0.0, cosBC = 0.0,
 nat = 6, ntyp = 2,
 ecutwfc = 80,
 ecutrho = 400,
 nbnd = 40,
 occupations = 'fixed',
/

 mixing_beta = 0.3,
 conv_thr = 1.0d-9,
 electron_maxstep = 900,
efield_cart(1) = 0.0d0,
efield_cart(2) = 0.0d0,
efield_cart(3) = 0.0108d0,
/

ion_dynamics = 'bfgs',
/
ATOMIC_SPECIES
Pd 106.42 Pd.pbe-mt_fhi.UPF
S 32.06 S.pbe-mt_fhi.UPF
ATOMIC_POSITIONS angstrom
Pd 0.  
0. 0.
S 1.76627922  
1.01975766 1.28191684
S 1.76627922  
-1.01975766 -1.28191684
Pd 0.  
0. 6.6000
S 1.76627922  
1.01975766 7.88191684
S 1.76627922  
-1.01975766 5.31808316

K_POINTS AUTOMATIC
16 16 4 0 0 0
-
Dr. Sohail AhmadDepartment of PhysicsKing Khalid UniversityAbha,
Saudi
Arabia


--
Dr. rer. nat. Thomas Brumme
Wilhelm-Ostwald-Institute for Physical and Theoretical Chemistry
Leipzig University
Phillipp-Rosenthal-Strasse 31
04103 Leipzig

Tel: +49 (0)341 97 36456

email: thomas.bru...@uni-leipzig.de




___
users mailing list
users@lists.quantum-espresso.org
https://lists.quantum-espresso.org/mailman/listinfo/users


Re: [QE-users] convergence problem

2018-03-06 Thread Dr. Thomas Brumme
Well, the error says what's the problem... You can't use the option  
lefield for

a metallic system. Depending on what you want to investigate you can try with
the sawtooth approach using tefield/dipfield.

Regards

Thomas


Zitat von Sohail Ahmad <sohailphys...@yahoo.co.in>:

Thanks Dr Thomas BrummeAs suggested,when i changed occuptaions =  
'fixed' to 'smearning'

it gives following error
%%
 Error in routine iosys (1):
 Berry Phase/electric fields only for insulators!
%%


 calculation = 'relax',
 restart_mode = 'from_scratch',
 pseudo_dir= '/home/sohail/pseudo',
 outdir='./OUT',
 prefix='PdS2bAAel02',
 lelfield = .true.,
 gdir = 3,
 etot_conv_thr = 1.0d-5,
 forc_conv_thr = 1.0d-4,
/

 ibrav = 4, a = 3.53, b = 3.53, c = 20.0, cosAB = -0.5, cosAC  
= 0.0, cosBC = 0.0,

 nat = 6, ntyp = 2,
 ecutwfc = 80,
 ecutrho = 400,
 nbnd = 40,
 occupations = 'smearing',smearing = 'gaussian', degauss = 0.001,
/

 mixing_beta = 0.3,
 conv_thr = 1.0d-9,
 electron_maxstep = 900,
efield_cart(1) = 0.0d0,
efield_cart(2) = 0.0d0,
efield_cart(3) = 0.0054d0,
/

ion_dynamics = 'bfgs',
/
ATOMIC_SPECIES
Pd 106.42 Pd.pbe-mt_fhi.UPF
S 32.06 S.pbe-mt_fhi.UPF
ATOMIC_POSITIONS angstrom
Pd 0.  
0. 0.
S 1.76627922  
1.01975766 1.28191684
S 1.76627922  
-1.01975766 -1.28191684
Pd 0.  
0. 6.6000
S 1.76627922  
1.01975766 7.88191684
S 1.76627922  
-1.01975766 5.31808316

K_POINTS AUTOMATIC
12 12 4 0 0 0



On Tuesday, 6 March 2018 6:11 PM, Dr. Thomas Brumme  
<thomas.bru...@uni-leipzig.de> wrote:



 Dear Sohail,

bilayer PdS2 becomes metallic and thus you need to change from
occupations='fixed' to 'smearing'...
See also:

http://onlinelibrary.wiley.com/doi/10.1002/anie.201309280/full

Regards

Thomas Brumme

Zitat von Sohail Ahmad <sohailphys...@yahoo.co.in>:


I wish to apply electric field in Z direction to bilayers but it
didnot get converge aven after 5 dayswhile in case of monolayer
its fine...
Please rectify the mistakes in the following
input--

 calculation = 'relax',
 restart_mode = 'from_scratch',
 pseudo_dir= '/home/sohail/pseudo',
 outdir='./OUT',
 prefix='PdS2bAAel04',
 lelfield = .true.,
 gdir = 3,
 etot_conv_thr = 1.0d-5,
 forc_conv_thr = 1.0d-4,
/

 ibrav = 4, a = 3.53, b = 3.53, c = 20.0, cosAB = -0.5, cosAC =
0.0, cosBC = 0.0,
 nat = 6, ntyp = 2,
 ecutwfc = 80,
 ecutrho = 400,
 nbnd = 40,
 occupations = 'fixed',
/

 mixing_beta = 0.3,
 conv_thr = 1.0d-9,
 electron_maxstep = 900,
efield_cart(1) = 0.0d0,
efield_cart(2) = 0.0d0,
efield_cart(3) = 0.0108d0,
/

ion_dynamics = 'bfgs',
/
ATOMIC_SPECIES
Pd 106.42 Pd.pbe-mt_fhi.UPF
S 32.06 S.pbe-mt_fhi.UPF
ATOMIC_POSITIONS angstrom
Pd 0.  
0. 0.
S 1.76627922  
1.01975766 1.28191684
S 1.76627922  
-1.01975766 -1.28191684
Pd 0.  
0. 6.6000
S 1.76627922  
1.01975766 7.88191684
S 1.76627922  
-1.01975766 5.31808316

K_POINTS AUTOMATIC
16 16 4 0 0 0
-
Dr. Sohail AhmadDepartment of PhysicsKing Khalid UniversityAbha,
Saudi
Arabia


--
Dr. rer. nat. Thomas Brumme
Wilhelm-Ostwald-Institute for Physical and Theoretical Chemistry
Leipzig University
Phillipp-Rosenthal-Strasse 31
04103 Leipzig

Tel: +49 (0)341 97 36456

email: thomas.bru...@uni-leipzig.de




___
users mailing list
users@lists.quantum-espresso.org
https://lists.quantum-espresso.org/mailman/listinfo/users


Re: [QE-users] convergence problem

2018-03-06 Thread Dr. Thomas Brumme

Dear Sohail,

bilayer PdS2 becomes metallic and thus you need to change from  
occupations='fixed' to 'smearing'...

See also:

http://onlinelibrary.wiley.com/doi/10.1002/anie.201309280/full

Regards

Thomas Brumme

Zitat von Sohail Ahmad <sohailphys...@yahoo.co.in>:

I wish to apply electric field in Z direction to bilayers but it  
didnot get converge aven after 5 dayswhile in case of monolayer  
its fine...
Please rectify the mistakes in the following  
input--


   calculation = 'relax',
   restart_mode = 'from_scratch',
   pseudo_dir= '/home/sohail/pseudo',
   outdir='./OUT',
   prefix='PdS2bAAel04',
  lelfield = .true.,
  gdir = 3,
  etot_conv_thr = 1.0d-5,
  forc_conv_thr = 1.0d-4,
/

  ibrav = 4, a = 3.53, b = 3.53, c = 20.0, cosAB = -0.5, cosAC =  
0.0, cosBC = 0.0,

  nat = 6, ntyp = 2,
  ecutwfc = 80,
  ecutrho = 400,
  nbnd = 40,
  occupations = 'fixed',
/
 
  mixing_beta = 0.3,
  conv_thr = 1.0d-9,
  electron_maxstep = 900,
efield_cart(1) = 0.0d0,
efield_cart(2) = 0.0d0,
efield_cart(3) = 0.0108d0,
/

ion_dynamics = 'bfgs',
/
ATOMIC_SPECIES
Pd  106.42  Pd.pbe-mt_fhi.UPF
S    32.06  S.pbe-mt_fhi.UPF
ATOMIC_POSITIONS  angstrom
Pd    0.    0.    0.
S 1.76627922    1.01975766    1.28191684
S 1.76627922   -1.01975766   -1.28191684
Pd    0.    0.    6.6000
S 1.76627922    1.01975766    7.88191684
S 1.76627922   -1.01975766    5.31808316
K_POINTS AUTOMATIC
16 16  4  0  0  0
 -
Dr. Sohail AhmadDepartment of PhysicsKing Khalid UniversityAbha,  
Saudi  
Arabia


--
Dr. rer. nat. Thomas Brumme
Wilhelm-Ostwald-Institute for Physical and Theoretical Chemistry
Leipzig University
Phillipp-Rosenthal-Strasse 31
04103 Leipzig

Tel:  +49 (0)341 97 36456

email: thomas.bru...@uni-leipzig.de


___
users mailing list
users@lists.quantum-espresso.org
https://lists.quantum-espresso.org/mailman/listinfo/users

  1   2   >